Vous êtes sur la page 1sur 86

EgyptianPediatrics

Yahoo Group
http://health.groups.yahoo.com/group/
EgyptianPediatrics/
Editor-in-Chief: Lawrence F. Nazarian, Rochester, NY
Associate Editors: Tina L. Cheng, Baltimore, MD
Joseph A. Zenel, Sioux Falls, SD
Editor, In Brief: Henry M. Adam, Bronx, NY
Consulting Editor, In Brief: Janet Serwint, Baltimore, MD
Editor, Index of Suspicion:
contents
Deepak M. Kamat, Detroit, MI
Consulting Editor Online and Multimedia
Projects: Laura Ibsen, Portland, OR
PediatricsinReview姞 Vol.32 No.2 February 2011
Editor Emeritus and Founding Editor:
Robert J. Haggerty, Canandaigua, NY
Managing Editor: Luann Zanzola
Medical Copy Editor: Deborah K. Kuhlman
Articles
Editorial Assistants: Sydney Sutherland, Kathleen Bernard
Editorial Office: Department of Pediatrics
University of Rochester
47 The Common Cold and Decongestant Therapy
Diane E. Pappas, J. Owen Hendley
School of Medicine & Dentistry
601 Elmwood Avenue, Box 777
Somatization Disorders:
Rochester, NY 14642
kbernard@aap.org
Editorial Board
56 Diagnosis, Treatment, and Prognosis
Tomas Jose Silber
Hugh D. Allen, Columbus, OH Jacob Hen, Bridgeport, CT

65
Margie Andreae, Ann Arbor, MI Hal B. Jenson, Springfield, MA
Richard Antaya, New Haven, CT Donald Lewis, Norfolk, VA
Denise Bratcher, Kansas City, MO
George R. Buchanan, Dallas, TX
Gregory Liptak, Syracuse, NY
Susan Massengill, Charlotte, NC
Focus on Diagnosis: Urine Electrolytes
Brian Carter, Nashville, TN Jennifer Miller, Gainesville, FL J. Bryan Carmody
Joseph Croffie, Indianapolis, IN Blaise Nemeth, Madison, WI
B. Anne Eberhard, New Hyde Park, NY Renata Sanders, Baltimore, MD
Philip Fischer, Rochester, MN Thomas L. Sato, Milwaukee, WI Ethics for the Pediatrician:
Rani Gereige, Miami, FL
Lindsey Grossman, Springfield, MA
Patricia Hamilton, London, United Kingdom
Publisher: American Academy of Pediatrics
Sarah E. Shea, Halifax, Nova Scotia
Andrew Sirotnak, Denver, CO
Nancy D. Spector, Philadelphia, PA 69 Ethical Issues in Organ Donation
Robert Macauley
Michael J. Held, Director, Division of Scholarly Journals and Professional Periodicals
Pediatrics in Review姞 Research and Statistics:
(ISSN 0191-9601) is owned and controlled by the American Academy of Distinguishing Statistical Significance from
Pediatrics. It is published monthly by the American Academy of Pediatrics, 141
Northwest Point Blvd., Elk Grove Village, IL 60007-1098
Statements and opinions expressed in Pediatrics in Review威 are those of the authors
and not necessarily those of the American Academy of Pediatrics or its Committees.
Recommendations included in this publication do not indicate an exclusive course
73 Clinical Importance: The Value of the P Value
Sharon R. Ghazarian
of treatment or serve as a standard of medical care.
Subscription price for 2010 for print and online/online only: AAP Fellow $172/
$131; AAP Candidate Fellow $161/$120; Nonmember $215/$167; Allied
Health or Resident $160/$108. Institutions call for pricing (866-843-2271). For
Index of Suspicion
overseas delivery, add $95. Current single issue price is $10 domestic, $12
international. Replacement issues must be claimed within 6 months from the date
Case 1: Nonbilious Projectile Vomiting in a Neonate
of issue and are limited to three per calendar year.
Periodicals postage paid at ARLINGTON HEIGHTS, ILLINOIS and at Case 2: Fever, Rash, Headache, Myalgia, and
additional mailing offices.
Arthralgia in an 11-year-old Boy
75
© AMERICAN ACADEMY OF PEDIATRICS, 2011. All rights reserved. Printed
in USA. No part may be duplicated or reproduced without permission of the
American Academy of Pediatrics. Case 3: Ecchymotic Lesions on the Backs of Asian Boys
POSTMASTER: Send address changes to PEDIATRICS IN REVIEW威, American
Academy of Pediatrics Customer Service Center, 141 Northwest Point Blvd., Elk Case 1: John D.A. Campagna, Minela Fernandez,
Grove Village, IL 60007-1098. Lance Yamashiroya
Pediatrics in Review Case 2: Ronen Zipkin, Marwa Moustafa, Shira Rosenberg
Print Issue Editorial Board Disclosures
The American Academy of Pediatrics (AAP) Policy on Disclosure of Financial Case 3: Ryan P. Flanagan, Neil Mullen
Relationships and Resolution of Conflicts of Interest for AAP CME Activities is
designed to ensure quality, objective, balanced, and scientifically rigorous AAP CME
activities by identifying and resolving all potential conflicts of interest before the
confirmation of service of those in a position to influence and/or control CME content.
All individuals in a position to influence and/or control the content of AAP CME
In Brief
activities are required to disclose to the AAP and subsequently to learners that the
individual either has no relevant financial relationships or any financial relationships with
the manufacturer(s) of any commercial product(s) and/or provider(s) of commercial
services discussed in CME activities. Commercial interest is defined as any entity
82 Cataracts
Karen M. Davenport, Archana A. Patel
producing, marketing, reselling or distributing health-care goods or services consumed
by, or used on, patients.
Each of the editorial board members, reviewers, question writers, PREP Coordinating
Committee members and staff has disclosed, if applicable, that the CME content he/
she edits/writes/reviews may include discussion/reference to generic pharmaceuticals,
84 Balance and Vertigo in Children
Jacques Benun
off-label pharmaceutical use, investigational therapies, brand names, and manufacturers.
None of the editors, board members, reviewers, question writers, PREP Coordinating
Committee members, or staff has any relevant financial relationships to disclose, unless
noted below. The AAP has taken steps to resolve any potential conflicts of interest.
Disclosures
86 Issues in Chemotherapy
Elizabeth A. Van Dyne
● Richard Antaya, MD, FAAP, disclosed that he participates in Astellas Pharma, US,
Inc., clinical trials, speaker bureau and advisory board; and that he participates in the
Novartis speaker bureau.
● Athos Bousvaros, MD, MPH, FAAP, disclosed that he has research grants from Merck
and UCB; and that he is a paid consultant and on the speaker bureau for Millennium.
Internet-Only Articles
● David N. Cornfield, MD, FAAP, disclosed that he has National Institutes of
Health grants. Abstracts appear on page 81.
● Donald W. Lewis, MD, FAAP, disclosed that he is a consultant for and has a
research grant from Astra Zeneca and Merck; and that he has research grants
from Ortho McNeil, Lilly, Bristol-Myers Squibb, GlaxoSmithKline, and
Boehringer Ingelheim Pharmaceutical.
Complementary, Holistic,
● Blaise Nemeth, MD, MS, FAAP, has disclosed he has an unrestricted
and Integrative Medicine:
educational grant for fellowship from Biomet.
● Janet Serwint, MD, FAAP, disclosed that she receives a research grant from
the Maternal and Child Health Bureau.
● Richard Sills, MD, FAAP, disclosed that he receives a research grant from Novartis.
e18 Therapies for Learning Disabilities
Elaine Z. Galicia-Connolly, Larissa Shamseer, Sunita Vohra

e25 Global Child Health


Donna Denno
Pediatrics in Review威 is supported, in part, through an
educational grant from Abbott Nutrition, a division of
Abbott Laboratories, Inc.

CME Statements:
Cover: The artwork on the cover of this The American Academy of Pediatrics (AAP) is accredited by the Accreditation Council for Continuing Medical Education (ACCME) to
provide continuing medical education for physicians.
month’s issue is by one of the winners of The AAP designates this journal-based CME activity for a maximum of 36 AMA PRA Category 1 CreditsTM. Physicians should claim only
the credit commensurate with the extent of their participation in the activity.
our 2009 Cover Art Contest, 8-year-old This activity is acceptable for a maximum of 36 AAP credits. These credits can be applied toward the AAP CME/CPD* Award available
to Fellows and Candidate Members of the AAP.
Erica G of Everett, WA. Erica’s pediatri- The American Academy of Physician Assistants accepts AMA PRA Category 1 CreditsTM from organizations accredited by the ACCME.
This program is approved for 36 NAPNAP CE contact hours; pharmacology (Rx) contact hours to be determined per the National
cian is Carolyn Sherman, MD. Association of Pediatric Nurse Practitioners (NAPNAP) Continuing Education Guidelines.
*Continuing Professional Development
How to complete this activity
Pediatrics in Review can be accessed and reviewed in print or online at http://pedsinreview.aappublications.org. Learners can claim credit
monthly online or submit their scannable answer sheet for credit upon completion of the 12-month activity. A CME scannable answer sheet
for recording your quiz answers can be found bound in the January 2011 issue. The deadline for submitting the 2011 answer sheet for this
activity is December 31, 2013. Credit will be recorded in the year in which it is submitted. It is estimated that it will take approximately
3 hours to complete each issue. This activity is not considered to have been completed until the learner documents participation in that
activity to the provider via online submission of answers or submission of the answer sheet. Course evaluations will be requested online and
Answer Key: 1. D; 2. D; 3. D; 4. E; 5. D; 6. B; 7. B; 8. A in print.
The Common Cold and Decongestant Therapy
Diane E. Pappas and J. Owen Hendley
Pediatr. Rev. 2011;32;47-55
DOI: 10.1542/pir.32-2-47

The online version of this article, along with updated information and services, is
located on the World Wide Web at:
http://pedsinreview.aappublications.org/cgi/content/full/32/2/47

Pediatrics in Review is the official journal of the American Academy of Pediatrics. A monthly
publication, it has been published continuously since 1979. Pediatrics in Review is owned,
published, and trademarked by the American Academy of Pediatrics, 141 Northwest Point
Boulevard, Elk Grove Village, Illinois, 60007. Copyright © 2011 by the American Academy of
Pediatrics. All rights reserved. Print ISSN: 0191-9601. Online ISSN: 1526-3347.

Downloaded from http://pedsinreview.aappublications.org. Provided by Pakistan:AAP Sponsored on February 1, 2011


Article respiratory

The Common Cold and Decongestant Therapy


Diane E. Pappas, MD, JD,*
Objectives After completing this article, readers should be able to:
J. Owen Hendley, MD*
1. Discuss the clinical presentation, diagnosis, and complications of the common cold in
children.
Author Disclosure 2. Describe the viral causes and pathogenesis of the common cold.
Drs Pappas and 3. Describe transmission of the common cold.
Hendley have 4. Explain the systemic effects of oral decongestants and antihistamines in infants and
disclosed no financial young children.
relationships relevant 5. Recognize that over-the-counter cough and cold preparations have not been ade-
to this article. This quately studied in children younger than 6 years of age and are not recommended.
commentary does 6. List the active ingredients and potential toxicities of over-the-counter cough and cold
contain a discussion medications.
of an unapproved/
investigative use of a
commercial
Introduction
The common cold heralds the beginning of the fall and winter seasons for pediatricians.
product/device.
Almost every ill patient presents with the runny nose, cough, and congestion that are the
hallmarks of the common cold. Although colds are self-limited, most patients (and their
parents) are tired and uncomfortable as a result of these symptoms. No effective treatments
can be prescribed or recommended beyond ordinary supportive care.

Clinical Presentation
Children typically present with cough, sneezing, nasal congestion, and runny nose. Nasal
discharge may be clear initially but often turns yellow-green within a few days. Fever may
be present initially in preschool-age patients, but vomiting and diarrhea are uncommon.
Parents also may report sleep disturbance and increased fatigue. Symptoms persist for at
least 10 days in most children but should begin to lessen by this time. This clinical picture
differs substantially from colds in adults, which present with the typical nasal discharge,
cough, and congestion but no fever and a usual duration of only 5 to 7 days. Sore throat
or hoarseness also may be present in children and adults. (1)

Diagnosis
Common cold is a clinical diagnosis. Subjective complaints may include nasal stuffiness,
sore throat, and headache. Objective findings are few but may include fever, anterior
cervical lymphadenopathy, erythema of the nasal mucosa and oropharynx, and nasal
discharge. Laboratory tests are not helpful; commercially available rapid tests are available
for detection of respiratory syncytial virus (RSV) and influenza. Other conditions to
consider in the diagnosis include nasal foreign body, allergic rhinitis, vasomotor rhinitis,
bacterial sinusitis, rhinitis medicamentosa, and structural abnormalities of the nose or
sinuses. History and physical examination should be sufficient to differentiate these
conditions from the common cold.

Complications
Secondary bacterial infections and wheezing may complicate the common cold. Bacterial
infections include otitis media, sinusitis, and pneumonia. About 30% of colds in preschool-
age children may be complicated by otitis media, (2) and this risk is highest in children 6 to
11 months of age. Sinusitis may occur in 5% to 10% of children who have colds and may be

*Department of Pediatrics, University of Virginia, Charlottesville, VA.

Pediatrics in Review Vol.32 No.2 February 2011 47


Downloaded from http://pedsinreview.aappublications.org. Provided by Pakistan:AAP Sponsored on February 1, 2011
respiratory common cold

considered when symptoms are not improving after occur. Cold season begins in September after children
10 days, although diagnostic criteria and the benefit of are back in school, at which time their frequency sharply
antimicrobial therapy are controversial. (3)(4) Other po- elevates and remains at a constant level until spring
tential complications include peritonsillar cellulitis and returns. This epidemic results not from a single cold virus
abscess, conjunctivitis, mastoiditis, and meningitis. but from a number of viruses moving through the com-
Infants and children who have histories of reactive munity during the fall and winter seasons. In the early
airway disease or asthma are at particular risk for compli- fall, rhinovirus begins to increase; parainfluenza viruses
cations, (5) most importantly for increased severity and follow in late fall, with RSV and coronavirus infections
duration of respiratory symptoms. As many as 50% of increasing during the winter months. Cold season con-
asthma exacerbations in children are associated with viral cludes with a final surge of rhinovirus infections in the
infections, especially rhinovirus. Children who have ele- spring. Only a few rhinovirus and enterovirus infections
vated immunoglobulin E concentrations and rhinovirus are still present in the community in the summer.
infection experience more severe respiratory symptoms Colds are most common in children younger than the
than do other children. Studies in adults suggest that an age of 6 years, who routinely experience six to eight colds
impaired cellular response to rhinovirus infection results annually. This frequency may result from susceptibility
in increased viral replication, leading to severe and pro- due to lack of previous exposure as well as from the
longed symptoms. RSV also is associated with wheezing natural attributes of childhood, namely, curious explora-
exacerbations. tion of the world with concomitant poor hygiene. Child
care attendance increases the number of colds experi-
Causes enced by young children as a result of repeated exposure
Rhinoviruses cause at least 50% of the colds in children to other children. By the teenage years, the frequency
and adults and, thus, are the most common sources of decreases to four to five colds every year, with parents of
cold infections. Other causes of the common cold in- young children experiencing only three to four colds
clude adenoviruses, influenza viruses, enteroviruses, annually. Adults who live with young children experience
RSV, and coronaviruses. Cold viruses are not part of the more colds than other adults living without young chil-
normal human flora but are transferred from person to dren in the home.
person and cause the appearance of symptoms 1 to 2 days
after inoculation. Pathogenesis
Some of these viral infections may present as a com- Viral infection of the nasopharyngeal mucosa does not
mon cold or as a more specific syndrome. For example, cause the symptoms of the common cold directly, instead
RSV infection in older children and adults typically pre- initiating a host inflammatory response that produces the
sents the same as any other cold, but RSV may produce symptoms. Cold virus is deposited on the mucosa of the
bronchiolitis involving the lower respiratory tract in in- nose or conjunctivae. Virus then attaches to receptors on
fants and toddlers and can produce a severe presentation. cells in the nasopharynx and enters the cells. Only a small
Similarly, infection with parainfluenza viruses may number of cells become infected. The infected cells
present as croup in younger children and as a typical cold release potent cytokines, including interleukin (IL)-8,
in the older child. Adenoviral infection may present as a which is a chemoattractant for polymorphonuclear cells
common cold or as pharyngoconjunctival fever, with (PMNs). PMNs accumulate in large numbers in the nasal
injected palpebral conjunctivae, watery eye discharge, secretions. Vascular permeability increases and plasma
and erythema of the oropharynx in addition to the usual proteins, including albumin and bradykinin, leak into the
fever and upper respiratory tract symptoms. Enterovi- nasal secretions, increasing the volume of secretions pro-
ruses may produce aseptic meningitis. Coxsackievirus A, duced.
an enterovirus, may cause herpangina, with fever and Bradykinin can cause rhinitis and sore throat, which
ulcerated papules on the posterior oropharynx. Infection may contribute further to the discomfort caused by the
with influenza viruses may present as a febrile respiratory cold. Mucociliary clearance is slowed. Histamine concen-
illness involving the lower respiratory tract, fatigue, and trations do not increase during the course of the com-
muscle aches. mon cold. The nasal mucosa is not destroyed during
rhinovirus and coronavirus infections, but adenovirus
Epidemiology and influenza viruses do destroy the nasal mucosa. Symp-
The occurrence of the common cold is predictable in tom severity correlates with IL-8 concentration over the
terms of who is most affected and when colds usually course of the infection.

48 Pediatrics in Review Vol.32 No.2 February 2011


Downloaded from http://pedsinreview.aappublications.org. Provided by Pakistan:AAP Sponsored on February 1, 2011
respiratory common cold

Infected cells are extruded and washed away with the Transmission
secreted inflammatory mediators, preventing spread of There are three proposed mechanisms for transmission of
the virus to nearby cells. Viral replication declines, the the common cold: small particle aerosols produced from
inflammatory response decreases, and symptoms begin coughing that are inhaled by another person, large par-
to subside. Although symptoms are improved, virus still ticle droplets produced from saliva expelled during a
can be recovered from the nasopharynx for at least sneeze that land on the conjunctivae or nasal mucosa of
2 weeks after inoculation. Finally, after 2 to 3 weeks, another person, or self-inoculation of one’s own con-
adequate neutralizing antibody is available to end the junctivae or nasal mucosa after touching a person or
infection. object contaminated with cold virus. In the experimental
As shown in healthy adults who have experimentally setting, sneezing (large-particle aerosol) has been shown
induced rhinovirus colds, bradykinins and PMNs accu- to be a very inefficient method of transmission of rhino-
mulate in nasal secretions at the onset and for the dura- virus. (6) Small-particle aerosol transmission of rhinovi-
tion of cold symptoms. It is believed that the presence of rus has been shown to occur but also appears to be
PMNs in the nasal secretions, as well as their enzymatic inefficient. (7)
activity, may be the source of the yellow-green color Substantial evidence from the experimental setting
typical of the nasal discharge of the common cold. The suggests that rhinovirus can be transmitted efficiently via
nasal mucosa in children has more secretory capacity than self-inoculation. Rhinovirus is excreted in nasal secre-
that in adults, which may contribute to significant nasal tions but is only present minimally in saliva. Fingers and
discharge for days. hands are frequently contaminated with rhinovirus, as are
Viral rhinosinusitis is a frequent finding during the telephones and other everyday objects. (8) In experimen-
course of the common cold. Imaging of the paranasal tal settings, hand-to-hand transfer and hand-to-surface-
sinuses in children who have uncomplicated colds dem- to-hand transfer have been shown to be feasible mecha-
onstrates abnormalities of the paranasal sinuses in about nisms for transfer of rhinovirus to susceptible individuals.
two thirds of children, most commonly the maxillary and Once the hands are contaminated with cold virus, self-
ethmoidal sinuses. Significant resolution occurs within inoculation readily occurs when a person touches his own
2 weeks. This finding implies that accumulation of fluid nose or eyes with the contaminated hand or fingers. (6)(9)
in sinus cavities may be part of the cold and is not Self-inoculation also appears to be an effective
diagnostic of a bacterial infection. It is not known if sinus method of rhinovirus transmission in the home environ-
involvement results from actual viral infection of the ment because secondary transmission in the home can be
sinus mucosa or from impaired mucus clearance. reduced if self-inoculation is interrupted. In one study,
Abnormal middle ear pressures also occur commonly mothers whose fingertips were treated with virucidal 2%
during the course of illness in both adults and children. aqueous iodine were much less likely to become infected
Abnormal middle ear pressures may be present in up to than mothers whose fingertips were treated with placebo.
two thirds of school-age children during the course of a (9) Similarly, the use of virucidal tissues to interrupt viral
cold. This effect occurs most frequently during the first transfer has been shown to decrease secondary transmis-
several days of illness and resolves within 2 to 3 weeks. sion of colds in the home modestly. (10)
Influenza viruses and coronaviruses can be transmit-
Immunity ted by small-particle aerosol. RSV is not transmitted by
The frequency of colds may be due either to their lack of small-particle aerosol but has been shown to be transmis-
producing lasting immunity or because there are so many sible by large-particle aerosol. RSV and rhinoviruses are
serotypes that immunity to some viral strains has no real not transmissible by oral inoculation.
impact. Viruses that do not produce lasting immunity
after infection include RSV, parainfluenza viruses, and Treatment
coronaviruses, resulting in an individual possibly suffer- Although much desired, effective treatments for the
ing recurrent infection with these same agents. Other common cold remain elusive. Over-the-counter (OTC)
viruses, such as rhinoviruses, adenoviruses, influenza vi- cough and cold medications are readily available for
ruses, and enteroviruses, do produce lasting immunity, children and are sold in various combinations to address
but there are so many serotypes that this immunity has no symptoms. Antihistamines, antitussives, expectorants,
real impact on reducing the frequency of cold infections. decongestants, and antipyretics/analgesics are com-
As a result, an effective vaccine for the common cold is monly sold in combinations. Every week, more than 10%
unlikely. of children in the United States are treated with a cough

Pediatrics in Review Vol.32 No.2 February 2011 49


Downloaded from http://pedsinreview.aappublications.org. Provided by Pakistan:AAP Sponsored on February 1, 2011
respiratory common cold

and cold medication, and most of these preparations are Antibiotics


multiple-ingredient products. Symptomatic relief is the pri- The common cold is a viral infection, and there is no role
mary goal of treatment, although preventing disease spread for antibiotics. Antibiotics cannot treat the underlying
and reducing the likelihood of secondary bacterial infec- infection and will not decrease the likelihood of bacterial
tions are also considerations. However, little scientific evi- complications. Antibiotics are indicated only when sec-
dence supports their use, and there is increasing evidence of ondary bacterial complications are diagnosed.
potential adverse effects that may result from their use.
In the past few years, the use of OTC cough and cold Antiviral Agents
medications for children and infants has been under Except in the case of influenza, no antiviral drugs are
intense scrutiny. A total of 123 deaths in children available to treat the common cold. For influenza, a
younger than the age of 6 years have resulted from OTC number of antiviral agents are available and provide
cough and cold medications in the past 20 years. OTC modest reduction in symptoms and duration, including
medications also are a common cause of emergency oseltamivir, amantadine, rimantadine, and zanamivir.
department visits because of adverse effects or accidental These medications inhibit release of virus from infected
ingestions. Poison control centers reported more than cells.
750,000 calls related to the use of OTC cough and cold
medications since 2000. Antihistamines (H1 Receptor Blockers)
The risk for accidental overdose and adverse effects is First-generation antihistamines are common ingredients
likely the result of many factors. Dosing guidelines for in OTC cough and cold medications. Triprolidine, di-
children have not been established but are extrapolated phenhydramine, hydroxyzine, and chlorpheniramine are
from adult data. Parents may be easily confused by prod- all in this class of medications. These medications are
uct labeling. The availability of multi-ingredient prod- well-absorbed, with onset of action within 15 to 30 min-
ucts may increase the likelihood of inadvertent overdose utes and duration of action of 3 to 6 hours or more.
because parents may not understand what they have Because these medications are also anticholinergic, they
given to their child. Finally, multiple caregivers for young decrease mucus secretion. In addition, anticholinergic
children may increase the risk of accidental overdose and action may result in dry mouth, blurred vision, and
adverse reactions further. urinary retention. Gastrointestinal upset may occur. Car-
As a result of a citizen petition filed in March 2007, diac effects have been reported, including tachycardia,
the United States Food and Drug Administration (FDA) prolongation of the QTc interval, heart block, and ar-
initiated a review of the safety and effectiveness of cough rhythmias. Central nervous system (CNS) effects such as
and cold medications for children. In October 2007, sedation, paradoxic excitability, respiratory depression,
FDA advisers voted to recommend that OTC cough and and hallucinations may result, especially in cases of over-
cold medications not be used for treatment of children dose. Dystonic reactions have been reported.
younger than the age of 2 years. Since then, the number Second-generation antihistamines (egs, terfenadine,
of emergency department visits for adverse events related astemizole, loratadine, and cetirizine) lack anticholin-
to the use of cough and cold medications in children ergic activity. These medications have fewer CNS effects
younger than 2 years of age has decreased by more than than first-generation antihistamines. Cardiac effects, al-
half. (11) Drug manufacturers voluntarily discontinued though rare, may occur, including prolonged QT inter-
marketing these products for children younger than age val, ventricular arrhythmia, and heart block.
2 years. The FDA has since issued a public health advisory In adults, several studies show that first-generation
recommending that cough and cold medications not be antihistamines (chlorpheniramine) provide some symp-
used in children younger than age 2 years. FDA advisory tomatic relief of cold symptoms, specifically, decreased
committees have voted to ban OTC cough and cold sneezing and increased mucociliary clearance. Another
medications for use in children younger than age 6 years. study showed decreased nasal discharge and duration of
FDA review of this recommendation is underway. Some symptoms when first-generation antihistamines (chlor-
manufacturers have changed the labeling of their prod- pheniramine) were used to treat adults who had the
ucts to recommend against their use in children younger common cold.
than the age of 4 years. (12) The American Academy of There are few studies of antihistamine use in children.
Pediatrics recommends against the use of OTC cough (14) In one study, children treated with an antihistamine-
and cold medications in children younger than the age of decongestant combination (brompheniramine maleate/
6 years. (13) phenylpropanolamine hydrochloride) showed no improve-

50 Pediatrics in Review Vol.32 No.2 February 2011


Downloaded from http://pedsinreview.aappublications.org. Provided by Pakistan:AAP Sponsored on February 1, 2011
respiratory common cold

ment in cough over placebo other than the treated children tis medicamentosa) when the medication is discontin-
being more likely to be asleep 2 hours after receiving med- ued.
ication. A study of clemastine treatment in children who No information supports the effectiveness of either oral
had colds showed no decrease in nasal discharge, although or topical decongestants in children who have colds. In
the color of the nasal discharge changed from yellow to children ages 6 months to 5 years, one study of a
white. In another study, diphenhydramine at bedtime was decongestant/antihistamine combination (phenyl-
no better than placebo or dextromethorphan in reducing propanolamine/brompheniramine) compared with pla-
cough or improving sleep. Few adverse effects are reported cebo found no improvement in nasal congestion, rhinor-
from first-generation antihistamine use in children. rhea, or cough. (15) In a study of children 6 to 18 months
of age treated with topical phenylephrine, neither nasal
obstruction nor abnormal middle ear pressures improved.
Decongestants (16) In another study in children, xylometazoline had no
Oral sympathomimetic decongestants are available and effect on eustachian tube function. (17) Because infants are
are common ingredients in OTC cough and cold medi- obligate nose-breathers, there is much concern about the
cations. A significant proportion of children (1 in potential dangers of these agents. Deaths in infants and
20) have taken pseudoephedrine in any given week, with young children treated with pseudoephedrine have been
the greatest use in children younger than the age of 2 reported.
years (1 in 12). Oral decongestants include pseudo-
ephedrine, phenylpropanolamine, and phenylephrine. Antitussives
These compounds are vasoconstrictors, acting on adren- Although cough is a protective action that clears airway
ergic receptors to decrease blood flow in the mucosa. secretions to maintain airway patency, it remains one of
Unfortunately, not only the nasal mucosa is affected; the the most bothersome symptoms of the common cold.
use of oral decongestants also results in generalized Suppressing cough effectively may actually be harmful
vasoconstriction with a resultant increase in blood pres- for some children, especially the child who has asthma
sure. Other adverse effects include headaches, seizures, and is unable to clear his or her airway. Nevertheless, a
nausea, vomiting, decreased appetite, agitation, tachy- multitude of OTC preparations are available purporting
cardia, nervousness, irritability, dystonia, and dysrhyth- to suppress cough in children.
mias. One of the most common antitussive ingredients is
Evidence in adults shows that both pseudoephedrine dextromethorphan, a narcotic analog that does not have
and phenylpropanolamine effectively reduce nasal symp- CNS effects unless excessive doses are used. Dextro-
toms, including nasal congestion and sneezing. Phenyl- methorphan is well tolerated at therapeutic doses, with
ephrine undergoes extensive biotransformation, result- few adverse effects. Unfortunately, there are few studies
ing in variable bioavailability, which limits its usefulness. of antitussives in children. In one study of children 18
Increasing governmental control has greatly limited months to 12 years of age, there was no difference in
the availability of oral decongestants. Phenylpropanol- cough between the placebo-, dextromethorphan-, or
amine has been associated with intracranial hemorrhage codeine-treated groups. Furthermore, cough decreased
and was removed from the market in 2000 after being in all three groups after 3 days. (18) Another study in
classified as “unsafe” by the FDA. As a result of the 2005 children ages 2 to 18 years who had acute cough treated
Combat Methamphetamine Act and potential for abuse with dextromethorphan found no improvement in
of the agent, products containing pseudoephedrine are cough with increasing dose. (19)
now kept behind pharmacy counters with monitoring Accidental overdose can cause respiratory depression,
who is purchasing such products. and there are reports of death in infants younger than 12
Topical decongestants act on the adrenergic receptors months of age after dextromethorphan ingestion. In one
in the nasal mucosa to cause vasoconstriction, causing series of accidental ingestions in children younger than 5
nasal tissues to shrink. Onset is rapid, within a few years old (average age, 28 months), the children re-
minutes, and may last several hours. Systemic absorption mained stable hemodynamically, although some experi-
is minimal. Common topical decongestants include enced sedation. Because of the lack of efficacy and the
oxymetazoline, xylometazoline, and phenylephrine. risk of serious adverse effects, the American Academy of
These topical agents can reduce nasal congestion in Pediatrics does not recommend the use of dextrometho-
adults, but their usefulness is limited to only a few days rphan in children. (20)
because of their potential for rebound congestion (rhini- Dextromethorphan can have serious CNS effects, es-

Pediatrics in Review Vol.32 No.2 February 2011 51


Downloaded from http://pedsinreview.aappublications.org. Provided by Pakistan:AAP Sponsored on February 1, 2011
respiratory common cold

pecially when ingested in large doses. Once ingested, larly susceptible to these effects and the subsequent
absorption from the gastrointestinal tract occurs quickly. development of apnea. Infants and young children may
Dextromethorphan crosses the blood-brain barrier, be at risk for toxicity due to their immature hepatic
where it blocks serotonin uptake while stimulating sero- enzyme systems because codeine is inactivated by conju-
tonin release. These effects may result in a serious adverse gation in the liver. Drug clearance is also reduced in
reaction called serotonin syndrome, characterized by infants and may increase the risk of toxic effects. Infants
autonomic instability, altered mental status, and neuro- who have impaired lung function (such as those who
muscular abnormalities. Dextromethorphan is also an have bronchiolitis) may have increased susceptibility to
N-methyl-d-aspartate receptor antagonist and inhibits respiratory depression. Death due to accidental overdose
the neurotransmitter glutamate, which can cause a disso- in young infants has been reported. Codeine may cause
ciative state and hallucinations. Dextromethorphan is CNS depression that is reversible with naloxone. Adverse
metabolized via the cytochrome P450 pathway in the effects in children treated with appropriate doses of co-
liver. Some people are “slow metabolizers” and have deine include nausea, vomiting, constipation, and dizzi-
difficulty metabolizing dextromethorphan due to ge- ness.
netic differences. Due to cross-reactivity, rapid immuno- First-generation antihistamines are also marketed to
assay drug screens may incorrectly report excessive dex- suppress cough. Studies in patients who have chronic
tromethorphan ingestion as phencyclidine. cough have reported a decrease in cough frequency when
In recent years, dextromethorphan has increasingly doses causing sedation are employed. These medications
become a drug of abuse for adolescents and young adults may cause thickening of bronchial secretions, and some
because of its ability to produce intoxication, hallucina- believe that their use is contraindicated in children who
tions, and dissociation. DXM, Dex, Skittles, Robo, and have acute wheezing or asthma. Studies in children of
Triple-c are slang terms for dextromethorphan. A num- antihistamine treatment alone or of decongestant/
ber of factors make dextromethorphan an attractive antihistamine combinations have found no improvement
choice for abuse: it is readily available OTC at drug and in cough when compared with placebo.
grocery stores, it is legal to purchase, it is inexpensive, A recent study suggests that honey may provide some
and it is considered harmless. It is also available in pow- relief from nighttime cough in children who have colds.
der form over the internet. With mild intoxication, dex- (22) In this study, a bedtime dose of honey was better
tromethorphan causes a mild stimulant effect that in- than no treatment in children who had cough from colds.
volves euphoria, stupor, and hyperexcitability. Other The generalizability of the results of this single study is
effects may include diaphoresis, nausea, vomiting, nys- limited. Honey is not recommended for children
tagmus, and mydriasis. At higher doses, dextromethor- younger than age 12 months because of the risk of
phan can cause hallucinations, delusions, an ataxic gait, exposure to botulinum spores.
and somnolence. At extreme doses, dextromethorphan
may cause a dissociative state, with paranoia, coma, and
Expectorants
death. (21) The reports in the literature of death in
Expectorants are medications intended to increase mu-
otherwise healthy adolescents and young adults as a
cus production. Guaifenesin is the most commonly avail-
result of dextromethorphan abuse is increasing.
able expectorant. In young adults who have colds, treat-
Codeine often is used as an antitussive. Codeine is a
ment with guaifenesin failed to decrease cough
narcotic that is believed to act centrally on the cough
frequency, but patients did report subjective improve-
center. It is also a mild analgesic and sedative. Although
ment in thickness and quantity of sputum. (23) Other
codeine is the “gold standard” antitussive, no studies in
studies in adults suggest that guaifenesin may reduce
adults or children support the antitussive properties
cough frequency. Studies in children demonstrating ef-
of codeine for treatment of cough associated with the
fectiveness do not exist.
common cold. In fact, in one study of patients ages
18 months to 12 years, codeine was no more effective
than placebo for cough suppression in children who Analgesics/Antipyretics
had nighttime cough due to a cold. (18) The American Analgesics/antipyretics such as acetaminophen, aspirin,
Academy of Pediatrics recommends against the use of and ibuprofen may be useful for the fever and general
codeine in children who have cough. (20) discomfort of the common cold. However, both aspirin
Narcotics such as codeine may cause dose-dependent and acetaminophen suppress the neutralizing antibody
respiratory depression, and infants seem to be particu- response, which results in increased nasal symptoms and

52 Pediatrics in Review Vol.32 No.2 February 2011


Downloaded from http://pedsinreview.aappublications.org. Provided by Pakistan:AAP Sponsored on February 1, 2011
respiratory common cold

prolonged viral shedding. The use of aspirin is not rec- Vaporizers


ommended in children because of its association with Efforts to establish steam inhalation as an effective treat-
Reye syndrome. ment for nasal congestion due to the common cold have
failed to demonstrate benefit. Some studies in adults have
Echinacea even shown that the duration and severity of symptoms
Echinacea is a common herbal therapy recommended for may increase after treatment with humidified air. Inhala-
treatment of the common cold, but few well-controlled tion of steam has not been shown to increase nasal
studies have evaluated its safety and effectiveness. The patency, although many people report subjective im-
most complete and well-controlled study to date dem- provement in nasal obstruction following inhalation. Be-
onstrated no effect of echinacea on severity of symptoms cause rhinoviruses replicate best at 33 to 34°C, inhala-
or rate of infection in adults. (24) tion of steam was hypothesized to reduce rhinovirus
replication, but steam did not reduce viral titers in nasal
secretions during rhinovirus infection.
Menthol
Menthol is a common ingredient in vapor therapies.
Objective evaluation of nasal resistance in adults before Saline Nose Drops/Bulb Suction
and after menthol inhalation showed no effect, although Saline drops with bulb suction are often used to moisten
patients did report a sense of improved air flow. In a the nasal mucosa and loosen secretions for removal from
recent study in school-age children, inhalation of men- infants and children. Adults may use saline nose sprays for
thol did not decrease cough or increase nasal patency, similar purposes. A recent study in school-age children
but patient perception of improved nasal patency was suggests that nasal symptoms and sore throat improve
reported. (25) In a recent study, bedtime application more quickly with daily saline washes. (28)
of a vapor rub (camphor, menthol, and eucalyptus oils)
to the chest and neck of children ages 2 to 11 years Prevention
resulted in symptomatic relief of nighttime cough, con- The best treatment of a cold is prevention. Annual influ-
gestion, and sleep difficulty when compared with petro- enza vaccination is recommended and is the only vaccine
latum or no treatment. (26) Chemical irritation of the available to prevent a respiratory viral infection. Hand-
nasal mucosa may result from topical treatment with washing effectively removes cold viruses from the hands.
menthol preparations. Gastrointestinal and CNS effects Virucidal tissues have been shown to reduce secondary
may result from accidental ingestion. transmission modestly in the home. Virucidal hand gels
are also available, but there are no published studies
Ipratropium Bromide evaluating their usefulness. Alcohol-based hand sanitiz-
Ipratropium bromide is a nasal spray that decreases nasal ers have not been shown to reduce secondary transmis-
discharge of the common cold via its anticholinergic sion of colds in the home or school environment, likely
activity. Unfortunately, its usefulness is limited to chil- because rhinovirus is not affected by these products.
dren older than age 5 years, and adverse effects include Limiting contact with one’s own nasal and conjunctival
nosebleeds, nasal dryness, and headache. mucosa can reduce self-inoculation.

Zinc References
Zinc often is proposed as a treatment for cold symptoms 1. Pappas DE, Hendley JO, Hayden FG, Winther B. Symptom
because in vitro studies have demonstrated that zinc profile of common colds in school-aged children. Pediatr Infect Dis
inhibits rhinovirus replication. Adverse effects include J. 2008;27:8 –11
nausea, bad taste, diarrhea, and mouth or throat irrita- 2. Revai K, Dobbs LA, Nair S, Patel JA, Grady JJ, Chonmaitree T.
Incidence of acute otitis media and sinusitis complicating upper
tion. Some studies in adults suggest that early treatment respiratory tract infection: the effect of age. Pediatrics. 2007;119:
with zinc gluconate can reduce the duration of cold e1408 – e1423
symptoms. Usefulness may be limited by the need for 3. Wald ER, Nash D, Eickhoff J. Effectiveness of amoxicillin/
frequent administration (5 to 6 times/day) and common clavulanate potassium in the treatment of acute bacterial sinusitis in
children. Pediatrics. 2009;124:9 –15
adverse effects, including bad taste and gastrointestinal
4. Garbutt JM, Goldstein M, Gellman E, Shannon W, Littenberg
upset. In schoolchildren (first through twelfth grade), B. A randomized, placebo-controlled trial of antimicrobial treat-
treatment with zinc lozenges did not decrease the dura- ment for children with clinically diagnosed acute sinusitis. Pediat-
tion of cold symptoms. (27) rics. 2001;107:619 – 625

Pediatrics in Review Vol.32 No.2 February 2011 53


Downloaded from http://pedsinreview.aappublications.org. Provided by Pakistan:AAP Sponsored on February 1, 2011
respiratory common cold

11. Shehab N, Schaefer MK, Kegler SR, Budnitz DS. Adverse


Summary events from cough and cold medications after a market withdrawal
of products labeled for infants. Pediatrics. 2010;126:1100 –1107
• Although colds are self-limited viral infections that 12. Kuehn BM. Debate continues over the safety of cold and
generally resolve in 10 to 14 days, they are a cough medicines for children. JAMA. 2008;300:2354 –2356
common cause of discomfort and distress for 13. American Academy of Pediatrics Urges Caution in Use of
children and their parents. Over-The-counter Cough and Cold Medicines [press release].
• Complications can occur, including secondary 2008. Accessed November 2010 at: http://www.aap.org/
bacterial infections and wheezing exacerbations. advocacy/releases/jan08coughandcold.htm
• Given the lack of proven benefit and the risk of 14. Smith SM, Schroeder K, Fahey T. Over-the-counter medica-
significant adverse effects, no prescription or OTC tions for acute cough in children and adults in ambulatory settings.
treatments are recommended for children; supportive Cochrane Database Syst Rev. 2008;1:CD001831
care remains the only recommended treatment. 15. Clemens CJ, Taylor JA, Almquist JR, Quinn HC, et al. Is an
• Education of parents should include the current antihistamine-decongestant combination effective in temporarily
recommendations against the use of cough and cold relieving symptoms of the common cold in preschool children?
medications in children younger than the age of 6 J Pediatr. 1997;464 – 466
years as well as the potential risks of such OTC 16. Turner RB, Darden PM. Effect of topical adrenergic decon-
treatments in children of all ages. gestants on middle ear pressure in infants with common colds.
• Although colds may be spread through large-particle Pediatr Infect Dis J. 1996;15:621– 624
aerosols, small-particle aerosols, and self- 17. van Heerbeek N, Ingels JAO, Zielhuis GA. No effect of a nasal
inoculation, increasing evidence suggests that self- decongestant on eustachian tube function in children with ventila-
inoculation is a common method of transmission in tion tubes. Laryngoscope. 2002;112:1115–1118
the home. (6)(9) 18. Taylor JA, Novack AH, Almquist JR, Rogers JE. Efficacy of
• Based on consensus, OTC cough and cold cough suppressants in children. J Pediatr. 1993;122:799 – 802
medications are not recommended for use in 19. Paul IM, Shaffer ML, Yoder KE, Sturgis SA, et al. Dose-
children younger than 6 years of age. (12) response relationship with increasing doses of dextromethorphan
• Based on moderate evidence, colds are a common for children with cough. Clin Ther. 2004;26:1508 –1514
trigger for asthma exacerbations in children. (5) 20. American Academy of Pediatrics Committee on Drugs. Use of
• Consensus evidence suggests that because of the codeine- and dextromethorphan-containing cough remedies in
many different serotypes of some cold viruses and children. Pediatrics. 1997;99:918 –920
other cold viruses not producing lasting immunity, it 21. Logan BK, Goldfogel G, Hamilton R, Kuhlman J. Five deaths
is unlikely that effective vaccines for the common resulting from abuse of dextromethorphan sold over the internet. J
cold will be developed. Analyt Toxicol. 2009;33:99 –103
22. Paul IM, Beiler J, McMonagle A, Shaffer ML, Duda L, Berlin
CM Jr. Effect of honey, dextromethorphan, and no treatment on
5. Rakes GP, Arruda E, Ingram JM, et al. Rhinovirus and respira- nocturnal cough and sleep quality for coughing children and their
tory syncytial virus in wheezing children requiring emergency care: parents. Arch Pediatr Adolesc Med. 2007;161:1140 –1146
IgE and eosinophil analyses. Am J Respir Crit Care Med. 1999;159: 23. Kuhn JJ, Hendley JO, Adams KF, Clark JW, Gwaltney JM Jr.
785–790 Antitussive effect of guaifenesin in young adults with natural colds:
6. Hendley JO, Wenzel RP, Gwaltney JM Jr. Transmission of objective and subjective assessment. Chest. 1982;82:713–718
rhinovirus colds by self-inoculation. N Engl J Med. 1973;288: 24. Turner RB, Bauer R, Woelkart K, Hulsey TC, Gangemi JD. An
1361–1364 evaluation of Echinacea angustifolia in experimental rhinovirus
7. Dick EC, Jennings LC, Mink KA, et al. Aerosol transmission of infections. N Engl J Med. 2005;353:341–348
rhinovirus colds. J Infect Dis. 1987;156:442– 448 25. Kenia P, Houghton T, Beardsmore C. Does inhaling menthol
8. Winther B, McCue K, Ashe K, Rubino JR, Hendley JO. Envi- affect nasal patency or cough? Pediatr Pulmonol. 2008;43:532–537
ronmental contamination with rhinovirus and transfer to fingers of 26. Paul IM, Beiler JS, King TS, Clap ER, et al. Vapor rub,
healthy individuals by daily life activity. J Med Virol. 2007;79: petrolatum, and no treatment for children with nocturnal cough
1606 –1610 and cold symptoms. Pediatrics. 2010;126:1092–1099
9. Gwaltney JM Jr, Hendley JO. Mechanisms of transmission of 27. Macknin ML, Piedmonte M, Calendine C, Janosky J, Wald E.
rhinovirus infections. Epidemiol Rev. 1988;10:242–258 Zinc gluconate lozenges for treating the common cold in children:
10. Farr BM, Hendley JO, Kaiser DL, Gwaltney JM. Two random- a randomized controlled trial. JAMA. 1998;279:1962–1967
ized, controlled trials of virucidal nasal tissues in the prevention of 28. Šlapak I, Skoupá J, Strnad P, Hornik P. Efficacy of isotonic
natural upper respiratory infections. Am J Epidemiol. 1988;128: nasal wash (seawater) in the treatment and prevention of rhinitis in
1162–1172 children. Arch Otolarngol Head Neck Surg. 2008;134:67–74

54 Pediatrics in Review Vol.32 No.2 February 2011


Downloaded from http://pedsinreview.aappublications.org. Provided by Pakistan:AAP Sponsored on February 1, 2011
respiratory common cold

PIR Quiz
Quiz also available online at http://pedsinreview.aappublications.org.

1. A 5-year-old boy is brought to your office because of clear nasal discharge, nasal congestion, sore throat,
and temperature of 38.2°C for 2 days. He appears well on physical examination, although his nasal mucosa
and pharynx are erythematous and he has yellowish nasal discharge. Which of the following is the most
likely etiologic agent causing his symptoms?
A. Adenovirus.
B. Haemophilus influenzae.
C. Respiratory syncytial virus.
D. Rhinovirus.
E. Streptococcus pneumoniae.

2. You are evaluating a 2-year-old girl who has a 3-day history of nasal congestion and clear rhinorrhea.
Findings on physical examination reveal erythema of the nasal mucosa and no other abnormalities. You
diagnose a simple viral upper respiratory tract infection. Which of the following is the most likely
complication of this condition?
A. Conjunctivitis.
B. Mastoiditis.
C. Meningitis.
D. Otitis media.
E. Reactive airway disease.

3. A mother brings in her 18-month-old daughter because of a runny nose and congestion for the sixth time
since she started child care 12 months ago. She is worried about the number of “colds” that her daughter
has had and is concerned that something else is wrong. The girl has grown well, and findings on physical
examination are normal except for mild nasal congestion and clear rhinorrhea. Which of the following is
the most likely reason for her recurrent symptoms?
A. Anatomic abnormality of the sinuses.
B. Bacterial colonization of her nasopharynx.
C. Immunoglobulin subclass deficiency.
D. Repetitive exposure to infected children who are coughing or sneezing.
E. Transmission of illness from child care staff.

4. A 3-year-old girl who has the acute onset of nasal congestion, sore throat, and cough presents to the
emergency department in the middle of the night because the cough is keeping her awake. Her parents are
upset because both they and the girl are losing sleep, and they request something to “get rid of the cough.”
Which of the following is the most appropriate recommendation at this time?
A. Oral codeine.
B. Oral dextromethorphan.
C. Oral diphenhydramine.
D. Topical (nasal) phenylephrine.
E. Topical (nasal) saline drops.

Pediatrics in Review Vol.32 No.2 February 2011 55


Downloaded from http://pedsinreview.aappublications.org. Provided by Pakistan:AAP Sponsored on February 1, 2011
The Common Cold and Decongestant Therapy
Diane E. Pappas and J. Owen Hendley
Pediatr. Rev. 2011;32;47-55
DOI: 10.1542/pir.32-2-47

Updated Information including high-resolution figures, can be found at:


& Services http://pedsinreview.aappublications.org/cgi/content/full/32/2/47
Subspecialty Collections This article, along with others on similar topics, appears in the
following collection(s):
Infectious Diseases
http://pedsinreview.aappublications.org/cgi/collection/infectious
_diseases Respiratory Disorders
http://pedsinreview.aappublications.org/cgi/collection/respirator
y_disorders
Permissions & Licensing Information about reproducing this article in parts (figures,
tables) or in its entirety can be found online at:
http://pedsinreview.aappublications.org/misc/Permissions.shtml
Reprints Information about ordering reprints can be found online:
http://pedsinreview.aappublications.org/misc/reprints.shtml

Downloaded from http://pedsinreview.aappublications.org. Provided by Pakistan:AAP Sponsored on February 1, 2011


Somatization Disorders: Diagnosis, Treatment, and Prognosis
Tomas Jose Silber
Pediatr. Rev. 2011;32;56-64
DOI: 10.1542/pir.32-2-56

The online version of this article, along with updated information and services, is
located on the World Wide Web at:
http://pedsinreview.aappublications.org/cgi/content/full/32/2/56

Pediatrics in Review is the official journal of the American Academy of Pediatrics. A monthly
publication, it has been published continuously since 1979. Pediatrics in Review is owned,
published, and trademarked by the American Academy of Pediatrics, 141 Northwest Point
Boulevard, Elk Grove Village, Illinois, 60007. Copyright © 2011 by the American Academy of
Pediatrics. All rights reserved. Print ISSN: 0191-9601. Online ISSN: 1526-3347.

Downloaded from http://pedsinreview.aappublications.org. Provided by Pakistan:AAP Sponsored on February 1, 2011


Article psychosocial

Somatization Disorders:
Diagnosis, Treatment, and Prognosis
Tomas Jose Silber, MD*
Objectives After completing this article, readers should be able to:

1. Identify the various manifestations of somatization disorders in children and


Author Disclosure adolescents.
Dr Silber has 2. Understand the association of psychosomatic disorders with personality traits and
disclosed no financial stressors such as physical and sexual abuse, bullying, parental anxiety, and pressure for
relationships relevant a child to perform.
to this article. This 3. Recognize secondary gains of somatization disorders.
commentary does not 4. Explain why it is necessary to focus on school attendance and the management of
contain a discussion school avoidance.
of an unapproved/ 5. Develop a partnership with patients and their parents to address the symptoms of a
investigative use of a somatization disorder.
commercial 6. Plan the treatment and management of somatization disorders
product/device.

Introduction
Children and adolescents suspected of having somatization disorders present a challenge
to pediatricians. Clinicians are often concerned about “missing something,” while also
worrying about alienating both the patient and the family when explaining the condition.
Many clinicians are baffled by the onslaught of symptoms, feel frustrated by the never-
ending recurrent complaints, or become annoyed by the time consumed in caring for
patients who might be perceived as “not really being sick.”
The research on somatization disorders is neither easily available nor conclusive.
Moreover, paradoxically, although somatoform disorders in children have been defined as
psychiatric disorders, psychiatrists seldom see these patients. By default, most children and
adolescents who have symptoms are seen by primary care physicians. (1)(2)(3)(4)
This review focuses on understanding, assessing, and developing strategies for manag-
ing somatization disorders.

Definition
Somatization is “the tendency to experience and communicate somatic distress and
symptoms unaccounted by pathological findings.” (5) However, it must be stressed that
somatization can coincide with another illness. Somatization is deemed to exist in
conjunction with a physical illness whenever the physical complaints resulting in impair-
ment are grossly in excess of what would be expected from the known illness or findings.
Thus, the central feature of somatoform disorders is that they present with symptoms
suggestive of an underlying medical condition, yet such a condition either is not found or
does not fully account for the level of impairment the child is exhibiting. (6)(7)(8)

Classification
The diagnostic criteria for somatoform disorders originally were established for adults. (6)(7)
A diagnostic classification of child and adolescent mental conditions in primary care has been
developed that takes into account developmentally appropriate considerations. (8) The manual
containing the classification develops an approach to somatization disorders in children that
pediatricians may consider more user friendly (Table 1).

*Professor, Department of Pediatrics, The George Washington University School of Medicine and Health Sciences; Director,
Adolescent Medicine Fellowship Program, Children’s National Medical Center, Washington, DC.

56 Pediatrics in Review Vol.32 No.2 February 2011


Downloaded from http://pedsinreview.aappublications.org. Provided by Pakistan:AAP Sponsored on February 1, 2011
psychosocial somatization disorders

ling?”). As this “psychosomatic pathway” is reinforced, it


Current Classification of
Table 1. manifests throughout a spectrum of somatization disor-
ders, ranging from the mild “somatic complaint varia-
Somatization Disorders in tion” (transient complaints that do not interfere with
Children and Adolescents normal functioning) to the more severe “somatoform
disorder” (associated with significant social and academic
• Somatic complaint variation (v 65.49)
• Somatic complaint problem (v 40.3) problems).
• Somatization disorder (300.82) The importance of psychosocial factors in the child’s
• Somatoform disorder (undifferentiated) (300.82) family is highlighted by the finding that if a family
• Somatoform disorder, not otherwise specified
member has a chronic physical illness, more somatic
(300.82)
• Pain disorder (307.8) symptoms occur among the children. Somatizing chil-
• Conversion disorders (300.6) dren often live with family members who complain of
Data from Wolraich ML, Felice ME, Drotar D. The classification of strikingly similar physical symptoms. Theoretical contri-
child and adolescent mental diagnosis in primary care. In: Diagnostic butions stemming from systemic family therapy also
and Statistical Manual for Primary Care (DSM-PC) Child and Adoles-
cent Versions. Elk Grove Village, IL: American Academy of Pediatrics; point toward the importance of the family’s circum-
1996. stances. For example, it has been proposed that the
symptoms displayed by the child are a way of protecting
a distressed parent who, galvanized into caring for the
Epidemiology suffering child, is then distracted from his or her own
The prevalence of symptoms associated with somatiza- personal concerns. (1)(2)(3)(4)
tion in the pediatric population is high: recurrent ab- Stress has been implicated as a triggering factor, and
dominal pain accounts for 5% of pediatric office visits, such stress often is bound to parental anxiety. The most
and headaches have been reported to affect 20% to 55% common form of stress consists of pressure on the child to
of all children, with 10% of teenagers reporting frequent perform. More serious problems, such as physical or sexual
headaches, chest pain, nausea, and fatigue. A sex disparity abuse or being bullied, also may lead to somatic complaints
in the prevalence of somatic symptoms during adoles- that often develop into a somatization disorder. (16)
cence (11% of girls and 4% of boys) persists into adult- (17)(18)(19)(20)(21)(22)(23)(24)(25)(26)(27)(28)(29)
hood. The rate of somatization is highest among lower
socioeconomic groups. (4)(9)(10)(11)(12)
Clinical Aspects
Pathogenesis Children and adolescents readily report pain and somatic
There is a genetic predisposition to somatization, prob- complaints in their sick visits. These complaints often
ably related to traits such as alexithymia (difficulty in
result from a disease such as gastroenteritis or urinary
reading one’s own emotions), trait anxiety (a continual
tract infection. However, children can voice similar com-
tendency to react with anxiety), and anxiety sensitivity
plaints in the absence of physical disease, and these
(a fear of anxiety symptoms and misinterpretation of
reports must be approached as possible somatization.
their meaning). Genetic studies have shown somatoform
(30)(31)(32)(33)(34)(35)(36)(37)(38)(39) Somato-
disorders to be concordant in identical twins. These
conditions also cluster above what would be expected by form disorders follow a developmental sequence as chil-
chance in families in which there is attention-deficit dren experience affective distress in the form of somatic
disorder and alcoholism. (13)(14)(15) sensations. In early childhood, these symptoms are recur-
Somatization, nevertheless, is a learned behavior. It rent abdominal pain and, somewhat later, headaches. As
begins with experiences of having a somatic complaint age increases, neurologic symptoms, insomnia, and fa-
more easily noticed or more acceptable in a household tigue tend to emerge.
than the expression of strong feelings, such as anxiety, Somatization disorders form a continuum that ranges
fear, jealousy, and anger. In such an environment, a child from everyday aches and pains to disabling symptoms.
may repeatedly garner no attention for emotional dis- These symptoms are spontaneous and not feigned
tress, instead obtaining more attention for the physical (which distinguishes them from malingering and facti-
symptoms that often accompany the disturbed emotional tious disorder) and are not explained better by another
state, such as abdominal pain (“Are you hurting, dar- mental illness (such as depression or an anxiety disorder).

Pediatrics in Review Vol.32 No.2 February 2011 57


Downloaded from http://pedsinreview.aappublications.org. Provided by Pakistan:AAP Sponsored on February 1, 2011
psychosocial somatization disorders

Somatic Complaint Variation with both a psychological and a general medical condi-
This variation involves discomfort and complaints that tion, and pain associated with a general medical condi-
do not interfere with everyday functioning. It is a univer- tion. The onset of pain may be related to psychological
sal experience. In early infancy, the complaints probably stressors. Often, a secondary gain is achieved by being
manifest as transient gastrointestinal distress. In child- excused from stressful situations or academic pressures.
hood, classic recurrent abdominal pain, headaches, and These symptoms may be associated with frequent visits
“growing pains” make their appearance. Adolescents to the pediatrician and parental pressure for unnecessary
may experience more headaches, chest pain, and other testing and interventions.
transient aches and pains, but these characteristically do
not impair their ability to function. Females report more Conversion Disorder
somatic complaints after puberty. In conversion disorders, one or more symptoms or defi-
cits affect a sensory or voluntary motor function (eg,
Somatic Complaint Problem blindness, paresis), suggesting a medical or neurologic
This condition consists of one or more physical com- condition, yet the findings are not consistent with any
plaints that cause sufficient distress and impairment known neuroanatomic/pathophysiologic explanation.
(physical, social, or school) to be considered a problem. (40)(41)(42) The symptoms tend to have a “symbolic
In infancy, this situation might occur when gastrointes- meaning,” dealing with an unsolved and unconscious
tinal symptoms seriously interfere with feeding and sleep. conflict (often relating to themes of repressed aggression
In childhood, such a condition might entail avoiding or or sexuality). The symptoms appear to be a symbolic
refusing to undertake expected activities (eg, increased attempt to resolve the conflict (primary gain), although
school absences). As adolescence approaches, in addition they also often result in increased attention for the pa-
to the somatic complaints, more emotional distress, so- tient (secondary gain).
cial withdrawal, and academic difficulties begin to ap- This form of somatization disorder frequently, but
pear. More severe complaints may result in refusal to not always, is accompanied by “la belle indifference,” an
attend school and recurrent pain syndromes. attitude of disinterest by the patient despite the serious
symptoms experienced. Although the symptoms usually
Undifferentiated Somatoform Disorder are self-limited, often resolving within 3 months, they
This condition emerges during adolescence, causing sig- may persist and ultimately be associated with chronic
nificant impairment. Multiple severe symptoms of at least sequelae, such as contractures. Frequently, there is a
6 months’ duration are required to make the diagnosis. model for the symptoms, such as a grandparent who has
Complaints include, but are not limited to, pain syn- cardiac symptoms, but sometimes the patient may be
dromes, gastrointestinal or urogenital complaints, fa- serving as his or her own model, as when pseudoseizures
tigue, loss of appetite, and pseudoneurologic symptoms. occur in patients who have epilepsy.
To qualify for this diagnosis, the symptoms should not be
explained better by another mental disorder, such as a Additional Somatoform Disorders
mood or anxiety disorder, and should not be feigned or Hypochondriasis (preoccupation with the idea of having
intentionally produced. A more severe form, the classic a serious disease) and body dysmorphic disorder (over-
somatization disorder, is an adult condition. preoccupation with an imagined or exaggerated defect in
physical appearance) are uncommon and seen primarily
Somatoform Disorder, Not Otherwise Specified during late adolescence and young adulthood.
This classification encompasses adolescents who have
unexplained physical complaints (fewer than 6 months’ Differential Diagnosis (Table 2)
duration) as well as those who manifest somatoform Organic illnesses must be considered in the differential
symptoms that do not meet the criteria for any specific diagnosis. Over time, up to one third of patients in whom
somatoform disorder, such as pseudocyesis, in which the conversion disorder is diagnosed eventually receive a
false belief of being pregnant can be accompanied by neurologic diagnosis. Endometriosis is often a very late
endocrine changes. diagnosis following years of “functional pelvic pain,” and
persistent epigastralgia may represent a Helicobacter py-
Pain Disorder lori infection. Psychiatric disorders, such as depression
There are three types of pain disorder: pain associated and anxiety disorder, often present initially with physical
exclusively with psychological factors, pain associated complaints such as poor concentration; fatigue; weight

58 Pediatrics in Review Vol.32 No.2 February 2011


Downloaded from http://pedsinreview.aappublications.org. Provided by Pakistan:AAP Sponsored on February 1, 2011
psychosocial somatization disorders

well-being?” and “Is there a family history of psychiatric


Differential Diagnosis of
Table 2. disorder?” A detailed school history that reviews the
numbers of days missed is essential.
Pediatric Somatization In the process of evaluating somatic complaints, the
• Unrecognized physical disease (eg, multiple sclerosis, clinician should avoid the temptation to perform unnec-
endometriosis) essary, repetitive, or extensive testing in an attempt to
• Unrecognized psychiatric disorder (eg, depression, demonstrate to the family that the presenting complaint
anxiety) is of psychosomatic origin. A cost-effective method of
• Factitious disorder/Munchausen by proxy syndrome
determining the appropriate extent of laboratory and
• Psychological factors affecting a medical condition
radiographic evaluation is to base the diagnostic plan on
the presence of “red flags.” Such complaints and findings
suggest an organic cause, such as syncope on exercise,
loss; and an increase in headaches, stomachaches, and asymmetric location of pain, anemia, or weight loss.
chest pains. It is important to look for specific psychiatric When the history and physical examination findings are
disorders. Epidemiologic studies show that 14% to 20% suggestive of somatization, a basic laboratory screening
of American children have one or more moderate-to- consisting of a complete blood count, an erythrocyte
severe psychiatric disorders, with the overall prevalence sedimentation rate or a C-reactive protein, a urine dip-
rising, and the treatment is somewhat different from that stick evaluation, and sometimes a blood chemistry and
of somatization. Factitious disorders, which sometimes occult blood stool test is sufficient. More extensive as-
are included in the somatization spectrum, can be ex- sessments should be reserved for the “red flags.”
cluded from this category because the signs and symp-
toms presented have been staged deliberately and have Management and Treatment
not really been experienced by the patient as somatiza- Eventually, the clinician must “bite the bullet.” The best
tion symptoms. method of persuasion is to precede any disclosure with a
clear demonstration that one has taken the complaint
Evaluation very seriously, which is accomplished best when pre-
Initially, it is usually unclear whether a particular com- ceded by careful history taking and a detailed physical
plaint eventually will declare itself to be caused by an examination. The aim is to convey a sense of specialness
underlying disease. Establishing the diagnosis of a so- to the child and family. This communication may serve as
matoform illness is accomplished over time along three a buffer to the emotional injury stemming from having to
simultaneous tracks: 1) ruling out an organic disease as recognize that “something is wrong” in the child’s life.
the cause of the symptoms, 2) identifying psychosocial Correct identification of somatization disorders may not
dysfunction, and 3) recognizing and alleviating stressors. be sufficient to provide help to patients and families, who
A concomitant biopsychosocial assessment is therapeutic often are reluctant to accept the explanation. (43) There-
and often followed by improvement and sometimes even fore, successful communication about the condition and
resolution of symptoms. the needed treatment is a crucial but sometimes elusive
However, the differential diagnosis and evaluation are goal (Table 3).
not based solely on a process of exclusion. It involves In preparation for disclosure of concerns about a
identification of a set of positive psychosocial findings. possible somatization disorder, it is very important to ask
During the initial evaluation, it is important to discuss the child and family about their fear or “fantasy of
explicitly the stressors detected and a possible psychoso- disease.” This query may elicit surprising answers, such as
matic cause as one among many considerations. This will fear that the child may have cancer or heart disease.
make for an easier future “disclosure.” Conversely, the reply may convey an already harbored
Findings that are highly suggestive of a somatization suspicion or understanding of the problem such as “It
disorder include a history of multiple somatic com- may be stress or nerves.” Patients are more willing to
plaints, multiple physician visits, and many specialty con- listen to the pediatrician only if he or she first listens to
sultations as well as the presence of a family member who them. A clear, supportive, matter-of-fact explanation also
has chronic and recurrent symptoms and dysfunction in should assure families that the pediatrician is available to
the primary areas of life (family, peers, and school). help with the onslaught of feelings that many families
Additional questions might include, “Does the parent experience at the time of diagnosis. During disclosure,
have any concern about the child’s behavior or emotional the clinician should remind the family of the earlier

Pediatrics in Review Vol.32 No.2 February 2011 59


Downloaded from http://pedsinreview.aappublications.org. Provided by Pakistan:AAP Sponsored on February 1, 2011
psychosocial somatization disorders

both processed by the central nervous system and influ-


Principles of Pediatric
Table 3. enced by personal experience, genetics, and the environ-
ment. To help patients and parents become more open to
Somatization Treatment the concept of somatization, they can be reminded of
• Form an alliance with the patient and family how themes in everyday language acknowledge the con-
• Be direct; avoid deception in explanations and nection between emotions and bodily processes. For
treatments example, we talk about having a “gut reaction,” having
• Offer reassurance when appropriate
“butterflies in my stomach,” feeling “all choked up,” or
• Use cognitive and behavioral interventions
• Use a rehabilitative approach that something “makes me vomit.” In addition, embar-
• Use positive and negative reinforcement rassment can manifest as blushing, fear as cold sweat, and
• Teach self-monitoring techniques (eg, hypnosis, anger as stiffening muscles and clenching teeth, thus
relaxation, biofeedback) facilitating explanations such as “blushing of the gut”
• Consider family and group therapies
• Improve communication between clinicians and and “spastic colon.” Another way to help patients and
school parents view somatization is as a sensitivity, a phenome-
• Consolidate care when possible non of “amplification” of otherwise normal body sensa-
• Aggressively treat comorbid psychiatric conditions tions, similar to the role of the amplifier in the transmis-
• Consider psychopharmacologic interventions
sion of music. (44)
• Monitor outcome
At the center of any successful program is the untiring
Adapted from Campo and Fritz. (2)
effort to motivate patients and parents toward a partner-
ship with the doctor in dealing with the symptoms and
complaints. Although it could be argued that the risk of
conversation referring to the possibility that the symp-
antagonizing patients and parents by presenting a diag-
toms may be related to stress, temperamental sensitivity,
nosis of somatization and causing them to leave the
anxiety, or whatever term may be tolerated by the family
practice for “doctor shopping” would recommend the
to facilitate acceptance of a behavioral intervention or
policy of helping patients by medicating them symptom-
even a request for psychological or psychiatric consulta-
atically with analgesics, tranquilizers, anxiolytics, and
tion.
other agents from the pharmacopeia, such action tends
It is important for pediatricians to recognize their own
to be only a short-term solution. Although tempting and
responses to the family resistance and reluctance to lay
certainly easier, such an approach may only reinforce the
aside the “search for disease” and not inadvertently con-
vey by word or body language their own frustration search for the “magic pill” and a never-ending pursuit of
about the difficult and time-consuming task they are a technological solution.
facing. At a deeper level, the reason for informing patients
A primary reason that many patients and their parents and families about the nature of the disorder involves the
are angry and reject the diagnosis of somatization disor- principle of respect for persons because it is an ethical
der is that they feel disrespected and not believed: “You duty, with few exceptions, for clinicians to share with
think it is all in my head, but I know I hurt and that there patients their understanding of their situation.
is something wrong.” In part, this conception relates to The primary exception to this rule, which allows for
the unfortunate term “psychosomatic,” which conveys justified paternalism and “face-saving” suggestive thera-
the mistaken notion of a body-mind duality and for some pies, may apply to those patients who have conversion
individuals still carries the connotation of being crazy disorders and cannot make use of the information. The
(“psycho”). Therefore, it is important to explain that the diagnosis of somatization should not lead a patient or
“functional versus organic” paradigm is old-fashioned parent to the perception that this diagnosis will be raised
and does not reflect current thinking, which suggests a as a barrier to preempt future complaints. Instead, it
more complex interplay of multiple factors, including should become clear that the diagnosis is made in the
genetic and biochemical components predisposing to the spirit of offering an interpretation that may call for newer
patient’s symptoms. and more effective treatments, such as stress manage-
Essentially, the pediatrician must convey an under- ment and individual or family counseling. Somatoform
standing that the patient’s pain is real; that is, the doctor disorders do respond to treatment and rehabilitation.
has learned and believes that pain is due to a neural Cognitive and behavioral interventions; use of positive
nociceptive component and an affective component, and negative reinforcements; and self-monitoring tech-

60 Pediatrics in Review Vol.32 No.2 February 2011


Downloaded from http://pedsinreview.aappublications.org. Provided by Pakistan:AAP Sponsored on February 1, 2011
psychosocial somatization disorders

niques such as hypnosis, relaxation, and biofeedback Often, families worry that the diagnosis of somatiza-
have been proven successful. (1)(2)(3)(4)(45)(46) tion will be followed by abandonment by the physician.
Many patients miss school because of their symptom- This concern can be dispelled by arranging frequent
atology. This practice can develop into a full-blown follow-up visits, which have the potential to “preempt”
school avoidance syndrome. This evolution often is not the frequent emergence of new symptoms, prevent emer-
recognized by the parents because “my child loves school gency department visits, and ease the overall manage-
and wants to go” (which may actually be true, but the ment of symptoms. It is helpful to emphasize that all
child is unable to do so). Family counseling and good forthcoming symptoms will be examined with the atten-
communication between the clinician and the school can tion they deserve because somatizing under stress is very
aid in addressing this situation. The instruction is that the common and does not “provide immunity” against ap-
child must be brought to school and stay in the building. pendicitis, flu, pneumonia, and other conditions. Most
Class attendance is not mandatory but going to school is. families, even when disagreeing with their physicians, can
If need be, the child may rest in the school infirmary, a accept (albeit grudgingly) treatment recommendations if
quiet area, or the library. There is no returning home they are assured of an attentive, open-minded, and reg-
unless the child is febrile. Class attendance usually fol- ularly scheduled follow-up.
lows spontaneously. Parents must be prepared for the The payment structure of medical services can con-
difficult days, which often occur on Sunday evening and spire against optimal care for patients who have somati-
the last vacation day before returning to school. Reach- zation disorders. In part, this situation has evolved be-
ing this type of agreement can be facilitated by one of the
cause procedural interventions historically have been
parents being ready and willing to help the other.
valued by insurance companies above spending time with
A patient not responding to intensive treatment
patients and in part because many insurers “carve out”
should be evaluated for the possibility of comorbidity
these types of disorders exclusively for treatment through
such as a mood disorder, anxiety disorder, and substance
the mental health insurance coverage. Frequently, such
abuse, which should be identified and treated to assure
“carve outs” mean that patients must pay their pediatri-
successful treatment. The presence of comorbid condi-
cians out of pocket or from their mental health benefits,
tions is why pediatricians treating children who have
which rarely is allowed. At other times, services simply go
recurrent somatic complaints must become familiar with
unpaid.
screening for anxiety disorder, depression, attention-
Depending on contractual arrangements, pediatri-
deficit/hyperactivity disorder, substance use disorder,
and conduct disorder. cians currently have three less-than-satisfying options:
Judicious use of psychopharmacologic treatment in 1) accept the rate of reimbursement for their services and
somatoform disorders may be appropriate when comor- bill the rest to the family, 2) refer the family to a consul-
bid depression or anxiety is suspected or the severity of tant and coordinate care, or 3) negotiate directly with the
symptoms has led to significant and prolonged impair- payer about the case. The reason to expend all this effort
ment. Once the pediatrician can convince the patient and is that gaining expertise in this type of work can make a
family to seek additional treatments, such as psycholog- great difference in the life of children, and the effort is
ical therapy or evaluation for the use of medication, it is rewarded by success and enormous professional satisfac-
important for the consulting mental health professional tion.
to receive a detailed description of what led to the
consultation because that history may be missing from
the patient’s narrative. Prognosis
Conversely, the psychiatrist or psychologist must be With appropriate intervention, the prognosis for most
asked to provide feedback to the pediatrician because somatization disorders in children and adolescents is very
some families may report that the mental health clinician good. On occasion, somatization is the proverbial “tip of
said there was “nothing wrong, it was all medical.” The the iceberg” that calls attention to a psychiatric disorder
consultant should be expected to tell the referring pedi- necessitating mental health consultation and treatment.
atrician what services will be provided and what the Unfortunately, many untreated children risk continuous
pediatrician is expected to monitor. For pediatricians somatization as adults. (47)(48)(49)(50) The most se-
who are sophisticated in the use of counseling and psy- vere form, the undifferentiated somatoform disorder, is
chotropic medications, a mental health referral might not related to personality disorders, is of long duration, and
be necessary. has a persistent course.

Pediatrics in Review Vol.32 No.2 February 2011 61


Downloaded from http://pedsinreview.aappublications.org. Provided by Pakistan:AAP Sponsored on February 1, 2011
psychosocial somatization disorders

Manual of Mental Disorders. 4th ed, Text Revision (DSM-IV-TR).


Summary Washington, DC: American Psychiatric Association; 2004
7. American Psychiatric Association. Diagnostic and Statistical
Manual of Mental Disorders Primary Care Version. 4th ed. Wash-
• The Diagnostic and Statistical Manual for Primary
Care (DSM-PC) Child and Adolescent Version (8) ington, DC: American Psychiatric Press; 2000
contains a useful pediatric classification of 8. Wolraich ML, Felice MD, Drotar D. The classification of child
somatization disorders that is based on consensus and adolescent mental diagnoses in primary care. In: Diagnostic and
due to lack of relevant clinical studies. Statistical Manual for Primary Care (DSM-PC) Child and Adoles-
• Based on some research evidence, somatization cent Version. Elk Grove Village, IL: American Academy of Pediat-
disorders occur in children who are genetically rics; 1996
predisposed and exhibit trait anxiety, anxiety 9. Haugland S, Wold B, Stevenson J, Aaroe LE, Woynarowska B.
sensitivity, and alexithymia. (13)(14) There is Subjective health complaints in adolescence. A cross-national com-
concordance in identical twins. parison of prevalence and dimensionality. Eur J Public Health.
• Environmental factors are suggested by the presence 2001;11:4 –10
of models in the family and the preponderance of 10. Alfven G. The covariation of common psychosomatic symp-
these conditions among children of families living in toms among children from socio-economically differing residential
lower socioeconomic strata. areas. An epidemiological study. Acta Paediatr. 1993;82:484 – 487
• Stress is an important contributory factor; the 11. Campo JV, Fritz GL. Somatization in children and adoles-
possibility of bullying or physical or sexual abuse cents. J Am Acad Child Adolesc Psychiatry. 1994;33:1223–1235
must be considered. 12. Fritz GK, Fritsch S, Hagino O. Somatoform disorders in
• Adolescents who have somatoform pain disorders are children and adolescents: a review of the past 10 years. J Am Acad
often afflicted by alexithymia. Child Adolesc Psychiatry. 1997;36:1329 –1337
• Agreement on the management of somatization 13. Muris P, Meesters C. Children’s somatization symptoms: cor-
disorders outlined in this article is based on relations with trait anxiety, anxiety sensitivity, and learning experi-
consensus. Patients who do not respond to pediatric ences. Psychol Rep. 2004;94:1269 –1275
intervention may suffer from comorbidity such as an 14. Burba B, Oswald R, Grigaliunien V, et al. A controlled study of
anxiety or mood disorder. alexithymia in adolescent patients with persistent somatoform pain
disorder. Can J Psychiatry. 2006;51:468 – 471
15. Torgersen S. Genetics of somatoform disorders. Arch Gen
Psychiatry. 1986;43:502–505
16. Berntsson LT, Gustafsson JE. Determinants of psychosomatic
Advocacy complaints in Swedish schoolchildren aged seven to twelve years.
From a professional development perspective, advocacy Scand J Public Health. 2000;28:283–293
work must continue to emphasize that change in medical 17. Friedrich WN, Schafer LC. Somatic symptoms in sexually
economics is needed, specifically, that the financial im- abused children. J Pediatr Psychol. 1995;20:661– 670
pact of somatization on utilization must be addressed. 18. Garber J, Van Styke DA, Walker LS. Concordance between
mother’s and children’s reports of somatic and emotional symp-
(51) Increased funding is needed for both research and
toms in patients with recurrent abdominal pain or emotional disor-
training in an area that has such great impact on quality ders. J Abnormal Child Psychol. 1998;26:381–391
of life. Teaching about this subject needs to be incorpo- 19. Greene JW, Walker LS. Psychosomatic problems and stress in
rated into the medical education of medical students, adolescence. Pediatr Clin North Am. 1997;44:1557–1572
residents, subspecialty fellows, and postgraduate stu- 20. Green M. Vulnerable child syndrome and its variants. Pediatr
Rev. 1986;8:75– 80
dents to increase all clinicians’ level of comfort and sense
21. Dutta S, Mehta M, Verma IC. Recurrent abdominal pain in
of effectiveness in dealing with somatization disorders. Indian children and its relation with school and family environment.
Indian Pediatr. 1999;36:917–920
22. Kinzl JF, Traveger C, Biebl W. Family background and sexual
References abuse associated with somatization. Psychother Psychosom. 1995;64:
1. Silber TJ, Pao M. Somatization disorders in children and ado- 82– 87
lescents. Pediatr Rev. 2003;24:255–266 23. Orr D. Adolescence, stress, and psychosomatization. J Adol
2. Campo JV, Fritz GL. A management model for somatization. Health Care. 1986;7:97S –108S
Psychosomatics. 2001;42:467– 476 24. Srabstein J, Joshi P, Due P, et al. Prevention of public health
3. Rickert VI, Jay MS. Psychosomatic disorders: the approach. risks linked to bullying: a need for a whole community approach.
Pediatr Rev. 1994;15:448 – 454 Int J Adolesc Med Health. 2008;20:185–199
4. Reese A, Strasburger VC. Is it “real” or is it “psychosomatic?” 25. Tanaka H, Tamai H, Terashima S, et al. Psychosocial factors
Basic principles of psychosomatic medicine in children and adoles- affecting psychosomatic symptoms in Japanese schoolchildren.
cents. In: Greydanus DE, Wolraich ML, eds. Behavioral Pediatrics. Pediatr Int. 2000;42:354 –358
New York, NY: Springer Verlag; 1992:352–356 26. Walker LS, Greene JW. Children with recurrent abdominal
5. Lipowski ZJ. Somatization: the concept and its clinical applica- pain and their parents: more somatic complaints, anxiety, and
tion. Am J Psychiatry. 1988;145:1358 –1368 depression than other patient families? J Pediatr Psychol. 1989;14:
6. American Psychiatric Association. Diagnostic and Statistical 231–243

62 Pediatrics in Review Vol.32 No.2 February 2011


Downloaded from http://pedsinreview.aappublications.org. Provided by Pakistan:AAP Sponsored on February 1, 2011
psychosocial somatization disorders

27. Walker LS, Garber J, Grene JW. Somatization symptoms in 41. Dvonch VM, Bunch WH, Siegler AH. Conversion reactions in
pediatric abdominal pain patients: relationship to chronicity of pediatric athletes. J Pediatr Orthop. 1991;11:770 –772
abdominal pain and parent somatization. J Abnorm Child Psychol- 42. Gooch JL, Wolcott R, Speed J. Behavioral management of
ogy. 1991;19:379 –394 conversion disorder in children. Arch Phys Med Rehabil. 1997;78:
28. Atlas JA, Wolfson MA, Lipschitz DS. Dissociation and soma- 264 –268
tization in adolescent inpatients with and without history of abuse. 43. Steinhauer PD. Resistances to the biopsychosocial approach:
Psychol Rep. 1995;76:1101–1102 individual, familial and systemic. Behavior Pediatr. 1990;11:330 –332
29. Brown RT. Adolescents with psychosomatic problems. Compr 44. Barsy AJ, Goodson JD, Lane RS, Cleary PD. The amplification
Ther. 1996;22:810 – 816 of somatic symptoms. Psychosom Med. 1988;50:510 –519
30. Forsyth R, Farrell K. Headache in childhood. Pediatr Rev. 45. Sugarman LI. Hypnosis: teaching children self-regulation.
1999;20:39 – 45 Pediatr Rev. 1996;17:5–11
31. Garralda ME. Psychosomatic illness in children. Practitioner. 46. Olness K. Hypnosis and biofeedback with children and adoles-
1992;236:621– 622 cents: clinical, research, and educational aspects. Introduction.
32. Hyams JS, Hyman PE. Recurrent abdominal pain and the bio- J Dev Behav Pediatr. 1996;17:299
psychosocial model of medical practice. J Pediatr. 1998;133:473– 478 47. Cohen P, Pine DS, Must A, Kasen S, Brook J. Prospective
33. Murray KF, Christie DL.Vomiting. Pediatr Rev. 1998;19:237 associations between somatic illness and mental illness from child-
34. Narchi H. The child who passes out. Pediatr Rev. 2000;21:384 hood to adulthood. Am J Epidemiol. 1998;147:232–239
35. Rudra HS, Park JH, Webbe-Hijazi N, et al. Index of suspicion. 48. Bass C, Murphy M. Somatoform and personality disorders:
Pediatr Rev. 2006;27:107–112 syndromal co-morbidity and overlapping developmental pathways.
36. Selbst SM, Ruddy RM, Clark BJ, et al. Pediatric chest pain: a J Psychosom Res. 1995;39:403– 427
prospective study. Pediatrics. 1988;82:319 –323 49. Berntsson LT, Kohler L. Long-term illness and psychosomatic
37. Sherry DD, McGuire T, Mellins E, et al. Psychosomatic mus- complaints in children aged 2–17 years in the five Nordic countries.
culoskeletal pain in childhood: clinical and psychological analyses of Comparison between 1984 and 1996. Eur J Public Health. 2001;
100 children. Pediatrics. 1991;88:1093–1099 11:35– 42
38. Willis J. Syncope. Pediatr Rev. 2000;21:201 50. Offord DR, Boyle MH, Szatmari P, et al. Ontario Child Health
39. Zuckerman B, Stevenson J, Bailey V. Stomach and headaches Study. II: Six month prevalence of disorder and rates of service
in a community sample of preschool children. Pediatrics. 1987;79: utilization. Arch Gen Psychiatr. 1987;44:832– 836
677– 682 51. Bernal P, Estroff DB, Aboudarham JF, et al. Psychosocial
40. Hogdman CH. Conversion and somatization in pediatrics. morbidity: the economic burden in a pediatric health maintenance
Pediatr Rev. 1995;16:29 –34 organization sample. Arch Pediatr Adolesc Med. 2000;154:261–266

PIR Quiz
Quiz also available online at http://pedsinreview.aappublications.org.

5. Which of the following statements regarding somatization disorders in children is true?


A. Neurologic symptoms are the most common manifestation in infants.
B. Symptoms are generally feigned with a secondary gain in mind.
C. They are more common in children who are encouraged to openly display their fear or anger.
D. They are more common in children who have a family member with a chronic disease.
E. They occur more often in higher socioeconomic groups.

6. You are evaluating a 16-year-old girl who complains of fatigue for the past 3 months. She reports no
fever, weight loss, or pain. She has difficulty sleeping at night and decreased appetite, and she has quit her
after-school drama class, which she previously enjoyed. Her family history reveals a mother who has
depression and fibromyalgia. Except for a flat affect, her physical examination results are normal. Which of
the following is the most likely diagnosis in this patient?
A. Conversion disorder.
B. Depression.
C. Somatic complaint problem.
D. Somatoform disorder, not otherwise specified.
E. Undifferentiated somatoform disorder.

Pediatrics in Review Vol.32 No.2 February 2011 63


Downloaded from http://pedsinreview.aappublications.org. Provided by Pakistan:AAP Sponsored on February 1, 2011
psychosocial somatization disorders

7. A 10-year-old boy comes to your clinic with a 2-month history of intermittent vision loss that lasts for
several minutes and resolves spontaneously. He describes a complete loss of vision in both eyes, and he has
missed approximately 20 days of school. He denies other symptoms. His family history reveals a
grandmother who lives with them and who has Alzheimer disease. Because of financial strain, he has been
asked to care for his grandmother in the afternoons and evenings. An ophthalmologic evaluation 3 weeks
ago showed normal findings. On physical examination, he reports normal vision and appears indifferent to
his reported symptoms. Findings on examination are normal, including the neurologic examination. Which
of the following is most likely?
A. Anxiety disorder.
B. Conversion disorder.
C. Optic neuritis.
D. Somatic complaint variation.
E. Undifferentiated somatoform disorder.

8. You have diagnosed undifferentiated somatoform disorder in an adolescent girl who has recurrent
abdominal pain. Even though she has no weight loss, diarrhea, or other symptoms and her examination
findings are normal, her parents are concerned that she may have inflammatory bowel disease because her
grandmother has Crohn disease. Which of the following should you do at this time?
A. Arrange a series of follow-up visits to monitor how she is doing and develop a program to help her and
explain the connection between emotions and bodily functions to the patient and her family.
B. Conduct a thorough search for pathologic causes, including endoscopy and upper gastrointestinal
radiographic series.
C. Assess whether there is occult blood in stools, obtain a C-reactive protein measurement, and reassure
the family that nothing is wrong and that the pain will resolve.
D. Encourage the girl to ignore her pain because it is not real pain and she needs to learn how to live with
it.
E. Refer the girl to a child psychiatrist for a psychopharmacology consultation to consider anxiolytic
therapy.

64 Pediatrics in Review Vol.32 No.2 February 2011


Downloaded from http://pedsinreview.aappublications.org. Provided by Pakistan:AAP Sponsored on February 1, 2011
Somatization Disorders: Diagnosis, Treatment, and Prognosis
Tomas Jose Silber
Pediatr. Rev. 2011;32;56-64
DOI: 10.1542/pir.32-2-56

Updated Information including high-resolution figures, can be found at:


& Services http://pedsinreview.aappublications.org/cgi/content/full/32/2/56
Subspecialty Collections This article, along with others on similar topics, appears in the
following collection(s):
Psychosocial Issues and Problems
http://pedsinreview.aappublications.org/cgi/collection/psychosoc
ial_issues_problems
Permissions & Licensing Information about reproducing this article in parts (figures,
tables) or in its entirety can be found online at:
http://pedsinreview.aappublications.org/misc/Permissions.shtml
Reprints Information about ordering reprints can be found online:
http://pedsinreview.aappublications.org/misc/reprints.shtml

Downloaded from http://pedsinreview.aappublications.org. Provided by Pakistan:AAP Sponsored on February 1, 2011


Focus on Diagnosis: Urine Electrolytes
J. Bryan Carmody
Pediatr. Rev. 2011;32;65-68
DOI: 10.1542/pir.32-2-65

The online version of this article, along with updated information and services, is
located on the World Wide Web at:
http://pedsinreview.aappublications.org/cgi/content/full/32/2/65

Pediatrics in Review is the official journal of the American Academy of Pediatrics. A monthly
publication, it has been published continuously since 1979. Pediatrics in Review is owned,
published, and trademarked by the American Academy of Pediatrics, 141 Northwest Point
Boulevard, Elk Grove Village, Illinois, 60007. Copyright © 2011 by the American Academy of
Pediatrics. All rights reserved. Print ISSN: 0191-9601. Online ISSN: 1526-3347.

Downloaded from http://pedsinreview.aappublications.org. Provided by Health Internetwork on April 20, 2011


focus on diagnosis

Urine Electrolytes
J. Bryan Carmody, MD* propriate effective circulating vol-
ume. Therefore, the major use of the
Introduction urine sodium concentration (UNa) is
Although electrolytes in the blood to assess a patient’s volume status.
must be maintained within narrow When the effective circulating vol-
limits for homeostasis, urine is the ume is decreased (hypovolemia), the
Author Disclosure body’s waste, and concentrations of kidney avidly retains sodium in an
Dr Carmody has disclosed no urine solutes vary widely, depending effort to maintain circulating vol-
financial relationships relevant to on the need for the specific solute to ume, and the UNa is low, generally
this article. This commentary does be excreted or retained. However, in less than 20 mEq/L. Without this
a variety of clinical situations, analysis stimulus, sodium excretion parallels
not contain a discussion of an
of urine electrolytes can be very in- dietary intake and generally is more
unapproved/investigative use of a
formative or even diagnostic. To than 40 mEq/L. (1)
commercial product/device. maximize the usefulness of such tests Although assessment of a pa-
(Table), the clinician must possess tient’s volume status is useful in a
both a basic knowledge of renal sol-
variety of situations, measurement of
ute handling and an understanding
the UNa is particularly useful in the
of the particular instances in which
evaluation of two specific states: hy-
measurement of urine electrolytes is
ponatremia and acute renal failure.
useful.
Hyponatremia is caused by excessive
Frequently, clinicians are hesitant
to obtain urine electrolytes due to retention of free water out of propor-
the perceived difficulty in interpret- tion to solute. In clinical practice,
ing them in the absence of “normal” this state is due either to the syn-
values. It is critically important to drome of inappropriate antidiuretic
realize that urine electrolytes can hormone secretion (SIADH) or to
only be interpreted in the context of the appropriate secretion of ADH
the patient and the clinical situation. that occurs in response to hypovole-
For example, a urine potassium con- mia. It is important to note that the
Abbreviations centration of 35 mEq/L may be ap- latter situation occurs not just in sit-
propriate for a healthy patient eating uations in which the total body water
ATN: acute tubular necrosis
a potassium-rich diet but would indi- is decreased such as gastroenteritis
FENa: fractional excretion of
cate inappropriate renal potassium or adrenal insufficiency but also in
sodium
wasting in a patient whose serum po- edematous states such as cirrhosis,
FEurea: fractional excretion of urea
tassium concentration is 2.0 mEq/L congestive heart failure, and ne-
NH4ⴙ: ammonium ion
(2.0 mmol/L). phrotic syndrome. In these cases, al-
RTA: renal tubular acidosis
though total body water is increased,
SIADH: syndrome of inappropriate
antidiuretic hormone
Urine Sodium Concentration the effective circulating volume is de-
Sodium is the body’s primary extra- creased, triggering the appropriate
secretion
cellular cation. Because water move- release of ADH. In volume-depleted
TTKG: transtubular potassium
gradient
ment follows solute movement, so- hyponatremic states, the renal tubules
UAG: urine anion gap
dium has a major physiologic role in reclaim as much sodium as possible,
UCl: urine chloride concentration
maintaining extracellular volume, and the UNa is low at less than
UK: urine potassium concentration
and the kidney actively varies reab- 20 mEq/L. In SIADH, however, pa-
UNa: urine sodium concentration
sorption of sodium to maintain ap- tients experience euvolemia, and so-
Uurea: urine urea nitrogen dium is excreted in the urine at the
concentration *The Children’s Hospital of the King’s Daughters, normal rate, making the urine sodium
Norfolk, VA. concentration greater than 40 mEq/L

Pediatrics in Review Vol.32 No.2 February 2011 65


Downloaded from http://pedsinreview.aappublications.org. Provided by Health Internetwork on April 20, 2011
focus on diagnosis

Table. Common Electrolyte Measurements


Urine Electrolyte Clinical Situation Interpretation
UNa Hyponatremia <20 mEq/L: Volume depletion
>40 mEq/L: SIADH
Acute renal failure <20 mEq/L or FENa <1%*: Prerenal azotemia
>40 mEq/L or FENa >2%*: ATN
UK Hypokalemia TTKG <2 to 3: Nonrenal losses
TTKG >2 to 3: Renal potassium wasting
Hyperkalemia TTKG <6 to 7: Defective potassium secretion
(hypoaldosteronism or renal failure)
TTKG >11: Increased potassium intake or transcellular shift
UCl Metabolic alkalosis <20 mEq/L: “Saline-responsive” alkalosis/volume depletion
with vomiting, nasogastric suctioning
>20 mEq/L: “Saline-resistant” alkalosis/renal chloride
wasting with diuretic use, Bartter/Gitelman syndromes
UAG Hyperchloremic (normal anion gap) Positive UAG: Renal tubular acidosis
metabolic acidosis Negative UAG: Gastrointestinal bicarbonate loss (diarrhea)
Uurea Acute renal failure FEurea <35%: Prerenal azotemia
FEurea >50%: ATN
*Because immature tubules have a reduced reabsorptive capacity for sodium, the FENa is higher in younger or preterm infants.
ATN⫽acute tubular necrosis, FENa⫽fractional excretion of sodium, FEurea⫽fractional excretion of urea, SIADH⫽syndrome of inappropriate antidiuretic
hormone secretion, TTKG⫽transtubular potassium gradient, UAG⫽urine anion gap, UCl⫽urine chloride concentration, UK⫽urine potassium concentra-
tion, UNa⫽urine sodium concentration, Uurea⫽urine urea nitrogen concentration

when the patient is consuming a nor- FENa⫽ place a urine collection bag, then
mal diet. (1) (UNa⫻plasma creatinine/plasma draw blood for chemistries immedi-
For similar reasons, the UNa is also ately after the child voids.
Na⫹⫻urine creatinine)⫻100%
valuable in the assessment of a pa-
tient who has acute renal failure. By expressing the percentage of the Urine Potassium
Here, the differential diagnosis typi- filtered sodium that is excreted, the Concentration
cally is between prerenal azotemia FENa provides a measure of the so- The kidney has an extraordinary ability
and intrinsic renal failure caused by dium handling that is independent of to vary potassium excretion, depend-
acute tubular necrosis (ATN). If the how dilute or concentrated the urine ing on dietary intake and the serum
failure is prerenal, the kidney retains may be. Appropriate sodium conser- potassium concentration. The major
sodium aggressively in response to vation (as might be seen in prerenal value of the urine potassium concen-
hypovolemia and the UNa is low. azotemia) results in an FENa of less tration (UK) is in cases of hypo- or
However, in patients who have ATN, than 1%; patients who have ATN have hyperkalemia to determine whether
the UNa is greater than 40 mEq/L FENa measurements greater than 2%. the kidney is responding appropriately
due to tubular damage that impairs The FENa must be interpreted to these stimuli. Although spot mea-
maximal reabsorption of sodium. (1) slightly differently in infants, who surements of UK may be helpful, uri-
In most situations, only a single have diminished ability to reabsorb nary potassium handling is assessed
random or “spot” measurement of so- sodium due to their immature tubules. best by calculating the transtubular po-
dium is adequate to provide diagnostic Thus, in term neonates, the FENa is less tassium gradient (TTKG).
information. However, because the than 3% in a volume-depleted state, (2)
concentration of sodium in the urine is and in critically ill preterm infants, TTKG⫽
dependent on the amount of free wa- FENa less than 4% indicates maximal (UK)/(plasma K⫹)⫻(plasma
ter in the urine, measuring a random sodium retention. (3) osmolarity)/(urine osmolarity)
UNa alone can be misleading in cases of Calculating the FENa requires si-
impaired renal concentrating ability or multaneous measurement of blood For patients eating normal diets, the
excess free water in the urine. Thus, it and urine sodium and creatinine. In TTKG typically is 8 to 9. However,
may be preferable to calculate the frac- younger children who are unable to when hypokalemia occurs due to gas-
tional excretion of sodium (FENa), void on command, the most practical trointestinal losses or inadequate po-
which is calculated as means of obtaining samples is to tassium intake, the kidney should

66 Pediatrics in Review Vol.32 No.2 February 2011


Downloaded from http://pedsinreview.aappublications.org. Provided by Health Internetwork on April 20, 2011
focus on diagnosis

conserve potassium and the TTKG administration of chloride will have


falls to less than 3. Higher values in minimal impact on the alkalosis be-
the face of hypokalemia indicate re- cause the administered chloride will
nal potassium wasting. (4) be lost in the urine. (1) For more
Conversely, in patients who de- complicated reasons, profound hy-
velop hyperkalemia (especially chronic pokalemia also leads to impaired
hyperkalemia), a TTKG less than 7 in- chloride transport and can cause
dicates defective potassium secretion metabolic alkalosis. (6)
because the appropriate renal response
would be to maximize potassium ex- Urine Anion Gap
cretion. Because potassium excretion The value of the urine anion gap
depends on the presence of aldoste- (UAG) stems from the understand-
rone and adequate fluid and solute de- ing that the total number of cations
livery to the site of aldosterone activity in the urine must necessarily equal
in the distal tubule, this finding sug- the number of excreted anions. Cal-
gests hypoaldosteronism or renal fail- culating the UAG can provide infor-
ure. (5) mation about urine composition
without directly measuring every
Figure 1. A graphic illustration of the
possible solute. The UAG is calcu- principle underlying the urine anion
Urine Chloride Concentration lated by subtracting the commonly gap. UCⴝunmeasured cations, UAⴝ
The major use of the urine chloride measured anions from the commonly unmeasured anions. Because the total
concentration (UCl) is in the evalua- measured cations in urine: cationic charge must necessarily equal
tion of a metabolic alkalosis. Typi- the total anionic charge, a change in the
cally, sodium and chloride are re- UAG⫽UNa⫹UK⫺UCl size of one “compartment” must be ac-
absorbed together throughout the companied by a corresponding change in
nephron. Accordingly, the UNa and Unlike the serum anion gap, which is another compartment. In the situation
UCl are usually approximately equal. always positive, the UAG can be pos- depicted, the anion gap is negative be-
cause the number of unmeasured cations
However, in some cases of metabolic itive or negative, depending on the
exceeds the number of unmeasured an-
alkalosis, the kidney’s need to ex- presence of excess unmeasured cat-
ions (the area indicated by the arrow).
crete bicarbonate may require so- ions or anions in the urine. Knowing Because the major unmeasured cation is
dium to be excreted with bicarbon- that the gap is positive (indicating ammonium, this indicates an appropri-
ate, making the UCl low (typically the presence of excess anions) or ate renal response to metabolic acidosis.
⬍20 mEq/L). This response occurs negative (indicating the presence of
with vomiting or nasogastric suc- excess cations) is clinically sufficient
tion or following hypercapnia. Clas- and more important than the magni- the excretion of protons, which are
sically, such situations were known tude of the gap. typically excreted as ammonium ion
as “saline-responsive” metabolic al- The UAG is most useful in the (NH4⫹).
kaloses because the administration evaluation of a normal anion gap (hy- In contrast, the various RTAs are
of saline and repletion of chloride perchloremic) metabolic acidosis. In characterized as an impaired ability
would correct the alkalosis. this situation, the clinician must dis- to excrete acid. The UAG, thus, is
Alternately, metabolic alkalosis tinguish between gastrointestinal positive (Fig. 2), indicating an inabil-
can be caused by a primary inability losses of bicarbonate (such as diar- ity to secrete NH4⫹ and impaired
to reabsorb chloride, forcing sodium rhea) and renal tubular acidosis renal response to acidosis. Further
to be reabsorbed with another anion (RTA). history and diagnostic testing are re-
such as bicarbonate. In such cases, In metabolic acidosis, the appro- quired to distinguish among the
which include genetic defects of priate renal response is to excrete acid three types of RTA. (7)
chloride reabsorption such as Bartter to return the serum pH to normal.
or Gitelman syndromes as well as di- Thus, with diarrhea or other gastro- Urine Urea Nitrogen
uretic use or abuse, the UCl is ele- intestinal losses of bicarbonate, the As described previously, measure-
vated (⬎20 mEq/L). Here, the alka- UAG is negative (Fig. 1), indicating ment of the UNa is an excellent indi-
losis is “saline-resistant,” and the excess cations in the urine due to cator of a patient’s volume status.

Pediatrics in Review Vol.32 No.2 February 2011 67


Downloaded from http://pedsinreview.aappublications.org. Provided by Health Internetwork on April 20, 2011
focus on diagnosis

make the measurement of the UNa References


unreliable. In such cases, measure- 1. Rose BD. Meaning and application of
ment of the urine urea nitrogen con- urine chemistries. In: Clinical Physiology of
Acid-Base and Electrolyte Disorders, 4th ed.
centration (Uurea) and calculation of New York, NY: McGraw-Hill; 1994:380
the fractional excretion of urea (FEurea) 2. Ellis EN, Arnold WC. Use of urinary
(analogous to the calculation of the indexes in renal failure in the newborn. Am J
FENa) can provide similar information Dis Child. 1982;136:615– 617
3. Tapia-Rombo CA, Velasques-Jones L,
in the evaluation of acute renal failure.
Fernandes-Celis JM, et al. Usefulness of
In low-flow states, the kidney re- fractional excretion of sodium in critically ill
tains urea due to the need to reab- pre-term newborns. Arch Med Res. 1997;
sorb sodium maximally (leading to 28:253–257
the out-of-proportion rise of blood 4. Joo KW, Chang SH, Lee JG, et al. Trans-
tubular potassium concentration gradient
urea nitrogen to serum creatinine (TTKG) and urine ammonium in differen-
that occurs in prerenal azotemia). tial diagnosis of hypokalemia. J Nephrol.
Thus, if the kidney senses decreased 2000;13:120 –125
effective circulating volume, the FEurea 5. Choi MJ, Ziyadeh FN. The utility of the
transtubular potassium gradient in the eval-
is less than 35%; in cases of ATN, the
uation of hyperkalemia. J Am Soc Nephrol.
Figure 2. A graphic illustration of a FEurea exceeds 50%. (8) 2008;19:424 – 426
positive urine anion gap, with the num- 6. Garella S, Chazan JA, Cohen JJ. Saline-
ber of unmeasured cations exceeding the resistant metabolic alkalosis or “chloride-
number of unmeasured anions. When wasting nephropathy.” Ann Intern Med.
this situation occurs in the context of
Conclusion
1970;73:31–38
metabolic acidosis, it is consistent with
Urine electrolytes can be analyzed 7. Kamel KS, Ethier JH, Richardson RM,
renal tubular acidosis, indicating an im- cheaply and quickly on standard lab- et al. Urine electrolytes and osmolality:
paired ability to excrete protons in the oratory equipment, and because when and how to use them. Am J Nephrol.
urine as ammonium. there is no requirement that the 1990;10:89 –102
urine be completely sterile, samples 8. Carvounis CP, Nisar S, Guro-Razuman
S. Significance of the fractional excretion of
can be obtained painlessly, even from urea in the differential diagnosis of acute
However, in cases of acute diuretic infants, by using a urine collection renal failure. Kidney Int. 2002;62:2223–
use, drug-induced natriuresis may bag. 2229

68 Pediatrics in Review Vol.32 No.2 February 2011


Downloaded from http://pedsinreview.aappublications.org. Provided by Health Internetwork on April 20, 2011
Focus on Diagnosis: Urine Electrolytes
J. Bryan Carmody
Pediatr. Rev. 2011;32;65-68
DOI: 10.1542/pir.32-2-65

Updated Information including high-resolution figures, can be found at:


& Services http://pedsinreview.aappublications.org/cgi/content/full/32/2/65
Supplementary Material Supplementary material can be found at:
http://pedsinreview.aappublications.org/cgi/content/full/32/2/65/
DC1
Subspecialty Collections This article, along with others on similar topics, appears in the
following collection(s):
Renal Disorders
http://pedsinreview.aappublications.org/cgi/collection/renal_diso
rders Fluid and Electrolyte Metabolism
http://pedsinreview.aappublications.org/cgi/collection/fluid_elec
trolyte_metabolism
Errata An erratum has been published regarding this article. Please see
next page or:
http://pedsinreview.aappublications.org/cgi/content/full/32/4/151

Permissions & Licensing Information about reproducing this article in parts (figures,
tables) or in its entirety can be found online at:
http://pedsinreview.aappublications.org/misc/Permissions.shtml
Reprints Information about ordering reprints can be found online:
http://pedsinreview.aappublications.org/misc/reprints.shtml

Downloaded from http://pedsinreview.aappublications.org. Provided by Health Internetwork on April 20, 2011


hematology thrombocytopenia

5. A 7-year-old girl presents with a 3-day history of bruising and an episode of epistaxis lasting 30 minutes.
On physical examination, the only abnormalities are scleral icterus, widespread bruising, and cutaneous as
well as mucosal petechiae. Laboratory results include a platelet count of 3ⴛ103/␮L (3ⴛ109/L), hemoglobin
of 7.8 g/dL (78 g/L), white blood cell count of 12.9ⴛ103/␮L (12.9ⴛ109/L), absolute neutrophil count of
8.8ⴛ103/␮L (8.8ⴛ109/L), and mean corpuscular volume of 86 fL. Urinalysis is negative for red blood cells.
The most appropriate next study is:
A. Antiplatelet antibodies.
B. Bone marrow aspirate.
C. Direct antiglobulin (Coombs) test.
D. Flow cytometry on peripheral blood.
E. Serum blood urea nitrogen and creatinine assessment.

Corrections
The caption for Figure 2 in the article entitled “Focus on Diagnosis: Urine Electrolytes” in
the February issue of the journal (Pediatr Rev. 2011;32:65– 68) is incorrect. The correct
caption should read, “A graphic illustration of a positive urine anion gap, with the number
of unmeasured anions exceeding the number of unmeasured cations. When this situation
occurs in the context of metabolic acidosis, it is consistent with renal tubular acidosis,
indicating an impaired ability to excrete protons in the urine as ammonium.” We regret the
error.
The caption for Figure 1 in the article entitled “Sacral Dimples” in the March issue of
the journal (Pediatr Rev. 2011;32:109 –114) is incorrect. The correct caption should read,
“Solitary dimple whose location is greater than 2.5 cm above the anus indicated the need
for further evaluation. . . .” We regret the error.

Pediatrics in Review Vol.32 No.4 April 2011 151


Downloaded from http://pedsinreview.aappublications.org. Provided by Health Internetwork on April 20, 2011
Ethics for the Pediatrician: Ethical Issues in Organ Donation
Robert Macauley
Pediatr. Rev. 2011;32;69-72
DOI: 10.1542/pir.32-2-69

The online version of this article, along with updated information and services, is
located on the World Wide Web at:
http://pedsinreview.aappublications.org/cgi/content/full/32/2/69

Pediatrics in Review is the official journal of the American Academy of Pediatrics. A monthly
publication, it has been published continuously since 1979. Pediatrics in Review is owned,
published, and trademarked by the American Academy of Pediatrics, 141 Northwest Point
Boulevard, Elk Grove Village, Illinois, 60007. Copyright © 2011 by the American Academy of
Pediatrics. All rights reserved. Print ISSN: 0191-9601. Online ISSN: 1526-3347.

Downloaded from http://pedsinreview.aappublications.org. Provided by Pakistan:AAP Sponsored on February 1, 2011


ethics for the pediatrician

Ethical Issues in Organ Donation


Robert Macauley, MD*

Objectives After completing this article, readers should be able to:

1. Define the dead donor rule and identify current controversies surrounding
it.
2. Describe the concept of donation after cardiac death (DCD).
3. Identify the requirements for children to serve as living hematopoietic stem
cell and solid organ donors.
4. Discuss ethical issues related to organ donation that are specific to
Author Disclosure
pediatrics.
Dr Macauley has disclosed no Introduction what was once termed “irreversible
financial relationships relevant to The primary ethical issues in organ coma,” which later came to be known
this article. This commentary does donation involve procurement of vi- as “brain dead.” The Uniform Deter-
tal organs (ie, organs necessary for mination of Death Act (1981) estab-
not contain a discussion of an
life) from deceased donors, procure- lished an all-encompassing definition:
unapproved/investigative use of a ment of nonvital organs from living a patient is dead if he or she has sus-
commercial product/device. donors, and subsequent allocation of tained either “irreversible cessation of
these organs. When children are in- circulatory and respiratory functions”
volved (either as donors or recipi- (cardiorespiratory criteria) or “irrevers-
ents), each of these issues has an ad- ible cessation of all functions of the
ditional layer of complexity. entire brain, including the brain stem”
(neurologic criteria).
Cadaveric Donation of Vital The debate about brain death
Organs continues, however, with such deter-
The fundamental principle of vital minations more complex in pediat-
organ donation is the “dead donor rics, especially for younger children.
rule,” which states that donors must (3) In addition, some critics argue
not be killed to obtain their organs that the current definition of “brain
(1) (or, alternatively, that persons death” is too restrictive, instead ad-
must be dead before their organs are vocating a “higher brain” definition
taken). (2) The matter of when a
of death, which would allow patients
patient is considered dead, however,
who have brainstem (but no cortical)
is rather complex. Before the advent
function to be declared dead. (4)
of mechanical ventilation—when
Other critics have questioned whether
cessation of brain function would in-
a patient who meets “whole brain”
evitably cause a patient to stop
brain death criteria is truly dead (based
breathing— determination of death
on, for example, continued neurohor-
depended solely on cardiorespiratory
monal regulation in the presence of a
parameters. Ventilators made it pos-
flat electroencephalographic tracing
sible to sustain cardiorespiratory
(5)). Although this perspective could
function even when a patient was in
mean that vital organs cannot be re-
trieved from such patients, some have
*Medical Director of Clinical Ethics, Fletcher Allen used this consideration as an occasion
Health Care; Associate Professor of Pediatrics,
University of Vermont College of Medicine, to question whether the dead donor
Burlington, VT. rule is even necessary, instead focusing

Pediatrics in Review Vol.32 No.2 February 2011 69


Downloaded from http://pedsinreview.aappublications.org. Provided by Pakistan:AAP Sponsored on February 1, 2011
ethics for the pediatrician

on the primacy of informed consent rologic criteria. Controlled DCD tion” of cardiorespiratory function,
for vital organ donation from neuro- typically involves patients who have arguing that the transplanted hearts
logically devastated or imminently experienced profound neurologic in- would not function in the donors
dying patients. (6) (These poignant jury (but who do not meet the ac- (because external resuscitation
questions are addressed in greater de- cepted definition of brain death) and would not be offered) but could still
tail in another article in the Ethics for who are expected to die soon after function in the recipients with medi-
the Pediatrician series: Antommaria cessation of life-sustaining treatment cal assistance. (12) In light of these
AHM. Conceptual and ethical issues in (LST). Once the patient (or surro- nuanced distinctions, it is not sur-
the declaration of death: current con- gate decision maker) decides to with- prising that a recent survey of pedia-
sensus and controversies. Pediatr Rev. draw LST and consents to DCD, a tricians found that most were not
2010;10:427– 430.) defined period of pulselessness pre- confident that DCD donors were
A specific example of the debate cedes the declaration of death, at truly dead. (13)
over the dead donor rule was the which point the transplant team ini-
brief endorsement by the American tiates organ retrieval. (Uncontrolled Living Donation
Medical Association (AMA) of ante- DCD is relatively rare and refers to a Although vital organs can be pro-
mortem vital organ retrieval from in- situation in which a potential donor cured only from deceased donors,
fants who had anencephaly. Despite experiences cardiopulmonary arrest hematopoietic cells and solid organs
the American Academy of Pediatrics before informed consent for organ (such as kidneys and sections of lung
(AAP) statement in 1992 that retrieval is obtained.) and liver) may be procured from liv-
“[counseled] extreme caution before Ethical issues related to DCD in- ing donors. The former is less con-
adopting a policy permitting organ clude the required length of time of troversial than solid organ transplan-
retrieval from anencephalic infants pulselessness, the propriety of inter- tation because the risks to the donor
who retain brain stem function,” (7) ventions intended solely to maximize “can be characterized as more than
the AMA defended the practice in organ viability (such as anticoagula- minimal, but they are nonetheless
1994 as ethically permissible because tion and large-vessel cannulation), generally modest.” (14) Parents have
these infants have “never experi- and the involvement of the organ turned not only to existing siblings as
enced and will never experience con- procurement team only after death sources of hematopoietic stem cells
sciousness.” (8) This stance was has been declared. (9) It also is criti- for an affected child, but also to the
widely criticized, largely based on the cal that the decision to withdraw LST purposeful conception of additional
slippery slope argument: If one could be made independent of the decision children in the hopes of finding an
retrieve organs from anencephalic in- to donate organs to avoid divided ideal match. In some cases, parents
fants because they would never expe- loyalties. The full range of palliative have even used prenatal diagnosis to
rience consciousness, why not take care must be available to DCD do- produce a human leukocyte antigen-
organs from patients in a vegetative nors, and medications should be ad- matched donor (so-called “savior
state? The AMA rescinded its opin- ministered for the patient’s comfort siblings”). The AAP does not oppose
ion the following year, and since that alone and not with any intention of this practice as long as the younger
time there has been little discussion hastening death to maximize organ sibling is loved and cared for in his or
of retrieving vital organs from such viability. her own right (and, thus, is not used
infants. The cardiorespiratory definition solely as a means to an end). The
Cardiorespiratory criteria for de- of death and the dead donor rule AAP identifies five necessary condi-
termination of death have also been were major considerations in a recent tions for a minor to serve as a hema-
debated, largely because of the re- debate concerning pediatric heart topoietic stem cell donor: (14)
newed interest in retrieving organs transplantation following DCD. (10) 1. There are no other donor op-
from nonheart-beating donors. Be- Critics assert that there could not be tions (ie, no willing, able, and medi-
fore the codification of brain death, “irreversible” cessation of cardiac cally equivalent histocompatible
DCD was the only method of pro- function because such function was adult relative).
curing vital organs. Protocols for successfully reestablished in the re- 2. There is a strong and positive
controlled DCD were first designed cipients. (11) Defenders of the pro- relationship between donor and re-
in the 1990s as a response to an in- cedure draw a distinction between cipient.
sufficient supply of organs available “‘permanent’ (will not reverse) and 3. The recipient is likely to bene-
from patients declared dead by neu- ‘irreversible’ (cannot reverse) cessa- fit from the transplant.

70 Pediatrics in Review Vol.32 No.2 February 2011


Downloaded from http://pedsinreview.aappublications.org. Provided by Pakistan:AAP Sponsored on February 1, 2011
ethics for the pediatrician

4. Risks to the donor are mini- Group and the AAP argue that chil- the core principles of biomedical eth-
mized. dren may do so in certain situations. ics. Initial attempts at determining
5. Parental permission and (where The AAP cites a lower age limit of who would benefit from the limited
appropriate) child assent are obtained. 11 to 14 years to provide meaningful supply of transplantable organs in-
Solid organ donation entails assent (in light of the increased risk cluded “social worth” criteria, such
greater risks to the donor, both phys- compared with hematopoietic stem as religious observance and commu-
ical (including a mortality risk of 2 in cell donation). The five necessary nity involvement. It quickly became
10,000 for kidney donation (15) and conditions are analogous to those for apparent that such criteria were not
the chance of the donor eventually hematopoietic stem cell transplant, appropriate in a pluralistic society,
suffering from the same disease that with the following modifications: and prioritization for organs now
necessitated the transplant for the af- (16) generally takes into account time on
fected sibling) and psychological (in- 1. Both the recipient and the do- the waiting list, tissue or blood-type
cluding guilt if the transplant is not nor are highly likely to benefit (which matching, level of current need, and
successful and a loss of attention if it essentially requires the recipient to geographic region.
is). Given that the transplant is per- be an immediate family member Over time it has become clear that
formed for the benefit of someone while also excluding donors who some of these factors could place chil-
else, there can be no physical benefits have severe cognitive disabilities be- dren at a competitive disadvantage, de-
for the donor. There may be psycho- cause they could not appreciate the spite the child’s life expectancy follow-
logical benefits, however, such as benefit to their relative). ing transplant being greater than an
greater self-esteem and potentially 2. Surgical risk for the donor is equivalent adult’s. Children currently
more attention if the recipient subse- extremely low.
make up 1.6% of the national waiting
quently requires less intense care. 3. The minor freely assents to do-
list, with more than 70% of such chil-
Thus, although it would be impossi- nate without coercion (established
dren waiting for either a kidney or
ble to argue that it is in the donor’s by an independent advocacy team).
liver. (18) Recognizing their unique
medical best interest, it could be in Although much of the ethical de-
needs, Congress passed the Children’s
his or her overall best interest. This bate surrounding transplantation
Health Act in 2000, which required
would only be the case, however, if and pediatrics involves children as
the Organ Procurement and Trans-
the recipient were a close relative donors, there are also ethical issues
plant Network (OPTN) to formulate
(most often a minor sibling). involving children as recipients. An
pediatric-specific criteria for organ al-
Parents are the presumed example is the discovery of misattrib-
location. Rather than relying exclu-
decision-makers for their children, uted paternity through screening the
for whom they must make choices presumed father as a potential donor, sively on length of time on the waiting
based on what is in their child’s best an issue identified in up to 0.5% of list, subsequent OPTN criteria priori-
interest. Because the parents are also living kidney donations. (17) Argu- tize children for pediatric donor lungs,
concerned with that sibling’s best in- ments in favor of disclosure include adolescent donor hearts, and kidneys
terest, their loyalty is divided, which the duty of truth-telling and the re- procured from donors younger than
can make it extremely difficult to de- quirement of informed consent; the age of 35 years. Children younger
cide whether to proceed with dona- those opposed cite family preserva- than the age of 11 years receive further
tion. There is also the matter of the tion and patient confidentiality. At prioritization for kidney transplant in
assent of the child and his or her present, there does not appear to be light of the effects of renal failure on
susceptibility to coercion, given the consensus as to the appropriate re- development. (19)
charged emotional family landscape. sponse. At a minimum, transplant Obviously, a child cannot receive an
This dilemma has been highlighted programs should formulate a policy organ unless the parent or guardian
in the popular press in Jodi Picoult’s for responding to such situations, has the patient listed for transplant and
novel My Sister’s Keeper and the sub- and this potential discovery should gives permission for the procedure.
sequent motion picture. be disclosed to prospective donors This situation raises the ethical ques-
Whereas the Amsterdam Consen- before testing. tions of whether failure to seek trans-
sus Panel determined that minors plant constitutes medical neglect on
should not be permitted to donate Allocation of Organs the part of the parents and whether an
solid organs, (15) both the United Equitable distribution of scarce re- adolescent may refuse a potentially life-
States Live Organ Donor Consensus sources is a matter of justice, one of saving transplant. Answering these

Pediatrics in Review Vol.32 No.2 February 2011 71


Downloaded from http://pedsinreview.aappublications.org. Provided by Pakistan:AAP Sponsored on February 1, 2011
ethics for the pediatrician

References 11. Veatch RM. Donating hearts after car-


Summary 1. Robertson J. The dead donor rule. Hast diac death: reversing the irreversible. N Engl
J Med. 2008;359:672– 673
Cent Rep. 1999;6:6
• Vital organs may be procured 2. Arnold RM, Youngner SJ. The dead do- 12. Bernat JL. How the distinction be-
only from patients declared nor rule: should we stretch it, bend it, or tween “irreversible” and “permanent” illu-
dead (dead donor rule). abandon it? Kennedy Inst Ethics J. 1993;3: minates circulatory–respiratory death deter-
• Increasing numbers of organs 263–278 mination. J Med Phil. 2010;35:242–355
are being procured from 3. Task Force for the Determination of 13. Joffe AR, Anton NR, deCaen AR. Sur-
patients declared dead by Brain Death in Children. Guidelines for the vey of pediatricians’ opinions on donation
cardiorespiratory criteria determination of brain death in children. after cardiac death: are the donors dead?
following withdrawal of life- Arch Neurol. 1987;44:587–588 Pediatrics. 2008;122:e967– e974
sustaining treatment (DCD). 4. Veatch RM. The impending collapse of 14. American Academy of Pediatrics. Policy
• The AAP holds that children the whole-brain definition of death. Hast statement: children as hematopoietic stem cell
may serve as living Cent Rep. 1993;23:18 –24
donors. Pediatrics. 2010;125:392– 404
hematopoietic stem cell and 5. Halevy A, Brody B. Brain death: recon-
15. Delmonico F; Council of the Trans-
solid organ donors as long as ciling definitions, criteria, and tests. Ann
plantation Society. A report of the Amster-
certain conditions are met. Intern Med. 1993;119:519 –525
dam Forum on the care of the live kidney
• Recent modifications to the 6. Truog RT, Robinson WM. Role of brain
donor: data and medical guidelines. Trans-
OPTN allocation scheme death and the dead-donor rule in the ethics
plantation. 2005;79:S53–S66
prioritizes children for organs of organ transplantation. Crit Care Med.
2003;31:2391–2396 16. Ross LF, Thistlethwaite JR, and the
procured from other children Committee on Bioethics. Minors as living
(and, in some cases, young 7. Committee on Bioethics. Infants with
anencephaly as organ sources: ethical con- solid-organ donors. Pediatrics. 2008;122:
adults). 454 – 461
siderations. Pediatrics. 1992;6:1119
8. American Medical Association. Opinion 17. Young A, Kim SJ, Gibney EM, et al. for
2.162: Anencephalic infants as organ do- the Donor Nephrectomy Outcomes Re-
nors. Code of Medical Ethics. Chicago, IL: search (DONOR) Network. Discovering
American Medical Association; 1994:30 misattributed paternity in living kidney do-
questions depends largely on the type nation: prevalence, preference, and practice.
9. Antommaria AHM. Conceptual and
of transplant, the likelihood of success, ethical issues in the declaration of death: Transplantation. 2009;87:1429 –1435
and any comorbidities that may affect current consensus and controversies. Pedi- 18. Organ Procurement and Transplant
the child’s quality or duration of life. In atr Rev. 2010;10:427– 430 Network website. Accessed November
10. Boucek MM, Mashburn C, Dunn SM, 2010 at: http://optn.transplant.hrsa.gov
the case of the adolescent, the patient’s
et al, Denver Children’s Pediatric Heart 19. American Academy of Pediatrics. Policy
level of understanding and maturity as Transplant Team. Pediatric heart transplan- statement: pediatric organ donation and
well as state laws regulating emanci- tation after declaration of cardiocirculatory transplantation. Pediatrics. 2010;125:822–
pated or “mature” minors are relevant. death. N Engl J Med. 2008;359:709 –714 828

72 Pediatrics in Review Vol.32 No.2 February 2011


Downloaded from http://pedsinreview.aappublications.org. Provided by Pakistan:AAP Sponsored on February 1, 2011
Ethics for the Pediatrician: Ethical Issues in Organ Donation
Robert Macauley
Pediatr. Rev. 2011;32;69-72
DOI: 10.1542/pir.32-2-69

Updated Information including high-resolution figures, can be found at:


& Services http://pedsinreview.aappublications.org/cgi/content/full/32/2/69
Subspecialty Collections This article, along with others on similar topics, appears in the
following collection(s):
Ethics for the Primary Care Pediatrician
http://pedsinreview.aappublications.org/cgi/collection/ethics_pe
diatrician
Permissions & Licensing Information about reproducing this article in parts (figures,
tables) or in its entirety can be found online at:
http://pedsinreview.aappublications.org/misc/Permissions.shtml
Reprints Information about ordering reprints can be found online:
http://pedsinreview.aappublications.org/misc/reprints.shtml

Downloaded from http://pedsinreview.aappublications.org. Provided by Pakistan:AAP Sponsored on February 1, 2011


Research and Statistics: Distinguishing Statistical Significance from Clinical
Importance: The Value of the P Value
Sharon R. Ghazarian
Pediatr. Rev. 2011;32;73-74
DOI: 10.1542/pir.32-2-73

The online version of this article, along with updated information and services, is
located on the World Wide Web at:
http://pedsinreview.aappublications.org/cgi/content/full/32/2/73

Pediatrics in Review is the official journal of the American Academy of Pediatrics. A monthly
publication, it has been published continuously since 1979. Pediatrics in Review is owned,
published, and trademarked by the American Academy of Pediatrics, 141 Northwest Point
Boulevard, Elk Grove Village, Illinois, 60007. Copyright © 2011 by the American Academy of
Pediatrics. All rights reserved. Print ISSN: 0191-9601. Online ISSN: 1526-3347.

Downloaded from http://pedsinreview.aappublications.org. Provided by Health Internetwork on April 20, 2011


research and statistics

Distinguishing Statistical Significance


from Clinical Importance: The Value
of the P Value
Sharon R. Ghazarian, PhD* often are excited by a significant P
value, but that value does not neces-
Case Presentation sarily mean that the finding is impor-
You have been providing handouts on tant or useful.
infant massage to your families who Determining statistical signifi-
have newborns for several years. You cance involves hypothesis testing and
had read about the benefits of infant comparison of study results to a pre-
massage, including increased bonding determined level or P value. The level
between parent and child, improved of significance is the probability of
infant digestion, facilitation of at- reporting an incorrect association or
Author Disclosure tachment, improved sleep, and better committing a type I error. Conven-
Dr Ghazarian has disclosed no infant weight gain. (1) However, you tional P values usually are set at either
recently read an article showing the 0.05 or 0.01. A P value of 0.05 indi-
financial relationships relevant to
results of a comparative panel design cates that the researcher is 95% sure
this article. This commentary does study in which the association between that the finding is a true association
not contain a discussion of an infant massage and weight gain was and not just due to chance. A P value
unapproved/investigative use of a statistically significant at 2 months of 0.01 indicates that the researcher
commercial product/device. but not at 4 months of age for mothers is 99% sure that the finding is a true
and their infants in Santiago, Chile. association and that the finding
(2) You now wonder whether infant could be due to chance only 1% of
massage is as beneficial as you once the time. Thus, if a study reports that
thought and whether you should con- an association between infant mas-
tinue to provide the information to sage and infant weight gain is signif-
parents of newborns. icant at the 0.05 level, the association
can be considered as true 95% of the
Introduction time and possibly due to chance 5%
Many practitioners wonder whether of the time. (3)
a lack of statistical significance means Although P values measure
a lack of clinical importance for their chance of association, they do not
patients. If a study finds an associa- measure strength of association or
tion to be statistically significant, clinical importance. P values tend to
does it mean that the finding also is be smaller when there is a larger nu-
clinically important? Alternatively, if meric difference between the two
a study finds that an association is not groups, a larger sample size, or less
statistically significant, does it mean variability within treatment groups.
that the finding is not clinically im- (4) Values are also affected by the
portant for the families in a practitio- proportion of those exposed and the
ner’s practice? Statistical significance proportion who have the targeted
demonstrates the likelihood that a outcome. In fact, because of the lim-
difference or relationship really exists itations and misuses of P values, the
and did not just occur by chance. We International Committee of Medical
Journal Editors suggests that “when
possible, quantify findings and
*Director, Bayview Biostatistics, Epidemiology and
Data Management Core, Department of Pediatrics, present them with appropriate indi-
Johns Hopkins University, Baltimore, MD. cators of measurement error or un-

Pediatrics in Review Vol.32 No.2 February 2011 73


Downloaded from http://pedsinreview.aappublications.org. Provided by Health Internetwork on April 20, 2011
research and statistics

certainty (such as confidence inter- months, the clinical importance is ited statistical significance, suggests
vals). Avoid relying solely on unclear. What does the statistical dif- that infant massage is beneficial for
statistical hypothesis testing, such as ference in weight gain suggest and parents and newborns. Final deci-
the use of p-values, which fails to what does the nonstatistical differ- sions about what to recommend to
convey important quantitative infor- ence in weight gain at 4 months sug- families should be based on broad
mation.” (5) gest? literature searches, studies generaliz-
The authors of the infant massage In addition, the study used a sample able to a clinician’s patient popula-
study (2) found a statistically signifi- of 100 newborns in a low-income tion, and careful consideration of
cant association between infant mas- neighborhood of Santiago, Chile. Un- what is statistically and clinically im-
sage and infant weight gain at 2 less a practitioner has a practice with portant to the families being served.
months of age (5,537 g for the mas- the same demographic characteristics
sage group versus 4,672 g for the of the study sample, study results may
not be generalizable to other children. References
control group) with a P value of 1. Lorenz L, Moyse K, Surguy H. The ben-
0.0016. Because the P value is less To ascertain whether infant massage is
efits of baby massage. Paediatr Nurs. 2005;
than 0.01, there is less than a 1% beneficial, numerous studies would 17:15–18
likelihood that the findings are sim- need to be examined with samples of 2. Serrano MSC, Doren FM, Wilson L.
ply due to chance. Accordingly, there families possessing a variety of demo- Teaching Chilean mothers to massage their
graphic characteristics, including vari- full-term infants. J Perinat Neonat Nurs.
probably is an association at 2 2010;24:172–179
months. Is this degree of weight ous levels of income, education, family
3. Howell DC. Statistical Methods for Psy-
structure, race/ethnicity, parity, mari- chology. Pacific Grove, CA: Duxbury; 2002
change important? The study also
tal status, and more. 4. Cummings P, Koepsell TD. P values ver-
shows that the association between
sus estimates of association with confidence
infant massage and weight gain at 4 intervals. Arch Pediatr Adolesc Med. 2010;
months was not statistically signifi- Conclusion 164:193–196
cant (7,044 g for the massage group Practitioners should consider clinical 5. International Committee of Medical
versus 7,119 g for the control Journal Editors. Uniform Requirements for
importance independent from statis-
Manuscripts Submitted to Biomedical Jour-
group). Although there was a statis- tical significance, in this case asking nals: Writing and Editing for Biomedical
tically significant association between whether broader literature, beyond Publication. Accessed November 2010 at:
massage and weight gain at 2 the single study demonstrating lim- http://www.icmje.org

74 Pediatrics in Review Vol.32 No.2 February 2011


Downloaded from http://pedsinreview.aappublications.org. Provided by Health Internetwork on April 20, 2011
Research and Statistics: Distinguishing Statistical Significance from Clinical
Importance: The Value of the P Value
Sharon R. Ghazarian
Pediatr. Rev. 2011;32;73-74
DOI: 10.1542/pir.32-2-73

Updated Information including high-resolution figures, can be found at:


& Services http://pedsinreview.aappublications.org/cgi/content/full/32/2/73
Subspecialty Collections This article, along with others on similar topics, appears in the
following collection(s):
Research and Statistics
http://pedsinreview.aappublications.org/cgi/collection/research_s
tatistics
Permissions & Licensing Information about reproducing this article in parts (figures,
tables) or in its entirety can be found online at:
http://pedsinreview.aappublications.org/misc/Permissions.shtml
Reprints Information about ordering reprints can be found online:
http://pedsinreview.aappublications.org/misc/reprints.shtml

Downloaded from http://pedsinreview.aappublications.org. Provided by Health Internetwork on April 20, 2011


Index of Suspicion • Case 1: Nonbilious Projectile Vomiting in a Neonate • Case
2: Fever, Rash, Headache, Myalgia, and Arthralgia in an 11-year-old Boy • Case
3: Ecchymotic Lesions on the Backs of Asian Boys
John D.A. Campagna, Minela Fernandez, Lance Yamashiroya, Ronen Zipkin, Marwa
Moustafa, Shira Rosenberg, Ryan P. Flanagan and Neil Mullen
Pediatr. Rev. 2011;32;75-80
DOI: 10.1542/pir.32-2-75

The online version of this article, along with updated information and services, is
located on the World Wide Web at:
http://pedsinreview.aappublications.org/cgi/content/full/32/2/75

Pediatrics in Review is the official journal of the American Academy of Pediatrics. A monthly
publication, it has been published continuously since 1979. Pediatrics in Review is owned,
published, and trademarked by the American Academy of Pediatrics, 141 Northwest Point
Boulevard, Elk Grove Village, Illinois, 60007. Copyright © 2011 by the American Academy of
Pediatrics. All rights reserved. Print ISSN: 0191-9601. Online ISSN: 1526-3347.

Downloaded from http://pedsinreview.aappublications.org. Provided by Pakistan:AAP Sponsored on February 1, 2011


index of suspicion

Case 1: Nonbilious Projectile Vomiting in a


Neonate
Case 2: Fever, Rash, Headache, Myalgia, and
Arthralgia in an 11-year-old Boy
Case 3: Ecchymotic Lesions on the Backs of Asian
Boys
Case 1 Presentation sic metabolic panel yields normal re-
A 16-day-old boy is evaluated in the sults. He has an indirect hyperbili-
The reader is encouraged to write rubinemia of 17.3 mg/dL (295.9
ED for 5 days of projectile vomiting.
possible diagnoses for each case before
turning to the discussion. We invite Thirty minutes after breastfeeding, ␮mol/L). Blood, CSF, and urine
readers to contribute case presentations he vomits milk at distances up to 2 ft. cultures are sent.
and discussions. Please inquire first by He also has been having difficulty Ultrasonography of the abdomen
contacting Dr. Deepak Kamat at feeding, staying on the breast for shows a normal pylorus. Chest radio-
dkamat@med.wayne.edu. graphs as well as CT scan of the chest
only 5 minutes, and is sleepy. The
frequency of emesis has been increas- and abdomen reveal the diagnosis.
ing to after almost every feeding.
Author Disclosure
Urine output and stooling frequency
have decreased. Case 2 Presentation
Drs Campagna, Fernandez,
The infant was born at term to a An 11-year-old boy presents to the
Yamashiroya, Zipkin, Moustafa, 37-year-old G2P2 woman after an ED with a 9-day history of tempera-
Rosenberg, Flanagan, and Mullen uncomplicated pregnancy. His new- ture to 40.2°C and a 2-day history of
have disclosed no financial born screening results were normal. a nonpruritic rash that began on his
relationships relevant to these cases. He developed icterus on postnatal trunk and spread to all four extremi-
This commentary does not contain a day 2, with a peak bilirubin value of ties. The rash is most prominent
discussion of an unapproved/
22 mg/dL (376.3 ␮mol/L) and a when he is febrile. He has headaches,
positive direct antiglobulin test. The myalgias, and joint pain, which
investigative use of a commercial
hyperbilirubinemia responded to worsen with his fevers. Animal expo-
product/device. phototherapy within 24 hours. sures include stray cats, turtles, and a
Physical examination reveals an ic- rat infestation in his backyard. He has
teric, nondysmorphic infant in no ap- a history of bipolar disorder, seizures,
parent distress. His temperature is and hypothyroidism, for which he
36.6°C, heart rate is 132 beats/min, takes lithium, levetiracetam, and
respiratory rate is 48 breaths/min, levothyroxine sodium, respectively.
and oxygen saturation is 99%. He has In the ED, his temperature is
lost more than 10% of his birth- 40.0°C, heart rate is 133 beats/min,
weight. His fontanelle is open, soft, respiratory rate is 20 breaths/min,
and flat. His lungs are clear, and his and blood pressure is 88/43 mm
abdomen is soft and nondistended, Hg. Physical examination reveals a
without any organomegaly, but his well-nourished, well-developed boy
bowel sounds are hypoactive. in no acute distress. He has a faint,
Laboratory studies reveal a WBC blanching, macular exanthem on his
count of 13.0⫻103/mcL (13.0⫻ trunk, all four extremities, and both
109/L) with a normal differential palms and soles. There are several
count, Hgb of 17.6 g/dL (176 g/ nonblanching macules on his ankles
L), Hct of 50.7% (0.51), and platelet and left wrist.
count of 528⫻103/mcL (528⫻109/ Laboratory evaluation reveals a
L). His C-reactive protein (CRP) WBC count of 5.29⫻103/mcL
measures less than 0.1 mg/dL. A ba- (5.29⫻109/L), with 22% neutrophils,

Pediatrics in Review Vol.32 No.2 February 2011 75


Downloaded from http://pedsinreview.aappublications.org. Provided by Pakistan:AAP Sponsored on February 1, 2011
index of suspicion

50% bands, 21% lymphocytes, and ing, petechiae, hemoptysis, hema-


6% monocytes; Hgb of 11.8 g/dL tochezia, fatigue, or weight loss.
(118 g/L); and platelet count of When questioned directly about the
109⫻103/mcL (109⫻109/L). His lesions, neither boy makes eye con-
CRP is 19.8 mg/dL, ESR is 18 mm/ tact or provides an explanation of
hr, AST is 89 U/L, and ALT is how the lesions occurred. Additional
53 U/L. Results of a chemistry panel history reveals the cause for the find-
and a urinalysis are normal. Addi- ings.
tional testing reveals the diagnosis.

Case 1 Discussion
Case 3 Presentation The patient was hospitalized for fluid
Two Asian boys, ages 9 and 11 years, management. His chest radiograph
who are neighbors, present to the showed a radiopaque mass between
ED 1 day apart complaining of sore the right cardiac border and right
throat, fever, headache, and abdom- diaphragm (Fig. 3). CT scan of the
inal pain for about 1 week. Both boys chest showed a gastric volvulus in the
are otherwise healthy and have no Figure 1. Numerous ecchymotic linear
thoracic cavity (Fig. 4). He was taken
known chronic illnesses, medication lesions on the back. to the operating room, where a
use, or previous hospitalizations. larger-than-average paraesophageal
Physical examination reveals nor- ous ecchymotic lesions on his back hernia with proximal bowel hernia-
mal vital signs in both boys. How- that are approximately 3⫻10 cm tion was found. The volvulus was
ever, both boys exhibit an erythema- (Fig. 1), and the 11-year-old boy reduced surgically. His hyperbiliru-
tous oropharynx, exudates on has numerous round areas of ecchy- binemia resolved within 48 hours of
tonsils, and bilateral submandibular mosis approximately 8 cm in diame- hospitalization, and urine, CSF, and
lymphadenitis. The findings on der- ter across his back (Fig. 2). All other blood cultures were negative.
matologic examinations are of con- physical findings are normal.
cern. The 9-year-old boy has numer- On further questioning, neither Differential Diagnosis
boy has a history of epistaxis, bruis- Nonbloody, nonbilious (NBNB)
vomiting in an infant (0 to 3 months
of age) has a broad differential diag-
Frequently Used Abbreviations nosis. Nonbilious vomitus suggests
ALT: alanine aminotransferase that the lesion lies proximal to the
AST: aspartate aminotransferase ampulla of Vater. Bilious vomiting
BUN: blood urea nitrogen often connotes a serious condition.
CBC: complete blood count The most common cause of non-
CNS: central nervous system bilious vomiting in this patient’s
CSF: cerebrospinal fluid age group is gastroesophageal reflux
CT: computed tomography (GER), which results from abnormal
ECG: electrocardiography function of the lower esophageal
ED: emergency department sphincter (LES). In infants, this dys-
EEG: electroencephalography function usually stems from immatu-
ESR: erythrocyte sedimentation rity of the LES. GER resolves as the
rate child ages. Infants who have GER
GI: gastrointestinal usually gain weight properly and
GU: genitourinary have normal physical examination
Hct: hematocrit findings. Vomiting tends to occur
Hgb: hemoglobin within 30 minutes of feeding.
MRI: magnetic resonance imaging Figure 2. Numerous round areas of ec- Milk protein allergy is another
WBC: white blood cell chymosis approximately 8 cm in diame- common cause of NBNB vomiting.
ter across the back. The condition represents hypersensi-

76 Pediatrics in Review Vol.32 No.2 February 2011


Downloaded from http://pedsinreview.aappublications.org. Provided by Pakistan:AAP Sponsored on February 1, 2011
index of suspicion

tend to present with weight loss and is free to rotate in the thoracic cavity.
hypochloremic, hypokalemic meta- Such twisting can lead to further
bolic alkalosis. vomiting and may progress to stran-
Infants who have inborn errors gulation and infarction. This out-
of metabolism, increased intracranial come is rare because gastric volvulus
pressure, and nonaccidental trauma tends to resolve spontaneously.
often present with NBNB vomiting. Paraesophageal hernia usually can
be seen as an abnormality on chest
The Condition radiograph, as in this patient. The
Congenital paraesophageal hernia is most common finding is a cystic mass
a rare condition that is distinctively in the posterior mediastinum. Ap-
different from a sliding hiatal hernia. proximately one third of radiographs
Sliding hiatal hernias occur when the show an air-fluid level. An upper GI
gastroesophageal junction slides into radiographic series can help establish
Figure 3. Chest radiograph shows a ra-
the thorax. In contrast, paraesopha- the cystic mass as a part of the GI
diopaque mass between the right cardiac geal hernia is characterized by a por- tract. CT scan generally is not indi-
border and right diaphragm. tion of the stomach itself, usually cated but does show the details of
the fundus, dislocating into the tho- abnormal anatomy.
tivity to milk protein and occurs with racic cavity. Congenital paraesopha-
human, cow, and soy milk feedings. geal hernias result from two distinct
abnormalities in the development of
Treatment and Prognosis
Affected patients often are irritable, Treatment of paraesophageal hernia
have feeding intolerance, and can the diaphragm. In one abnormality
is surgical correction. The stomach is
have hematochezia. In the most se- that occurs in the development of
emptied by nasogastric suctioning,
vere cases, the children fail to thrive. the right pneumoenteric recess, the
the hernial sac is excised, and the
Vomiting can occur at any time. recess becomes flattened and elon-
diaphragm is repaired by crural ap-
Another important cause of gated, which allows the gastric fun-
proximation. Because it has been es-
NBNB vomiting is pyloric stenosis, dus to move through the diaphragm.
tablished that there is significant dis-
which occurs due to hypertrophy of The second abnormality occurs in
ruption of the normal anatomy, most
pyloric smooth muscle, resulting in the development of mesodermal cells
pediatric surgeons perform an anti-
blocked passage of gastric contents. around the aorta. In all cases, the
reflux procedure simultaneously.
Pyloric stenosis frequently occurs in gastric fundus is surrounded by a
Surgical correction is relatively safe,
boys between 2 and 6 weeks of age, peritoneal sac, and the mass resides
and corrected hernias are associated
but girls also are affected. Vomiting posterior to the heart and anterior to
with minimal morbidity. Usually,
usually occurs soon after feeding and the esophagus.
long-term complications are limited
often is projectile. History usually re- In both abnormalities, the gastric
to GER.
veals a progressively increasing inten- body migrates into the thoracic cav-
Paraesophageal hernias in chil-
sity and frequency of emesis. Patients ity. Depending on the size of the
dren are less likely to strangulate than
hernia, bowel can follow. Paraesoph-
in adults, even if there is radiographic
ageal hernias become symptomatic
evidence of volvulus.
when herniated stomach muscles
This patient tolerated his surgical
are unable to pass gastric contents
procedure well. He had no further
through the pylorus effectively.
episodes of emesis and gained weight
There is evidence that the herniated
normally.
viscus can obstruct the esophagus,
leading to vomiting that may be in-
termittent. Lessons for the Clinician
As the herniation progresses, the
risk of gastric volvulus increases. The ● NBNB vomiting in an infant is
abnormal position of the stomach common and usually represents
Figure 4. CT scan of the chest shows a weakens the ligamentous anchors of GER, but the clinician must be
gastric volvulus in the thoracic cavity. the normal stomach, and the fundus aware of more serious conditions,

Pediatrics in Review Vol.32 No.2 February 2011 77


Downloaded from http://pedsinreview.aappublications.org. Provided by Pakistan:AAP Sponsored on February 1, 2011
index of suspicion

such as gastric volvulus, and act Mountain spotted fever (RMSF), Less often, infectious causes such as
quickly. caused by R rickettsii, often have a Mycoplasma infection have been im-
● Early diagnosis improves the suc- rash that begins on the wrists and plicated. The rash of SJS typically
cess of surgical intervention and ankles and spreads centrally to in- involves mucous membranes (con-
overall prognosis considerably. volve the trunk and extremities. junctivitis, urethritis, oral ulcers) and
Blanching macules and papules can include bullae associated with a
(John D.A. Campagna, MD, MPH,
evolve into petechiae over time and positive Nikolsky sign.
TM, Minela Fernandez, MD, Lance
can include the palms and soles. This patient’s persistent fever and
Yamashiroya, MD, Tripler Army
RMSF is a life-threatening illness and complaints of joint pain and myalgias
Medical Center, Honolulu, HI)
rarely has a benign course when left raised the possibility of systemic JIA.
untreated. Ehrlichiosis may present However, children who have JIA
with a maculopapular rash similar to typically demonstrate joint stiffness
Case 2 Discussion RMSF but rarely is petechial and of- after periods of prolonged inactivity
While in the hospital, the boy’s fever ten has a variable distribution. rather than unremitting myalgias and
persisted, with a maximum tempera- Salmon-pink macules or copper- joint pain. Fevers classically involve
ture of 40.4°C. He was started em- colored papules on the palms and temperatures greater than 39.0°C
pirically on intravenous cefotaxime soles are suggestive of secondary and occur in one to two peaks at the
and doxycycline. With ongoing my- syphilis and typically appear 4 to same time each day. Fever must be
algias associated with limited range 8 weeks after the primary infection. present for more than 2 weeks to
of motion in his shoulders, rheuma- The rash associated with measles be- meet diagnostic criteria. The rash of-
tology was consulted to evaluate for gins on the face and spreads in a ten is characterized as salmon pink,
systemic-onset juvenile idiopathic ar- cephalocaudal distribution, occur- nonpruritic, and primarily affecting
thritis (JIA). Laboratory results were ring 2 to 4 days after the initial pro- the trunk and extremities.
negative for Mycoplasma titers; Bar- drome of cough, coryza, and con-
tonella titers; parvovirus and adeno- junctivitis. The measles rash typically The Condition
virus polymerase chain reaction test- is erythematous and can include Murine typhus (flea-borne typhus)
ing; and blood, urine, and stool purple-red macules or papules. Ru- is caused by R typhi and typically
cultures. His rickettsial typhus im- bella can have a similar distribution constitutes a less severe illness than
munoglobulin M titer was positive and may be scarlatiniform or morbil- epidemic typhus (louse-borne R
at 1:512, supporting the diagnosis liform, at times being confused with prowazekii). Rats are the primary res-
of murine typhus. He defervesced acne. Neither rash typically involves ervoir for R typhi, although cats in-
within 72 hours of starting cefo- the palms or soles. fested with fleas also can serve as a
taxime and doxycycline and was dis- Dengue fever also can present reservoir. The rat flea Xenopsylla
charged from the hospital to com- with a distinct rash, often seen in two Cheopis is the vector usually respon-
plete a 10-day course of doxycycline separate phases, and is associated sible for rodent-to-human transmis-
monotherapy. His rash and com- with headache and myalgias. The first sion. After feeding on infected ro-
plaints of myalgias were no longer rash is transient, macular, and ap- dents, fleas become carriers for life.
present on discharge. pears within 1 to 2 days of the onset Humans become infected when con-
of fever. The second rash occurs 1 to tact with flea feces occurs through
Differential Diagnosis 2 days after defervescence, can be a compromised skin barrier. Most
Although the differential diagnosis maculopapular or morbilliform, and cases have been reported in Califor-
of fever and rash is broad, the type typically spares the palms and soles. nia, Texas, and Hawaii.
of rash and its location can be help- Stevens-Johnson syndrome (SJS) Laboratory findings can include
ful in narrowing the possibilities. should be considered among patients leukopenia, elevated inflammatory
Maculopapular rashes that start on taking mood-stabilizing medications markers, and slightly elevated liver
the trunk and extend to the extrem- such as lithium. SJS more often is transaminases. Symptoms can resem-
ities, palms, and soles (ie, centrifugal associated with the use of antiepilep- ble other rickettsial infections and
rash) include rickettsial diseases such tic agents such as phenytoin, carbam- can include fever, chills, headaches,
as infection with louse-borne R azepine, and valproic acid as well as and myalgias. The rash occurs in ap-
prowazekii, R felis, and flea-borne R nonsteroidal anti-inflammatory agents proximately 60% of patients and
typhi. Patients who have Rocky and sulfa-containing medications. generally presents 4 to 7 days after

78 Pediatrics in Review Vol.32 No.2 February 2011


Downloaded from http://pedsinreview.aappublications.org. Provided by Pakistan:AAP Sponsored on February 1, 2011
index of suspicion

the onset of symptoms. The rash typ- Case 3 Discussion created between the cup and the
ically is macular or maculopapular Both boys had positive rapid strepto- skin, resulting in bruising of the skin.
and often remains discrete with coccal latex agglutination tests and Several cultural healing methods
sparse lesions. The illness rarely lasts were started on penicillin therapy. are mistaken for child abuse, the
longer than 2 weeks, although severe Because of concern about the ecchy- most common being cupping, co-
cases that are not treated can be fatal. moses, screening coagulation studies ining, moxibustion, and spooning.
were performed, and results were Moxibustion is a Southeast Asian
Treatment within normal limits. Both boys were practice considered to be a variation
The drug of choice for murine ty- interviewed without adult supervi- of acupuncture in which a Chinese
phus is doxycycline (2.2 mg/kg ev- sion and still were not willing to pro- herb (Artemisia vulgaris) is burned
ery 12 hours to a maximum of vide an explanation for the bruis- onto the skin using an instrument
300 mg/day). The duration of fe- ing on their backs. Child protective such as yarn or incense near the af-
brile illness averages 12 days, with a services (CPS) was contacted to ini- fected area to draw out “the evil.”
median time to defervescence of tiate an evaluation for nonaccidental Spooning is a practice in China that
2 days. Therapy often is continued trauma in both cases. is similar to coining.
for 3 days after resolution of fever. Asian folk healers have been prac-
Although most patients appear well Diagnosis ticing these techniques in major met-
and do not require intensive care, On further questioning of the par- ropolitan cities that have large Asian
complications can include renal fail- ents, it was learned that an Asian folk populations. In both these cases, the
ure, meningoencephalitis, respira- healer had treated both boys earlier ED physicians taking care of these
tory failure, and myocarditis. Prompt that week. The 9-year-old boy was boys were not aware of these healing
treatment can shorten the duration treated with coining and the 11-year- methods, despite practicing medi-
of fever, and relapses are very rare old boy with cupping. cine in an institution where many of
when patients are treated properly. their patients came from ethnic and
Postexposure prophylaxis is not indi- The Condition cultural populations that practice
cated, but R typhi infections should The practices of cupping and coining such faith-healing methods. Because
be reported to local or federal public are believed to heal individuals af- the patients had physical findings
health agencies. fected by respiratory and febrile ill- of bruising in unusual patterns over
nesses. Coining (cao gao) is per- the back without explanation, the
Lessons for the Clinician formed by massaging the back or ED physicians suspected nonacci-
chest with mentholated oil, then ag- dental trauma, thereby warranting
● The combination of fever and rash
gressively scraping the edge of a coin reports to CPS, which determined
is a common presenting complaint
back and forth linearly until bruising that the bruising was the result of
in children, and most patients who
or petechiae develop. Coining is not nonharmful attempts to heal the pa-
have these signs have a benign, self-
intended to cause permanent bodily tients.
limited illness.
harm and is a widespread practice These procedures had parental
● A subset of patients have pro-
throughout much of Vietnam and consent and, therefore, did not con-
longed symptoms that require a
parts of Southeast Asia. Although stitute nonaccidental trauma. In
more thorough history and physi-
coining typically does not inflict seri- both cases, the intent behind the
cal examination, including a travel
ous harm, there is a reported case of a treatment methods was based on cul-
history as well as a detailed history
renal contusion with hematuria fol- turally accepted traditional beliefs.
of animal exposures.
lowing coining. Thus, it was determined that the in-
● Narrowing the differential diagno-
Cupping is a traditional folk heal- juries were not a result of physical
sis according to physical findings
ing practice rooted in Latin Ameri- abuse, and CPS released the boys to
can lead to a more cost-effective
can, Asian, and Eastern European their parents.
evaluation and quicker resolution
beliefs. For cupping, a small amount Most bruising in children is due
of symptoms.
of alcohol is ignited within a glass cup to accidental trauma, and 93% of
(Ronen Zipkin, MD, Marwa to heat the air inside the container. accidental bruises occur over bony
Moustafa, MD, Shira Rosenberg, MD, The cup is then placed on the bare prominences. In the ambulatory
Children’s Hospital of Los Angeles, Los skin of the affected individual. As the child, accidental bruises generally oc-
Angeles, CA) air within the cup cools, a vacuum is cur on the extensor surfaces of the

Pediatrics in Review Vol.32 No.2 February 2011 79


Downloaded from http://pedsinreview.aappublications.org. Provided by Pakistan:AAP Sponsored on February 1, 2011
index of suspicion

lower leg, the anterior thigh, and the child’s evaluation, and approximately Lessons for the Clinician
forehead. Accidental bruising on the 50% of children in whom an inflicted ● Health-care practitioners must
chest, abdomen, back, buttocks, soft fracture is diagnosed have more than
consider the possibility of physical
tissue of the face, ears, and “pro- one fracture at the time of diagnosis.
abuse when faced with a child who
tected areas” such as the neck, genital For children older than 2 years of
has suffered an unexplained trau-
area, and inner thighs is uncommon age, further evaluation for underly- matic injury.
in children of all ages. The presence ing injuries should be initiated if the ● Lack of awareness of the cultural
of a bruise in any of these uncommon patient is symptomatic. and traditional healing practices
locations may signal intentional in- The differential diagnosis for used by various communities can
jury and possible serious underly- bruising includes Mongolian spots, lead a health-care professional to
ing pathology such as intracranial phytophotodermatitis, coagulopathy, suspect potential physical abuse
injury, skeletal fracture, or intra- Henoch-Schönlein purpura, Ehlers- when encountering certain physi-
abdominal trauma, particularly in a Danlos syndrome, erythema multi- cal findings.
child younger than 2 years of age. forme, and cultural practices such ● Although the stigma of suspected
Bruises due to intentional trauma as “coining” and “cupping.” Most of child abuse can be painful for fam-
often assume a pattern that reflects these diagnoses can be distinguished ilies, it is extremely important to
the shape of the impacting object. based on the history and physical and investigate fully any suspected non-
Common “patterned” inflicted inju- laboratory evaluation to rule out accidental trauma to prevent fur-
ries include pinch marks; human bleeding disorders. ther harm to children.
bites; hand slaps; ligature marks Because of the high morbidity and
(Ryan P. Flanagan, MD, Chil-
around the neck or extremities; and mortality associated with child abuse,
dren’s Hospital of Pittsburgh Heart
marks from electrical cords, belts, health-care personnel need to be
Center, Pittsburgh, PA; Neil Mullen,
brushes, shoes, and kitchen utensils. sensitive to the possibility of abuse.
MD, Department of Pediatrics, Ma-
In the case of a child younger than Although reporting suspected child
digan Army Medical Center, Fort
2 years of age who has suspicious abuse might cause discomfort for the
Lewis, Tacoma, WA)
bruises, the clinician should consider family, not reporting child abuse may
further evaluation for abuse (such as cause significant harm to the child. To view Suggested Reading lists for
imaging studies) because 43% of all However, the clinician should be these cases, visit http://pedsinreview.
inflicted skeletal fractures are unsus- aware of cultural practices that may aappublications.org and click on
pected clinically at the time of the appear to hurt a child but do not. “Index of Suspicion.”

80 Pediatrics in Review Vol.32 No.2 February 2011


Downloaded from http://pedsinreview.aappublications.org. Provided by Pakistan:AAP Sponsored on February 1, 2011
Index of Suspicion • Case 1: Nonbilious Projectile Vomiting in a Neonate • Case
2: Fever, Rash, Headache, Myalgia, and Arthralgia in an 11-year-old Boy • Case
3: Ecchymotic Lesions on the Backs of Asian Boys
John D.A. Campagna, Minela Fernandez, Lance Yamashiroya, Ronen Zipkin, Marwa
Moustafa, Shira Rosenberg, Ryan P. Flanagan and Neil Mullen
Pediatr. Rev. 2011;32;75-80
DOI: 10.1542/pir.32-2-75

Updated Information including high-resolution figures, can be found at:


& Services http://pedsinreview.aappublications.org/cgi/content/full/32/2/75
Supplementary Material Supplementary material can be found at:
http://pedsinreview.aappublications.org/cgi/content/full/32/2/75/
DC1
Subspecialty Collections This article, along with others on similar topics, appears in the
following collection(s):
Infectious Diseases
http://pedsinreview.aappublications.org/cgi/collection/infectious
_diseases Psychosocial Issues and Problems
http://pedsinreview.aappublications.org/cgi/collection/psychosoc
ial_issues_problems Fetus and Newborn Infant
http://pedsinreview.aappublications.org/cgi/collection/fetus_new
born_infant Skin Disorders
http://pedsinreview.aappublications.org/cgi/collection/skin_disor
ders Emergency Care
http://pedsinreview.aappublications.org/cgi/collection/emergenc
y_care Gastrointestinal Disorders
http://pedsinreview.aappublications.org/cgi/collection/gastrointe
stinal_disorders
Permissions & Licensing Information about reproducing this article in parts (figures,
tables) or in its entirety can be found online at:
http://pedsinreview.aappublications.org/misc/Permissions.shtml
Reprints Information about ordering reprints can be found online:
http://pedsinreview.aappublications.org/misc/reprints.shtml

Downloaded from http://pedsinreview.aappublications.org. Provided by Pakistan:AAP Sponsored on February 1, 2011


Cataracts
Karen M. Davenport and Archana A. Patel
Pediatr. Rev. 2011;32;82-83
DOI: 10.1542/pir.32-2-82

The online version of this article, along with updated information and services, is
located on the World Wide Web at:
http://pedsinreview.aappublications.org/cgi/content/full/32/2/82

Pediatrics in Review is the official journal of the American Academy of Pediatrics. A monthly
publication, it has been published continuously since 1979. Pediatrics in Review is owned,
published, and trademarked by the American Academy of Pediatrics, 141 Northwest Point
Boulevard, Elk Grove Village, Illinois, 60007. Copyright © 2011 by the American Academy of
Pediatrics. All rights reserved. Print ISSN: 0191-9601. Online ISSN: 1526-3347.

Downloaded from http://pedsinreview.aappublications.org. Provided by Pakistan:AAP Sponsored on February 1, 2011


in brief

In Brief
Cataracts
Karen M. Davenport, MD Cataracts are a leading cause of pre- CHARGE (coloboma, heart defects,
Archana A. Patel, MD ventable blindness, responsible for 5% choanal atresia, retardation of growth,
University of Arizona College of to 20% of childhood blindness cases genital abnormalities, ear abnormali-
Medicine worldwide. In the United States, the prev- ties) syndrome, Alport syndrome, Lowe
Tuscon, Arizona alence of cataracts in children younger syndrome, and Sturge-Weber syndrome.
than 18 years of age ranges from 1 to 6 Unilateral cataracts are generally caused
per 10,000. A cataract is an opacity of by primary ocular disorders in the ab-
Author Disclosure the lens and is considered visually sig- sence of trauma. These abnormalities
Drs Davenport, Patel, and Serwint nificant when greater than 3 mm and include posterior lenticonus, aniridia,
have disclosed no financial centrally located. The World Health Or- microphthalmos, persistent hyperplas-
ganization (WHO) defines cataract as a tic primary vitreous, and anterior seg-
relationships relevant to this In Brief.
cause of blindness in which sight can ment dysgenesis.
This commentary does not contain a be restored through tertiary prevention. Multiple metabolic diseases are as-
discussion of an unapproved/ Provision of quality surgery within an sociated with cataracts, the most com-
investigative use of a commercial optimal time frame is one of the WHO’s mon being galactosemia. Cataracts in
product/device. targets for Vision 2020, a global initia- infants born with galactosemia can
tive to eliminate preventable causes of resolve if the infant is changed to a
blindness. lactose-free formula soon after birth.
Cataracts may occur at any age. Other metabolic conditions that cause
Eye Examination in Infants, Children
and Young Adults by Pediatricians.
Approximately 50% of congenital cat- cataracts are associated with hypopar-
American Academy of Pediatrics, aracts are idiopathic; the remainder athyroidism, hypoglycemia, and copper
Committee on Practice and Ambula- are due to congenital infections, ter- metabolism disorders. Hence, it is es-
tory Medicine and Section on Oph- atogens, genetic or metabolic disease, sential to perform an eye examination
thalmology. Pediatrics. 2003;111: and ocular disorders. Despite improved on children who exhibit failure to thrive
902–907 prenatal care and immunizations, con- and have suspected metabolic disease.
Preferred Practice Pattern Guidelines: genital infections remain a major source After infancy, most cataracts occur
Pediatric Eye Evaluations. American of cataracts, particularly in developing from trauma, systemic disease (such as
Academy of Ophthalmology Pediatric nations. Although rubella is the most juvenile idiopathic arthritis), or thera-
Ophthalmology/Strabismus Panel. San common infectious cause of congenital peutic intervention (ie, corticosteroid use
Francisco, CA: American Academy of cataracts, other congenital infections, and radiation).
Ophthalmology; 2007:1–39 such as cytomegalovirus, varicella, her- The major cause of cataract-
Infantile Cataracts. Krishnamurthy R, pes simplex, toxoplasmosis, and syphi- associated blindness is deprivation am-
VanderVeen DK. Int Ophthalmol Clin. lis, may cause them. Teratogens, such blyopia. There appears to be a latent
2008;48:175–192 as alcohol and corticosteroids, may also period during which visual deprivation
Pediatric Cataract: The Toronto cause congenital cataracts. is reversible before final vision is af-
Experience-Etiology. Lim Z, Rubab S,
Bilateral cataracts are more com- fected. Therefore, timely identification
Chan YH, Levin A. Am J Ophthalmol.
monly identified with genetic disease, and treatment of cataracts is essential.
2010;149:887– 892
often related to autosomal dominant Optimally, surgical intervention for con-
Advances in the Management of
Congenital and Infantile Cataract. traits. Cataracts are associated with nu- genital cataracts should occur within
Lloyd IC, Ashworth J, Biswas S, Abadi merous genetic syndromes, the most 6 weeks of birth for unilateral cataracts
RV. Eye. 2007;21:3101–1309 common being trisomies 13, 18, and 21. and 10 weeks for bilateral cataracts.
Evaluation of Screening Procedures for Although less frequent, cataracts may Unfortunately, this time frame often is
Congenital Cataracts. Magnusson G, also be seen in WAGR syndrome (Wilms not met. This short window for inter-
Jakobsson P, Kugelberg J, et al. Acta tumor, aniridia, genital abnormalities, vention makes it crucial for pediatric
Paediatr. 2003;92:1468 –1473 retardation), neurofibromatosis type 2, clinicians to identify and refer any chil-

82 Pediatrics in Review Vol.32 No.2 February 2011


Downloaded from http://pedsinreview.aappublications.org. Provided by Pakistan:AAP Sponsored on February 1, 2011
in brief

dren who have abnormal eye findings follow objects. The presence of nystag- tion is performed in the newborn nurs-
promptly to a pediatric ophthalmolo- mus or squinting warrants a referral. At ery and in the first postnatal week. At
gist. Studies have emphasized the im- 3 years of age, it is reasonable to begin times, seeing the red reflex in a new-
portance of nursery screenings in timely testing visual acuity. In addition, paren- born can be a challenge, but strategies
diagnosis and treatment of cataracts. tal observation of any ocular abnormal- such as darkening the room or lifting
The nursery examination, combined with ities and a family history of childhood an infant from a horizontal to vertical
the American Academy of Pediatrics cataracts are valuable determinants of position result in the infant opening his
recommended newborn visit at 3 to an infant’s risk. or her eyes. Most causes of unilateral
5 days, offers at least two opportunities In general, the operative outcome cataracts are idiopathic, but when the
for diagnosis within the recommended of congenital cataracts is good. Many cataracts are bilateral, it is prudent to
time frame for surgical intervention. children can achieve a visual acuity of
search further for a cause, whether
Physical findings that should prompt 20/60. Complications of surgery in-
genetic, metabolic, or infectious. Of
referral to an ophthalmologist include clude, but are not limited to, visual axis
interest, the first genetic mapping of
abnormal red reflex, structural abnor- opacifications, glaucoma, retinal detach-
an autosomal disease in humans was
malities, or any irregularity or asymme- ment, and strabismus. Children should
accomplished for congenital cataracts.
try of the pupils. Ideally, the red reflex have close follow-up evaluation by an
examination should be performed in a ophthalmologist following surgery to Although congenital cataracts from
darkened room, with each eye exam- ensure optimal visual outcome. rubella infection have decreased in fre-
ined individually from a distance of 1 to quency in developed countries, they still
2 feet. Immediate referral is warranted Comment: I still remember reading present a problem in countries where
for the findings of an absent red reflex, in college biology class about the ex- the measles, mumps, rubella vaccine is
dark spots in the red reflex, or a white periments in which a cat’s eye was not universally administered. Research-
reflex. The Bruckner test should follow sutured shut to identify the critical ers are investigating best practices for
the red reflex examination. In this eval- time for visual development and the intraocular lenses and approaches to
uation, both eyes are viewed simulta- resultant harm to visual acuity from decrease the postoperative morbidities
neously using an ophthalmoscope from light deprivation. Cataracts cause a from cataract surgery, such as glau-
a distance of 2 to 3 feet, allowing the similar light deprivation. Because the coma and the inflammatory response.
viewer to note asymmetry of the red early identification and treatment of
reflex that may suggest amblyopia. cataracts can prevent blindness, pedia-
After age 3 months, it is important tricians have an incredibly important Janet R. Serwint, MD
also to test a child’s ability to fix and role in ensuring that an eye examina- Consulting Editor, In Brief

Pediatrics in Review Vol.32 No.2 February 2011 83


Downloaded from http://pedsinreview.aappublications.org. Provided by Pakistan:AAP Sponsored on February 1, 2011
Cataracts
Karen M. Davenport and Archana A. Patel
Pediatr. Rev. 2011;32;82-83
DOI: 10.1542/pir.32-2-82

Updated Information including high-resolution figures, can be found at:


& Services http://pedsinreview.aappublications.org/cgi/content/full/32/2/82
Subspecialty Collections This article, along with others on similar topics, appears in the
following collection(s):
Genetics/Dysmorphology
http://pedsinreview.aappublications.org/cgi/collection/genetics_
dysmorphology Disorders of the Eye
http://pedsinreview.aappublications.org/cgi/collection/eye_disor
ders
Permissions & Licensing Information about reproducing this article in parts (figures,
tables) or in its entirety can be found online at:
http://pedsinreview.aappublications.org/misc/Permissions.shtml
Reprints Information about ordering reprints can be found online:
http://pedsinreview.aappublications.org/misc/reprints.shtml

Downloaded from http://pedsinreview.aappublications.org. Provided by Pakistan:AAP Sponsored on February 1, 2011


Balance and Vertigo in Children
Jacques Benun
Pediatr. Rev. 2011;32;84-85
DOI: 10.1542/pir.32-2-84

The online version of this article, along with updated information and services, is
located on the World Wide Web at:
http://pedsinreview.aappublications.org/cgi/content/full/32/2/84

Pediatrics in Review is the official journal of the American Academy of Pediatrics. A monthly
publication, it has been published continuously since 1979. Pediatrics in Review is owned,
published, and trademarked by the American Academy of Pediatrics, 141 Northwest Point
Boulevard, Elk Grove Village, Illinois, 60007. Copyright © 2011 by the American Academy of
Pediatrics. All rights reserved. Print ISSN: 0191-9601. Online ISSN: 1526-3347.

Downloaded from http://pedsinreview.aappublications.org. Provided by Pakistan:AAP Sponsored on February 1, 2011


in brief

In Brief
Balance and Vertigo in Children
Jacques Benun, MD Balance is a motor function that en- The most common cause of vertigo
Warren Alpert Medical School ables a person to control coordinated in children is middle ear disease, which
Brown University movements. Any dysfunction of the may lead to balance disorders in walking.
Providence, RI neuroaxis that commands this control The external and middle ears should be
results in a disturbance of balance, examined for evidence of effusion, acute
Author Disclosure leading to symptoms of dizziness. Ver- otitis media, or malformations such as
Drs Benun and Serwint have tigo is included under the subcategory cholesteatoma or perilymphatic fistula.
disclosed no financial relationships of dizziness. Perilymphatic fistula may follow minor
relevant to this In Brief. This The definition of vertigo is a sensa- head trauma, resulting from a rupture
commentary does not contain a tion of movement that may be rota- of the round or oval window of the
discussion of an unapproved/ tional or moving forward and can in- middle ear into the vestibule, creating a
investigative use of a commercial clude tilting. Vertigo may result from fistula. Symptoms from middle ear dis-
product/device. abnormalities of the labyrinth system or ease tend to improve after appropriate
the central nervous system that spans medical treatment. These diseases are
from the vestibular nerve to the brain- very familiar to pediatricians and can
Vertigo. MacGregor DL. Pediatr Rev. stem and cerebellum. As with any dys- easily be diagnosed by otoscopic exam-
2002;23:10 –15 function, the key to correct diagnosis ination.
Idiopathic Benign Paroxysmal Vertigo is performing a complete history, phys- It is more challenging to develop a
in Children, A Migraine Precursor. ical examination, and further testing differential diagnosis of vertigo in chil-
Ralli G, Atturo F, deFilippis C. based on clinical indications. dren who have normal tympanic mem-
Int J Pediatr Otorhinolaryngol.
Obtaining an accurate history in branes. Potential causes include:
2009;73(suppl):S16 –S18
young children may be difficult be- ●
Various Causes and Clinical Character- Benign paroxysmal vertigo (BPV)
cause they may not have the vocabu-
istics in Vertigo in Children With ● Migraine
Normal Eardrums. Choung YH, lary to describe the sensation of vertigo. ● Vestibular neuritis due to viral infec-
Park K, Moon SK, Kim CH, Ryu SJ. However, some authors suggest using
tions
Int J Pediatr Otorhinolaryngol. playground terms such as “spinning, ● Head trauma
2003;67:889 – 894 swinging, sliding, or being on the merry- ● Ménière disease
Etiology of Vertigo in Children. go-round.” Symptoms associated with ● Cerebellar and brainstem tumors
Balatsouras DG, Kaberos A, vertigo include nausea, vomiting, pal-
Assimakopoulos D, lor, and perspiration but not loss of After middle ear disease, BPV is the
Katatomichelakis M, Economou NC, consciousness. Additional historical ques- most common cause of vertigo in chil-
Korres SG. Int J Pediatr Otorhino- tions should address head trauma, fe- dren. The age of onset ranges between
laryngol. 2007;71:487– 494 ver, infections, and family history of 2 and 12 years, with an average age of
Too Young to Talk of Vertigo? vertigo or migraines. onset of 6 years. The condition is more
Miyahara M, Hirayma M, Yuta A, The physical examination focuses on common in females and typically pre-
Takeuchi K, Inoki T. Lancet.
evaluation of the external and middle sents as a sudden onset of vertigo that
2009;373:516
ear; ophthalmologic assessment for signs last seconds to minutes, with complete
Meniere’s Disease in Childhood. Akagi H,
of nystagmus; and neurologic exami- resolution of symptoms between epi-
Yuen K, Maeda Y, et al. Int J Pediatr
Otorhinolaryngol. 2001;61:259 –264 nation of the cranial nerves, cerebellar sodes. The attack is not induced by head
Syncope. Delgado CA. In: Fleisher GR, function, and gait. Additional testing to movements or positional changes. As-
Ludwig S, eds. Textbook of Pediatric consider includes hearing testing and sociated symptoms may include nausea
Emergency Medicine. 6th ed. Phila- audiometry. Brain imaging, such as mag- and vomiting, and abnormal eye move-
delphia, PA: Lippincott Williams & netic resonance imaging, is only needed ments such as nystagmus may be
Wilkins, a Wolters Kluwer business; if the history and physical findings present, but there are no neurologic
2010:589 –595 suggest an intracranial process. changes or altered state of conscious-

84 Pediatrics in Review Vol.32 No.2 February 2011


Downloaded from http://pedsinreview.aappublications.org. Provided by Pakistan:AAP Sponsored on February 1, 2011
in brief

ness. The diagnosis is based on the Ménière disease is another con- panic attacks, and depression may
brevity of the symptoms, presence of sideration when evaluating vertigo. Al- present as a result of prolonged vertigo
nystagmus during the episodes, though rare in children, the prevalence and should be treated accordingly.
symptom-free intervals, and frequently ranges from 1.5% to 4% among chil-
a family history of migraines. BPV is dren in whom vertigo is diagnosed, and Comment: Vertigo can be challeng-
included among the periodic syndromes cases have been described as young as ing to diagnose, especially in young
of childhood and may be considered a 4 years of age. The key symptoms are children. Pediatricians need to be aware
migraine equivalent. The prognosis is tinnitus, vertigo, and progressive hear- that the causes of vertigo in children
good and the symptoms disappear ing loss. differ from those in adults. Vertigo may
spontaneously before adolescence, al- Cerebellar and brainstem tumors are be diagnosed infrequently in young
though a subset of patients may de- rare causes of vertigo. Associated find- children because symptoms may be in-
velop migraines in the future. Migraines ings include headache, emesis, gait dis- correctly attributed to their develop-
must also be considered as a potential turbance, and papilledema. Brain imag- mental status and lack of coordination
cause of vertigo. ing is indicated. rather than to a balance dysfunction,
Vestibular neuritis is caused by Treatment of vertigo depends on the and young children may have difficulty
viruses such as herpes simplex, varicella- cause and duration of symptoms. For describing their symptoms. Pediatri-
zoster, and influenza. Symptoms of nau- acute symptoms that resolve quickly, cians must be skilled in differentiating
sea and vomiting develop suddenly and reassurance may be all that is needed. the common causes of vertigo, such as
may persist for weeks. Medications to suppress symptoms in- middle ear disease, which is easily
Head trauma with or without tempo- clude promethazine and diazepam. Per- treated, and BPV, which resolves, from
ral bone fracture can cause labyrinthine sistent severe nausea or vomiting can rarer but more serious conditions that
concussion. Affected patients develop be treated with antiemetics such as may be progressive and require more
symptoms immediately after the trauma. metoclopramide or prochlorperazine. If thorough testing and imaging.
The child is unsteady and tends to fall the vertigo becomes chronic or recur-
toward the affected side. The symptoms rent, clonazepam or carbamazepine Janet R. Serwint, MD
usually improve after 4 to 6 weeks. may be indicated. Symptoms of anxiety, Consulting Editor, In Brief

Pediatrics in Review Vol.32 No.2 February 2011 85


Downloaded from http://pedsinreview.aappublications.org. Provided by Pakistan:AAP Sponsored on February 1, 2011
Balance and Vertigo in Children
Jacques Benun
Pediatr. Rev. 2011;32;84-85
DOI: 10.1542/pir.32-2-84

Updated Information including high-resolution figures, can be found at:


& Services http://pedsinreview.aappublications.org/cgi/content/full/32/2/84
Subspecialty Collections This article, along with others on similar topics, appears in the
following collection(s):
Neurologic Disorders
http://pedsinreview.aappublications.org/cgi/collection/neurologi
c_disorders Ear, Nose and Throat Disorders
http://pedsinreview.aappublications.org/cgi/collection/ear_nose_
throat_disorders
Permissions & Licensing Information about reproducing this article in parts (figures,
tables) or in its entirety can be found online at:
http://pedsinreview.aappublications.org/misc/Permissions.shtml
Reprints Information about ordering reprints can be found online:
http://pedsinreview.aappublications.org/misc/reprints.shtml

Downloaded from http://pedsinreview.aappublications.org. Provided by Pakistan:AAP Sponsored on February 1, 2011


Issues in Chemotherapy
Elizabeth A. Van Dyne
Pediatr. Rev. 2011;32;86-87
DOI: 10.1542/pir.32-2-86

The online version of this article, along with updated information and services, is
located on the World Wide Web at:
http://pedsinreview.aappublications.org/cgi/content/full/32/2/86

Pediatrics in Review is the official journal of the American Academy of Pediatrics. A monthly
publication, it has been published continuously since 1979. Pediatrics in Review is owned,
published, and trademarked by the American Academy of Pediatrics, 141 Northwest Point
Boulevard, Elk Grove Village, Illinois, 60007. Copyright © 2011 by the American Academy of
Pediatrics. All rights reserved. Print ISSN: 0191-9601. Online ISSN: 1526-3347.

Downloaded from http://pedsinreview.aappublications.org. Provided by Pakistan:AAP Sponsored on February 1, 2011


in brief

In Brief
Issues in Chemotherapy
Elizabeth A. Van Dyne, MD to infection directs the scheduling of ing in encephalitis, hepatitis, pneumo-
Kaiser Permanente Medical Center vaccine administration and the urgent nia, and, in some cases, death. An
Los Angeles, CA treatment of the febrile child. extended period of vesicular eruptions
Live (attenuated) vaccines should may occur. Immunocompromised pa-
not be given to patients undergoing tients who have well-documented ex-
Author Disclosure immunosuppressive therapy, specifically posure to varicella and no history of
Drs Van Dyne and Serwint have chemotherapy and radiation treatment. varicella infection or serologic titers
disclosed no financial relationships Live vaccines include those manufac- should receive varicella-zoster im-
relevant to this In Brief. This tured to protect against Salmonella mune globulin (under the brand name
typhi; influenza (live intranasal vac- VariZIGTM). (Note: VariVIG is distributed
commentary does contain a
cine); measles, mumps, rubella (MMR); in the United States by FFF Enterprises,
discussion of an unapproved/
varicella; rotavirus; polio (oral vaccine); Temecula, CA, and can be requested at
investigative use of a commercial yellow fever; and smallpox. The recom- any time by calling 800-843-7477.)
product/device. mendation is to wait at least 3 months Varicella virus replicates rapidly, ceas-
after completion of the last immuno- ing replication within 24 hours of the
suppressive treatment before adminis- onset of rash, at which time treatment
Immunizations in Special Clinical Cir- tering live vaccines, and many oncolo- with immunoglobulins is less effec-
cumstances. American Academy of gists wait 6 to 12 months. Because the tive. It is crucial to treat the infection
Pediatrics. In: Pickering LK, Baker CJ, time to immune reconstitution de- as soon as possible, at the latest by
Kimberlin DW, Long SS, eds. Red pends on the type of chemotherapy 96 hours after exposure, and not base
Book: 2009 Report of the Committee given, the dosage, and the patient’s treatment on presentation of the rash.
on Infectious Diseases. 28th ed. Elk degree of responsiveness to treatment, Some experts recommend treatment
Grove Village, IL: American Academy the recommendations must be tailored with VariZIG whether or not there is a
of Pediatrics; 2009:72– 86
to the individual patient and his or her past history of varicella infection be-
Varicella-Zoster Infections. American
Academy of Pediatrics. In: Pickering
circumstances. Vaccination with in- cause immunologic responses are low
LK, Baker CJ, Kimberlin DW, Long SS, activated vaccines can be completed and serologic tests can be unreliable in
eds. Red Book: 2009 Report of the without increased risk of infectious immunocompromised children. VariZIG
Committee on Infectious Diseases. complications, but the timing is impor- is a new investigational drug and some-
28th ed. Elk Grove Village, IL: tant because the immune response may what difficult to obtain but preferable
American Academy of Pediatrics; be inadequate in immunocompromised to immune globulin intravenous because
2009:714 –727 patients. An adequate response is usu- it contains concentrated immunoglob-
Fever and Neutropenia in Pediatric ally achieved from 3 months to 1 year ulin specific to varicella. If VariZIG is
Patients with Cancer. Meckler G, following the discontinuation of the unavailable, one dose of immune glob-
Lindemulder S. Emerg Med Clin radiation or chemotherapy. Inactivated ulin intravenous can be administered
North Am. 2009;27:525–544
influenza vaccine is recommended be- instead. If neither is available or 96 hours
2002 Guideline for the Use of Anti-
microbial Agents in Neutropenic
fore each influenza season for patients has passed, chemoprophylaxis with
Patients with Cancer. Hughes WT, 6 months of age and older. If possible, acyclovir should be initiated specifically
Armstrong D, Bodey GP, et al. influenza vaccination should be post- at 7 to 10 days postexposure to atten-
Clin Infect Dis. 2002;34:730 –751 poned until no sooner than 3 to uate or prevent infection.
4 weeks after a chemotherapy course is Another major concern for the im-
Children who have cancer may be se- completed and when the granulocyte munocompromised child who has can-
verely immunocompromised by the and lymphocyte counts are greater than cer is fever in the setting of neutrope-
oncologic process and the immuno- 1,000 cells/␮L. nia. Patients must present emergently
suppression that results from chemo- Varicella infection can be severe in for rapid triage, evaluation, and prompt
therapy. Their increased susceptibility immunocompromised children, result- administration of antibiotics. Increased

86 Pediatrics in Review Vol.32 No.2 February 2011


Downloaded from http://pedsinreview.aappublications.org. Provided by Pakistan:AAP Sponsored on February 1, 2011
in brief

risk of morbidity and mortality due to perative for the febrile neutropenic pa- minimize disease transmission to the
infection correlates with the severity tient because infection can progress patient. Family members and household
and duration of neutropenia. Bone mar- rapidly. Frequent monitoring of vital contacts should not be given smallpox
row suppression, impaired immunologic signs and surveillance of urine output or oral polio vaccines because of poten-
responses, the presence of a vascular is essential because tachycardia and tial transmission to the patient but
access device, and the breakdown of decreased urine output may be the should receive MMR, varicella, and
mucocutaneous barriers from chemo- initial signs of compensated shock. rotavirus vaccines, which have a very
therapy all contribute to the risks for After drawing cultures, immediate anti- low risk of transmission to the patient.
the immunocompromised child. biotic coverage for gram-positive and If a household member develops a rash
A thorough physical examination -negative organisms (including Pseudo- following the varicella vaccine, the
must be performed on the child who monas) should be administered. Indi- child who has cancer should avoid
has neutropenia, assessing vital signs; viduals who appear toxic should receive contact with that person, but no other
evaluating potential sites of infection; dual gram-negative coverage. Consid- precautions are currently warranted.
and paying special attention to all areas eration must also be given to fungal, Inactivated influenza vaccine should be
of the skin, mucous membranes, gastro- viral, and other opportunistic infec- given to household contacts 6 months
intestinal tract (eg, mouth, rectum), tions and appropriate therapy tailored of age and older, and age-appropriate
and vascular access devices. A patient to the patient. Oral antibiotics are not inactivated vaccines are important to
who has neutropenia may not mount appropriate in pediatric patients who administer. The universal administra-
an inflammatory response, resulting in have fever and neutropenia. tion of varicella vaccine hopefully has
minimal erythema, exudate, or pain at The child undergoing treatment for decreased the risk exposure for patients
infected sites. A complete blood count cancer presents the additional concern who have cancer. For those who have
with differential count and blood cul- for life-threatening infections. Practi- neutropenia and fever, urgent evalua-
tures from all lumens of vascular ac- tioners must be aware of established tion and administration of antibiotics
cess devices are recommended. Patients guidelines and consultative resources is imperative. Specific antibiotic and
who have an absolute neutrophil count to provide the highest quality of care. antifungal choices must be guided by
less than 500 cells/mm3 or who appear individual risk factors and local micro-
ill are at greatest risk. Although a micro- Comment: This In Brief reviews im- biologic profiles and tailored to the
biologic diagnosis is made in 10% to portant concepts for all who provide individual patient. Hence, the coordi-
30% of pediatric patients who have medical care to children who have can- nated care of the cancer patient with
neutropenia, most do not have an ob- cer. Provision of care requires collabo- oncology and infectious disease con-
vious source of fever or clear infection. ration and consultation among the sultation is important to provide indi-
Further evaluation with chest radiogra- patient’s primary care clinician, emer- vidualized care and demonstrates the
phy, urinalysis, urine cultures, or addi- gency department staff, the patient’s importance of true medical home prin-
tional cultures is warranted, based on oncologist, and an infectious disease ciples.
the patient’s symptoms. specialist, along with the patient and
Aggressive intravenous hydration and family. The immunization of household Janet R. Serwint, MD
urgent antibiotic administration is im- contacts and siblings is imperative to Consulting Editor, In Brief

Pediatrics in Review Vol.32 No.2 February 2011 87


Downloaded from http://pedsinreview.aappublications.org. Provided by Pakistan:AAP Sponsored on February 1, 2011
Issues in Chemotherapy
Elizabeth A. Van Dyne
Pediatr. Rev. 2011;32;86-87
DOI: 10.1542/pir.32-2-86

Updated Information including high-resolution figures, can be found at:


& Services http://pedsinreview.aappublications.org/cgi/content/full/32/2/86
Subspecialty Collections This article, along with others on similar topics, appears in the
following collection(s):
Infectious Diseases
http://pedsinreview.aappublications.org/cgi/collection/infectious
_diseases Disorders of Blood/Neoplasms
http://pedsinreview.aappublications.org/cgi/collection/disorders_
of_blood_neoplasms
Permissions & Licensing Information about reproducing this article in parts (figures,
tables) or in its entirety can be found online at:
http://pedsinreview.aappublications.org/misc/Permissions.shtml
Reprints Information about ordering reprints can be found online:
http://pedsinreview.aappublications.org/misc/reprints.shtml

Downloaded from http://pedsinreview.aappublications.org. Provided by Pakistan:AAP Sponsored on February 1, 2011


Complementary, Holistic, and Integrative Medicine: Therapies for Learning
Disabilities
Elaine Z. Galicia-Connolly, Larissa Shamseer and Sunita Vohra
Pediatr. Rev. 2011;32;e18-e24
DOI: 10.1542/pir.32-2-e18

The online version of this article, along with updated information and services, is
located on the World Wide Web at:
http://pedsinreview.aappublications.org/cgi/content/full/32/2/e18

Pediatrics in Review is the official journal of the American Academy of Pediatrics. A monthly
publication, it has been published continuously since 1979. Pediatrics in Review is owned,
published, and trademarked by the American Academy of Pediatrics, 141 Northwest Point
Boulevard, Elk Grove Village, Illinois, 60007. Copyright © 2011 by the American Academy of
Pediatrics. All rights reserved. Print ISSN: 0191-9601. Online ISSN: 1526-3347.

Downloaded from http://pedsinreview.aappublications.org. Provided by Pakistan:AAP Sponsored on February 1, 2011


Article complementary medicine

Complementary, Holistic, and Integrative


Medicine: Therapies for Learning Disabilities
Elaine Z. Galicia- Background
Connolly, MD,* Larissa Between 5% and 10% of schoolchildren have learning disabilities (LD), and up to 80% of

Shamseer, MSc, Sunita such children have some form of dyslexia. (1) The diagnosis is based on significantly lower
academic achievement than the intelligence quotient (IQ) would suggest. (2) Although
Vohra, MD†
disorders of learning and attention frequently occur together, this review specifically
examines the effectiveness of complementary and alternative medicine (CAM) interven-
tions for children who have LD. The studies might include children who have comorbid
Author Disclosure
conditions such as attention-deficit/hyperactivity disorder (ADHD).
Dr Galicia-Connolly
Existing interventions for LD focus on remedial teaching in the early years and offering
and Ms Shamseer compensatory mechanisms and lifestyle adjustments at later ages. The most robust
have disclosed no predictive factors correlating with the success of LD interventions are attention and
financial relationships behavior, general verbal ability, prior level of component reading skills, and total IQ. (1) In
relevant to this a survey of 148 schoolchildren who had dyslexia, Bull found that 55.4% had used CAM for
dyslexia, most commonly nutritional supplements or special diets, homeopathy, and
article. Dr Vohra has
osteopathy or chiropractic manipulation. (3)
disclosed receiving
salary support from
Methods
the Alberta Heritage We conducted a search for articles related to learning disabilities and CAM therapies on
Foundation for MEDLINE, Embase, AMED, PsycInfo, the Cochrane Database of Systematic Reviews,
Medical Research and and Cochrane’s Central Register of Controlled Trials. Search terms were: (learning
Canadian Institutes of disorders OR learning disabilities) AND (complementary therapies), with terms varying by
Health Research. This database and no limits on language or date. Studies were included if they examined a
strictly pediatric population (0 to 18 years) of individuals who had LD and who experi-
commentary does
enced CAM as the intervention.
contain a discussion
of an unapproved/
Results
investigative use of a Eleven articles were relevant to this review (Table).
commercial Nutritional Supplements
product/device. POLYUNSATURATED FATTY ACIDS (PUFAS). The PUFAs eicosapentaenoic acids (EPA)
and docosahexaenoic acid (DHA) are believed to be critical for brain development
and function. (16) Kairaluoma and associates (4) conducted a double-blind, placebo-
controlled, randomized, controlled trial (RCT) evaluating the effects of single daily
supplementation with 500 mg EPA and 400 mg carnosine over a 3-month period on the
reading-related skills of 61 children (mean age, 10.6 years) who had dyslexia. They found
no significant differences in the two groups in measures of reading accuracy or speed,
spelling, decoding, fluency, arithmetic skills, reading-related language skills, and attention
or behavioral problems.
A double-blind RCT evaluated the effect of supplementation of highly unsaturated fatty
acids on ADHD symptoms of 41 children (mean age, 10.25 years; standard deviation
[SD]⫽0.74 years) who had both literacy problems and features of ADHD. (5) Although
the authors reported that the studied children were not formally diagnosed as having
ADHD, the Connors Parent rating scale for ADHD was employed as the primary endpoint

*Former Clinical Fellow, University of Alberta, Stollery Children’s Hospital CARE Program, Edmonton, Alberta, Canada.

Complementary and Alternative Medicine Research and Education (CARE) Program, Department of Pediatrics, Faculty of
Medicine, University of Alberta on behalf of the American Academy of Pediatrics Section on Complementary and Integrative
Medicine.
NOTE: The agents discussed in this series are designated as dietary supplements rather than drugs. Although dietary
supplements are regulated by the United States Food and Drug Administration (FDA), their manufacturers may make claims
with little evidence and need not prove safety prior to marketing. The burden is on the FDA to monitor safety after the
product is on the market. Readers are referred to the 1994 Dietary Supplement Health and Education Act (www.cfsan.fda.gov/
⬃dms/dietsupp.html).

e18 Pediatrics in Review Vol.32 No.2 February 2011


Downloaded from http://pedsinreview.aappublications.org. Provided by Pakistan:AAP Sponsored on February 1, 2011
Table 1.
Literature Review of Complementary and Alternative Medicine Therapies for
Learning Disabilities
Critical Appraisal
Author Design Population Intervention Efficacy and Safety of Study
I. Nutritional Supplements
A. Polyunsaturated Fatty Acids (PUFAs)
Kairaluoma, Double-blind, nⴝ61 children with EPA 500 mg/day ⴙ • No differences in reading, • Well-designed study
2009 (4) placebo- observed reading carnosine 400 mg/day spelling, or reading-related • Sample size planned to
controlled difficulties; mean age, versus skills between treatment and detect treatment effect
RCT 10.6 years Placeboⴝtriglycerides ⴙ control groups with >80% power at
PUFA, nⴝ30 cellulose administered over • 2 children in placebo group Pⴝ0.05 level
Placebo, nⴝ31 3 months reported abdominal pain and • Dosing and duration of
difficulty swallowing pills observation comparable to
previous studies
Richardson, Double-blind, nⴝ41 12 weeks of daily • Significant symptom • High dropout rate (9/
2002 (5) placebo- 8- to 12-year-old supplementation of HUFA reduction only in Conners’ 41ⴝ22%)
controlled children diagnosed (EPA 186 mg, DHA ADHD index (Pⴝ0.03); no • Mild adverse effects
RCT with dyslexia; mean 480 mg, GLA 96 mg, significant improvement in reported: digestive upset
age, 10.25ⴞ0.74 years vitamin E 60 IU, cis- cognitive problems (1 each in HUFA group
HUFA, nⴝ22 linoleic acid 864 mg, (Pⴝ0.07) or other ADHD and placebo group),
Placebo, nⴝ19 AA 42 mg, and thyme subscales after 3 months of swallowing problems (1 in
oil 8 mg) HUFA supplementation HUFA), noncompliance (3
versus in each group), incomplete
Placebo (olive oil) end evaluation for 3 (2
HUFA, 1 placebo)
Lindmark, Open-label pilot 19 children with dyslexia; 5 months of • 76% of children showed • No placebo or control
2007 (6) study mean age, 12 years supplementation with 8 significant improvement in group
capsules daily containing word-chain tests (P<0.04), • No randomization
480 mg DHA, 108 mg with 60% improvement in • Recall/reporting bias with
EPA, 96 mg GLA, and reading speed (P<0.01) and self- and parent
35 mg AA 23% improvement in assessments
motoric-perceptual speed • 21% dropout rate
(P<0.05) • Pharmaceutical-sponsored
study
Stordy, 2000 Open-label pilot nⴝ15 children with 480 mg DHA, 35 mg AA, • Significant improvement in • No placebo or control
(7) study dyspraxia, 5 to 12 96 mg linoleic acid, manual dexterity (P<0.007), group
complementary medicine

years old 80 mg vitamin E, and ball skills (P<0.02), static/ • No randomization


24 mg thyme oil daily dynamic balance (P<0.03) • No blinding
administered over 4 • Significant improvement in • Reporting bias with parent
months total impairment score assessments
(P<0.0001) and checklist • 13% dropout (nⴝ2)

Downloaded from http://pedsinreview.aappublications.org. Provided by Pakistan:AAP Sponsored on February 1, 2011


score
(Continued)
learning disabilities

Pediatrics in Review Vol.32 No.2 February 2011 e19


Table 1.
Literature Review of Complementary and Alternative Medicine Therapies for
Learning Disabilities—continued
Author Design Population Intervention Efficacy and Safety Critical Appraisal of Study
B. Gingko Biloba Extract
Donfrancesco, Open-label pilot nⴝ15 children, 5 to 16 Daily 80-mg morning dose • Significantly improved scores • No control group
complementary medicine

2007 (8) study years old (mean, 10.7 of standardized plant in reading accuracy (list of • No blinding or
years) with DSM-IV- extract of gingko biloba words P<0.01, list of randomization
diagnosed dyslexia administered for 34.4 days nonwords P<0.02, and • Small sample size
reading text P<0.05), but no • Short duration of
change in reading speed treatment

e20 Pediatrics in Review Vol.32 No.2 February 2011


• Brief period of headache
reported by parents in 2
learning disabilities

children
II. Neurofeedback
Fernandez, Double-blind, nⴝ10 children with LD NF 30-minute sessions, • Total IQ (Pⴝ0.02), verbal • No randomization
2003 (9) placebo- and abnormally high biweekly over 10 to (Pⴝ0.03) and performance • Small sample size
controlled EEG theta/alpha ratio 12 weeks (20 sessions (Pⴝ0.04) WISC scores • 16% dropout rate (nⴝ2)
trial for age total) increased significantly only in • No mention of adverse
nⴝ5 experimental the experimental group events
nⴝ5 control despite no significant changes
in EEG recordings before and
after treatment
• No changes noted in control
group
Becerra, 2006 Prospective nⴝ9 children with LD No new interventions since • Significant increase in global • Small sample size
(10) interventional nⴝ5 NF (mean age, 11.2 previous investigation IQ (Pⴝ0.04) and performance • No mention of adverse
study (2-year years) scores (P<0.05) but decrease events
follow-up versus in verbal scores in NF group;
period) nⴝ5 placebo (mean age, remission of LD symptoms in
12.1 years) 80% of children (4 of 5) 2
years after NF treatment
• No significant differences in
WISC-R or ADHD scores from
TOVA for control group
Linden, 1996 Single-blind nⴝ18 treatment-naive Forty 45-minute twice- • Significant reduction of • Sample size inadequate to
(11) RCT children with LD or weekly sessions of EEG parent-reported inattentive detect changes in IQ,
ADD/ADH, 5 to 15 biofeedback over 6 months behavior in treatment group inattention, and
years old (Pⴝ0.04) oppositional behavior
nⴝ9 NF • Significant improvement in • Authors attributed
versus IQ scores posttreatment treatment effects to
nⴝ9 waiting list as (Pⴝ0.02) enhanced attention

Downloaded from http://pedsinreview.aappublications.org. Provided by Pakistan:AAP Sponsored on February 1, 2011


control • No significant differences in • No mention of adverse
parent-reported oppositional events
behavior posttreatment
(Continued)
Table 1.
Literature Review of Complementary and Alternative Medicine Therapies for
Learning Disabilities—continued
Critical Appraisal
Author Design Population Intervention Efficacy and Safety of Study
III. Chiropractic
Bull, 2007 RCT nⴝ70 children 6 to 13 Eight sunflower therapy • No statistical differences in • Short observation period
(12) years old with dyslexia (applied kinesiology, performance of children in may not have been
and no other medical, osteopathic manipulation, any of cognitive or literacy sufficient to detect the
developmental, or massage, herbal remedies, tests between groups treatment effect (13)
behavioral difficulties and neurolinguistic • Statistically significant • Unclear method of
(mean, 9.9ⴞ1.8 years) programming) sessions improvements in academic randomization and
lasting 40 minutes over (P<0.001) and reading self- allocation
15 weeks esteem (P<0.001) for • No mention of adverse
treatment group effects
Cuthbert, Prospective nⴝ157 children 6 to Individualized chiropractic • Improvement in 8 • Heterogeneous study
2009 (14) interventional 13 years old with multimodal (applied psychometric tests and 20 population
study developmental delay kinesiology) technique, areas of cognitive function • No control group
syndromes over a ranging from 2 to 50 • No adverse events occurred • No randomization
14-year period sessions
IV. Music
Register, Prospective Group 1: nⴝ8 children 12-lesson music/reading • Significant improvement in • Small study sample
2007 (15) interventional with reading disability program for 4 weeks word decoding (Pⴝ0.04), • No randomization
study Group 2: nⴝ17 students word knowledge (Pⴝ0.01), • Suggested extending
in an intact grade 2 and reading comprehension curriculum to 6 weeks
class assigned as (Pⴝ0.01) in children with • Used both parametric and
treatment class reading disability nonparametric tests
Group 3: nⴝ16 students • Greater gains on all subtests without describing data
in an intact grade 2 in treatment group, but • No mention of adverse
class assigned as significant improvement only events
control class in word knowledge in
treatment group compared
with control
complementary medicine

AA⫽amino acids, ADD⫽attention-deficit disorder, ADHD⫽attention-deficit/hyperactivity disorder, DHA⫽docosahexaenoic acid, DSM⫽Diagnostic and Statistical Manual of Mental Disorders,
EEG⫽electroencephalography, EPA⫽eicosapentaenoic acid, GLA⫽gamma linolenic acid, HUFA⫽highly unsaturated fatty acids, IQ⫽intelligence quotient, LD⫽learning disability,
NF⫽neurofeedback, PUFA⫽polyunsaturated fatty acid, RCT⫽randomized, controlled trial, TOVA⫽tests of variables of attention, WISC⫽Weschler Intelligence Scale for Children

Downloaded from http://pedsinreview.aappublications.org. Provided by Pakistan:AAP Sponsored on February 1, 2011


learning disabilities

Pediatrics in Review Vol.32 No.2 February 2011 e21


complementary medicine learning disabilities

and measured at 12 weeks. PUFA supplementation ap- biweekly neurofeedback sessions during which partici-
peared to alleviate ADHD-related symptoms in dyslexia. pants were “trained” to inhibit their brain’s theta activity
An open-label pilot study of 24 children who had over 10 to 12 weeks or to placebo sessions that were
dyslexia and whose mean age was 12 years evaluated a comprised of similar sessions but with random and non-
patented single daily supplement containing tuna fish oil contingent feedback. Tests of variables of attention
(480 mg DHA, 108 mg EPA) and primrose oil over a (TOVA) and EEG were obtained after 20 sessions and
5-month period. (6) The authors reported that 76% of the Weschler Intelligence Scale for Children (WISC) was
children showed significant improvement in word-chain readministered after 6 months. The authors reported a
tests (P⬍0.04), with 60% improvement in reading speed significant improvement in WISC performance, with no
(P⬍0.01) and 23% improvement in motoric-perceptual corresponding significant changes in the EEG readings
speed (P⬍0.05) at the end of the study. Both parents and of the treatment group.
children reported improvement in speed of reading and Nine of the 10 children from the previously men-
schoolwork. Although this treatment appears promising, tioned cohort were subsequently followed for 2 years to
it would benefit from more rigorous study with a placebo examine the long-term effects of neurofeedback. (10)
control group to reduce potential bias. The authors reported a significant increase in global IQ
An open-label pilot study examined administration of and performance scores and a remission of LD symptoms
the same supplement to 15 children who had dyspraxia in four of the five children who had undergone neuro-
over a 4-month period. (7) Motor skills using the Move- feedback. In contrast, all of the children in the control
ment Assessment Battery for Children were measured as group continued to exhibit LD. Further studies with
the primary outcome at 4 months. The authors reported larger sample sizes would be helpful.
statistically significant differences in all the children’s One RCT evaluated the efficacy of 40 45-minute
movement skills. Unfortunately, in the absence of a neurofeedback sessions given over 6 months to enhance
control group, the potential for biased estimates of treat- beta activity and suppress theta activity in 18 treatment-
ment effect is high. naı̈ve children ages 5 to 15 years who had received a
diagnosis of LD or ADD/ADHD. (11) Fifty percent of
GINGKO BILOBA. Gingko biloba has traditionally been the children on a wait list acted as a control group. There
used in Chinese medicine as a memory “booster.” An was a significant improvement in IQ scores (P⫽0.02) and
open-label trial examined the effect of a single morning parent-reported inattention (P⫽0.04) in the children
80-mg dose of gingko biloba over a 34-day period in 15 who underwent neurofeedback, but there were no dif-
children (mean age, 10.8 years, SD⫽2 years) who had a ferences in parent-reported oppositional behavior be-
Diagnostic and Statistical Manual of Mental Disorders, tween groups. The authors attributed the improvement
4th edition, text revision (DSM-IV-TR) diagnosis of in the children’s IQ scores to enhanced attention.
“dyslexia not taking psychotropic medications or under-
going rehabilitative treatment.” (8) The authors found a Chiropractic Therapies
significant improvement in reading accuracy but not An RCT evaluated an intervention called “sunflower
reading speed. A double-blind, placebo-controlled RCT therapy” (described as a integrative course of treatment
would be helpful to confirm these preliminary results. that combines applied kinesiology, osteopathic manipu-
lation, massage, herbal remedies, homeopathy, and neu-
Neurofeedback rolinguistic programming) administered over 15 weeks
Children who have LD have higher values of theta activ- by using cognitive, literacy, and self-esteem tests in 70
ity on encephalography (EEG) than unaffected children, children ages 6 to 13 years who had dyslexia. (12)
(17) and neurofeedback has been described as an “oper- Participants were randomized to either eight 40-minute
ant conditioning procedure” that allows an individual to sessions of sunflower therapy or a control group and
learn to modify the electrical activity of his or her own were retested within a mean of 15.7 weeks (SD 2.3).
brain. (18) Quantitative EEG evaluation attests to the The author found no differences between groups in
heterogeneity of children who have LDs, and studies on cognitive or literacy tests but noted statistically signifi-
the efficacy of neurofeedback remain sparse, needing cant improvements in academic self-esteem (P⬍0.001)
larger sample sizes to replicate their findings. (19) and reading self-esteem (P⬍0.001) scores within the
A double-blind RCT evaluated neurofeedback in 10 treatment group. We concur with the author that a
children ages 7 to 11 years who had a diagnosis of LD. control group with sham treatment to exclude placebo
(9) Participants were assigned to either 40 45-minute effects would be helpful. This study has been criticized

e22 Pediatrics in Review Vol.32 No.2 February 2011


Downloaded from http://pedsinreview.aappublications.org. Provided by Pakistan:AAP Sponsored on February 1, 2011
complementary medicine learning disabilities

for too short of a treatment period to detect treatment PUFAs include nausea, nosebleeds, and mild fever. (20)
effects. (13) (21)(22) Other potential adverse events are gastrointes-
Applied kinesiology is a chiropractic diagnostic tinal upset, altered immune response, reduced blood
method that evaluates muscle function using manual pressure, and increased risk of bleeding at high doses.
muscle testing, based on the premise of a link between (23)(24) Potential drug interactions may occur with
motor impairments and developmental delay. A case anticoagulants and antiplatelet drugs. (25)(26)
series of 157 children ages 6 to 13 years who had devel- The use of gingko biloba generally has been associ-
opmental delay syndromes, including children who had ated with mild and transient effects, the most common
ADHD, involved psychometric testing that monitored being headache. Reported serious adverse effects include
cognitive function, including patient- or parent-reported seizures, spontaneous bleeding, and intracerebral hem-
improvements in school performance, social interaction, orrhage. (27)(28)(29)
and sporting activities before and after chiropractic treat- None of the studies (9)(10)(11) reported adverse
ment. (14) The authors reported enhanced cognitive effects during neurofeedback. However, with 20 or more
performance in this group of children based on 6.6% to sessions, reported adverse effects have included head-
33.2% improvement between scores from psychometric aches, dizziness, increased irritability, moodiness, loss of
tests that were administered within 5 days to 18 months attention, and hyperactivity in children who were being
after 2 to 50 chiropractic sessions. These promising concomitantly treated with stimulants. (30)
results would benefit from further rigorous study, ideally A systematic review on the safety of pediatric spinal
including a less heterogeneous sample, a randomized manipulation identified 14 serious adverse events that
control group, and blinded outcome assessment. resulted in hospitalization, permanent disability, or
death. Others involved delayed diagnosis or inappropri-
Music Therapy ate provision of chiropractic care. (31) The estimated risk
The effect of music in enhancing reading skills of second- of a serious adverse event is 0.13% in adults, (32) but an
grade students and students who had reading disabilities accurate estimate from high-quality pediatric studies is
was examined in a prospective interventional study. (15) not available. (33)
A class of eight children who had reading disability
underwent a specially designed 4-week curriculum using
music to target reading comprehension and vocabulary Summary
skills. The authors used two intact second-grade classes
for comparison. One participated in their normal reading • Preliminary evidence suggests possible positive
program as a control class and the other underwent both effects of neurofeedback, PUFA, gingko biloba,
chiropractic, and music in the treatment of LD, but
the music-augmented reading program and their normal
more rigorous data are needed.
reading program as a treatment class.
The authors reported significant improvements in
word decoding (P⫽0.04), word knowledge (P⫽0.01), ACKNOWLEDGMENTS. The authors would like to ac-
and reading comprehension (P⫽0.01) in the children knowledge the invaluable assistance of Connie Winther
who had LD and attributed these findings to increased and Soleil Surette in the literature search for this manu-
attention and more active participation because of the script.
smaller class size. For second graders, the treatment
group had significantly greater gains in word knowledge
compared with the control group. The music/reading References
curriculum appeared to be an effective supplement to the 1. Lagae L. Learning disabilities: definitions, epidemiology, diag-
nosis, and intervention strategies. Pediatr Clin North Am. 2008;55:
current reading program. It would be more compelling if
1259 –1268
children who had a reading disability had been included 2. Ewen JB, Shapiro BK. Disorders of attention or learning in
in the control group for better assessment of the effect of neurodevelopmental disorders. Semin Pediatr Neurol. 2005;12:
music therapy in this population. 229 –241
3. Bull L. Survey of complementary and alternative therapies used
by children with specific learning difficulties (dyslexia). Int J Lang
Adverse Events
Commun Disord. 2009;44:224 –235
Of the 11 studies included in this review, only four 4. Kairaluoma L, Narhi V, Ahonen T, Westerholm J, Aro M. Do
explicitly reported on the presence or absence of adverse fatty acids help in overcoming reading difficulties? A double-blind,
events during the study. Adverse events associated with placebo-controlled study of the effects of eicosapentaenoic acid and

Pediatrics in Review Vol.32 No.2 February 2011 e23


Downloaded from http://pedsinreview.aappublications.org. Provided by Pakistan:AAP Sponsored on February 1, 2011
complementary medicine learning disabilities

carnosine supplementation on children with dyslexia. Child Care 19. Cantor DS, Chabot R. QEEG studies in the assessment and
Health Dev. 2009;35:112–119 treatment of childhood disorders. Clin EEG Neurosci. 2009;40:
5. Richardson AJ, Puri BK. A randomized double-blind, placebo- 113–121
controlled study of the effects of supplementation with highly 20. Amminger GP, Berger GE, Schafer MR, Klier C, Friedrich
unsaturated fatty acids on ADHD-related symptoms in children MH, Feucht M. Omega-3 fatty acids supplementation in children
with specific learning difficulties. Prog Neuropsychopharmacol Biol with autism: a double-blind randomized, placebo-controlled pilot
Psychiatry. 2002;26:233–239 study. Biol Psychiatry. 2007;61:551–553
6. Lindmark L, Clough P. A 5-month open study with long-chain 21. Emsley R, Niehaus DJ, Oosthuizen PP, et al. Safety of the
polyunsaturated fatty acids in dyslexia. J Med Food. 2007;10: omega-3 fatty acid, eicosapentaenoic acid (EPA) in psychiatric
662– 666 patients: results from a randomized, placebo-controlled trial. Psy-
7. Stordy BJ. Dark adaptation, motor skills, docosahexaenoic acid, chiatry Res. 2008;161:284 –291
and dyslexia. Am J Clin Nutr. 2000;71(1 suppl):323S–326S 22. Sinn N, Bryan J, Wilson C. Cognitive effects of polyunsatu-
8. Donfrancesco R, Ferrante L. Ginkgo biloba in dyslexia: a pilot rated fatty acids in children with attention deficit hyperactivity
study. Phytomedicine. 2007;14:367–370 disorder symptoms: a randomised controlled trial. Prostaglandins
9. Fernandez T, Herrera W, Harmony T, et al. EEG and behavioral Leukotrienes Essent Fatty Acids. 2008;78:311–326
changes following neurofeedback treatment in learning disabled 23. Goodnight SH Jr, Harris WS, Connor WE. The effects of
children. Clin Electroencephalogr. 2003;34:145–152 dietary omega 3 fatty acids on platelet composition and function in
10. Becerra J, Fernandez T, Harmony T, et al. Follow-up study of man: a prospective, controlled study. Blood. 1981;58:880 – 885
learning-disabled children treated with neurofeedback or placebo. 24. Wensing AG, Mensink RP, Hornstra G. Effects of dietary n-3
Clin EEG Neurosci. 2006;37:198 –203 polyunsaturated fatty acids from plant and marine origin on platelet
11. Linden M, Habib T, Radojevic V. A controlled study of the aggregation in healthy elderly subjects. Br J Nutr. 1999;82:
effects of EEG biofeedback on cognition and behavior of children 183–191
with attention deficit disorder and learning disabilities. Biofeedback 25. Buckley MS, Goff AD, Knapp WE. Fish oil interaction with
Self. 1996;21:35– 49 warfarin. Ann Pharmacother. 2004;38:50 –53
12. Bull L. Sunflower therapy for children with specific learning 26. Woodman RJ, Mori TA, Burke V, et al. Effects of purified
difficulties (dyslexia): a randomised, controlled trial. Complement eicosapentaenoic acid and docosahexaenoic acid on platelet, fi-
Ther Clin Pract. 2007;13:15–24 brinolytic and vascular function in hypertensive type 2 diabetic
13. Mathews MO, Thomas E, Yeung A. Rebuttal paper to “Sun- patients. Atherosclerosis. 2003;166:85–93
flower therapy for children with specific learning difficulties (dys- 27. Haemorrhage due to ginkgo biloba? Prescrire Int. 2008;
lexia): a randomised, controlled trial.” Complement Ther Clin Pract. 17(93):19
2009;15:44 – 46 28. Hasegawa S, Oda Y, Ichiyama T, Hori Y, Furukawa S. Ginkgo
14. Cuthbert SC, Barras M. Developmental delay syndromes: psy- nut intoxication in a 2-year-old male. Pediatr Neurol. 2006;35:
chometric testing before and after chiropractic treatment of 157 275–276
children. J Manipulative Physiol Ther. 2009;32:660 – 669 29. Yagmur E, Piatkowski A, Groger A, Pallua N, Gressner AM,
15. Register D, Darrow AA, Standley J, Swedberg O. The use of Kiefer P. Bleeding complication under gingko biloba medication.
music to enhance reading skills of second grade students and Am J Hematol. 2005;79:343–344
students with reading disabilities. J Music Ther. 2007;44:23–37 30. Monastra VJ, Lynn S, Linden M, Lubar JF, Gruzelier J,
16. Brulotte J, Bukutu C, Vohra S. Complementary, holistic, and LaVaque TJ. Electroencephalographic biofeedback in the treat-
integrative medicine: fish oils and neurodevelopmental disorders. ment of attention-deficit/hyperactivity disorder. Appl Psychophysiol
Pediatr Rev. 2009;30:e29 – e33 Biofeedback. 2005;30:95–114
17. Chabot RJ, di Michele F, Prichep L, John ER. The clinical role 31. Vohra S, Johnston BC, Cramer K, Humphreys K. Adverse
of computerized EEG in the evaluation and treatment of learning events associated with pediatric spinal manipulation: a systematic
and attention disorders in children and adolescents. J Neuropsychi- review. Pediatrics. 2007;119:e275– e283
atr Clin Neurosci. 2001;13:171–186 32. Carnes D, Mullinger B, Underwood M. Defining adverse
18. Monastra VJ. Electroencephalographic biofeedback (neuro- events in manual therapies: a modified Delphi consensus study.
therapy) as a treatment for attention deficit hyperactivity disorder: Man Ther. 2010;15:2– 6
rationale and empirical foundation. Child Adolesc Psychiatr Clin 33. Humphreys BK. Possible adverse events in children treated by
North Am. 2005;14:55– 82 manual therapy: a review. Chiropr Osteopat. 2010;18:12

e24 Pediatrics in Review Vol.32 No.2 February 2011


Downloaded from http://pedsinreview.aappublications.org. Provided by Pakistan:AAP Sponsored on February 1, 2011
Complementary, Holistic, and Integrative Medicine: Therapies for Learning
Disabilities
Elaine Z. Galicia-Connolly, Larissa Shamseer and Sunita Vohra
Pediatr. Rev. 2011;32;e18-e24
DOI: 10.1542/pir.32-2-e18

Updated Information including high-resolution figures, can be found at:


& Services http://pedsinreview.aappublications.org/cgi/content/full/32/2/e18

Subspecialty Collections This article, along with others on similar topics, appears in the
following collection(s):
Disorders of Cognition, Language, Learning, and Attention
http://pedsinreview.aappublications.org/cgi/collection/cognition
_language_learning_attention_disorders Complementary,
Holistic, and Integrative Medicine
http://pedsinreview.aappublications.org/cgi/collection/compleme
ntary_holistic_integrative
Permissions & Licensing Information about reproducing this article in parts (figures,
tables) or in its entirety can be found online at:
http://pedsinreview.aappublications.org/misc/Permissions.shtml
Reprints Information about ordering reprints can be found online:
http://pedsinreview.aappublications.org/misc/reprints.shtml

Downloaded from http://pedsinreview.aappublications.org. Provided by Pakistan:AAP Sponsored on February 1, 2011


Global Child Health
Donna Denno
Pediatr. Rev. 2011;32;e25-e38
DOI: 10.1542/pir.32-2-e25

The online version of this article, along with updated information and services, is
located on the World Wide Web at:
http://pedsinreview.aappublications.org/cgi/content/full/32/2/e25

Pediatrics in Review is the official journal of the American Academy of Pediatrics. A monthly
publication, it has been published continuously since 1979. Pediatrics in Review is owned,
published, and trademarked by the American Academy of Pediatrics, 141 Northwest Point
Boulevard, Elk Grove Village, Illinois, 60007. Copyright © 2011 by the American Academy of
Pediatrics. All rights reserved. Print ISSN: 0191-9601. Online ISSN: 1526-3347.

Downloaded from http://pedsinreview.aappublications.org. Provided by Pakistan:AAP Sponsored on February 1, 2011


Article preventive pediatrics

Global Child Health


Donna Denno, MD, MPH*
Objectives After completing this article, readers should be able to:

1. State the current annual number of child deaths globally and percentage of these
Author Disclosure deaths occurring in developing countries.
Dr Denno has 2. List the seven leading causes of childhood deaths worldwide and the interventions
disclosed no financial recommended by international guidelines to prevent these deaths.
relationships relevant 3. Be aware of the percentage of childhood deaths that could be prevented through
to this article. This widespread implementation of these interventions.
commentary does not 4. Describe Millennium Development Goal 4 and progress toward its achievement.
contain a discussion 5. Discuss the socio-political-economic factors and policies that affect child health.
of an unapproved/ 6. Identify specific ways in which child health professionals in the United States can
investigative use of a affect child health globally.
commercial
product/device.
Introduction
Each year, millions of children die, the vast majority in poor countries. Tragically, most
of these deaths are preventable with technologies that are currently available and recom-
mended for universal implementation. Progress is being made: 8.1 million children
younger than the age of 5 years died in 2009, down from 12.4 million in 1990.
Unfortunately, rates of reduction in child mortality are much less than what can be
accomplished realistically with available technology. Furthermore, progress lags far behind
what was expected when 189 nations committed to reduce poverty and improve health and
development via the Millennium Development Goals (MDGs). The MDGs consist of
eight specific goals, including MDG4, which explicitly relates to child health and aims to
reduce child mortality by two thirds between 1990 and 2015. The world is not on track to
meet MDG4, although several developing countries are demonstrating that substantial
improvements in child health can be made by focusing on
making key lifesaving interventions available to children who
need them the most.
Abbreviations
This article explores the scope and causes of childhood
ACT: artemisinin combination therapy mortality, interventions to combat these killers, trends in
AIDS: acquired immune deficiency syndrome child health, potential reasons for these trends, and oppor-
CHW: community health worker tunities for progress in improving global child health.
Hib: Haemophilus influenzae type b
HIV: human immunodeficiency virus Scope of the Problem
IMCI: integrated management of childhood illness Health is more than just survival, but as a starting point, it is
IPT: intermittent presumptive treatment hard to avoid the fact that more than 22,000 children
ITN: insecticide-treated net younger than age 5 years are estimated to die each day.
MDG: Millennium Development Goal Ninety-nine percent of these deaths occur in developing
ORS: oral rehydration salts countries, with 92% in Africa and Asia. A child born in
ORT: oral rehydration therapy sub-Saharan Africa faces a 1 in 8 chance of dying before his or
PCP: Pneumocystis jiroveci pneumonia her fifth birthday compared with 1 in 130 for a child born in
U5MR: under-five mortality rate the United States and an average of 1 in 167 for developed
UNICEF: United Nations Children’s Fund countries.
VA: vitamin A “Child,” as used in this article and as typically defined in
WHO: World Health Organization the global health field, includes persons younger than 5 years
of age because of their particular biologic and social vulner-

*Department of Pediatrics, Department of Global Health, University of Washington, Seattle, WA.

Pediatrics in Review Vol.32 No.2 February 2011 e25


Downloaded from http://pedsinreview.aappublications.org. Provided by Pakistan:AAP Sponsored on February 1, 2011
preventive pediatrics global child health

and help to set priorities within


programs and policies to improve
child survival.

Major Causes of Child


Mortality
Eighty-four percent of global child
deaths are due to seven major caus-
es: neonatal problems, pneumonia,
diarrhea, malaria, measles, human
immunodeficiency virus (HIV)/
acquired immune deficiency syn-
drome (AIDS), and injuries (Fig. 1).
Undernutrition is an enormous
risk factor for childhood mortality
and morbidity. As defined in recent
research that estimates it contrib-
Figure 1. Causes of death among children younger than age 5 years. Reproduced from utes to 35% of all child deaths, un-
UNICEF ChildInfo website: http://www.childinfo.org/mortality.html and based on Black R,
dernutrition includes: 1) lack of
Cousens S, Johnson H, et al. Global, regional, and national causes of child mortality in
sufficient macronutrient (eg, pro-
2008: a systematic analysis. Lancet. 2010;375:1969 –1987.
tein) and caloric intake; 2) micro-
nutrient deficiencies, especially vi-
ability. Age-specific mortality rates decline appreciably tamin A (VA), zinc, iron, and iodine; and 3) lack of early
beyond 5 years. Under 5 mortality rates (U5MRs) are initiation of breastfeeding and exclusive breastfeeding in
commonly used as indicators to assess the health of a the first 6 months after birth. Undernutrition increases
general population. If conditions favor the health and susceptibility to infectious diseases and is associated with
welfare of this vulnerable group, the situation generally long-term cognitive impairment and poor school perfor-
can be considered favorable for the overall society. mance as well as with poor health consequences in adult-
Statistics from developing countries can carry a wide hood and into the next generation (eg, low-birthweight
margin of uncertainty due to lack of resources to support offspring among affected women).
surveillance and registry systems and limited availability It is worth noting that undernutrition and the com-
of diagnostics for precise determination of causes of mon direct causes of child deaths are not exotic tropical
illness and death. Furthermore, many important health- diseases, but rather diseases of poverty. Many are the
related events take place outside of health facilities, lim- same diseases that killed children in the United States and
iting the acquisition of information in centralized data other developed countries 100 years ago: neonatal prob-
sources. For example, parents often seek care for child- lems, diarrhea, pneumonia, and measles. These scourges
hood illnesses from pharmacy shops or street vendors. were largely controlled even before the advent of antimi-
Most births and neonatal deaths occur at home and often crobials and vaccinations by social changes addressing
are unrecorded. crowding, sanitation, nutrition, and basic living condi-
However, there are additional sources of helpful data, tions. Medical breakthroughs such as antibiotics, immu-
including nationally representative surveys, such as De- nizations, and insecticides (to prevent malaria, a vector-
mographic and Health Surveys (largely funded by borne disease that was endemic in parts of the United
United States Agency for International Development) States) further accelerated gains in controlling disease
and the United Nations Children’s Fund (UNICEF) and preventing childhood mortality.
Multiple Indicator Cluster Surveys. Expert panels such as An overview of causes of childhood mortality would
the Inter-agency Group for Child Mortality Estimation not be complete without mention of less proximate, but
and the Child Health Epidemiology Reference Group perhaps even more important underlying causes: the
have been working on improving estimation methodol- social determinants of health. Children living in absolute
ogies and standardizing reporting of estimates. Despite poverty have limited access to the basics necessary to
less-than-desirable margins of certainty, these collective support survival (eg, essential medicines and health care,
sources of information allow for examination of trends water, sanitation, adequate nutrition, education). Nearly

e26 Pediatrics in Review Vol.32 No.2 February 2011


Downloaded from http://pedsinreview.aappublications.org. Provided by Pakistan:AAP Sponsored on February 1, 2011
preventive pediatrics global child health

Interventions to Reduce Child Mortality


The following section reviews current recommendations
for preventing deaths from leading childhood killers. The
term intervention is used to describe “a biologic agent or
action intended to reduce morbidity or mortality.” (1)

Neonatal Causes
More than one third of children who die do so during the
first month after birth, accounting for more than 3.2 mil-
lion deaths annually. Three major causes comprise 80%
of deaths: prematurity and low birthweight, neonatal
infections, and asphyxia. Most deaths occur at home and
Figure 2. Under 5 mortality rate (per 1,000 live births) by in the first postnatal week. A large proportion of neonatal
wealth group. Reproduced from Closing the Gap in a Gener- deaths can be prevented if babies and their mothers
ation: Health Equity Through Action on the Social Determi- receive known, effective interventions that span the
nants of Health. Final Report of the Commission on Social ante-, intra-, and postpartum periods. Examples include
Determinants of Health. Geneva, Switzerland: World Health antenatal screening and care, delivery by skilled birth
Organization; 2008. attendants, promotion of early and exclusive breastfeed-
ing, hygienic skin and umbilical cord care, keeping babies
warm and dry (eg, skin-to-skin contact), family recogni-
1.4 billion people live on less than $1.25/day, the in- tion of signs of illness requiring prompt medical atten-
ternational poverty line. Another 1.2 billion live on tion, and early assessment of newborns by trained health
$1.25 to $2.00/day, which is closer to a practical poverty workers. Unfortunately, coverage rates in developing
line, especially in middle-income countries. Altogether, countries for recommended neonatal interventions are
approximately 40% of the world’s population lives on less poor (Fig. 4).
than $2.00/day, an extreme hardship that reduces Widespread implementation of a package of inexpen-
chances for survival and optimal health. sive, simple, and effective interventions could cut neona-
Relative poverty is also an important determinant of tal deaths in half and bring global neonatal death rates to
health, as borne out by research in both developed and levels comparable with those of industrialized countries
developing countries. Large gaps between rich and poor that were recorded just before the widespread availability
within a society are associated with worse health out- of neonatal intensive care.
comes among the poorer compared with the wealthier
strata within the society (Fig. 2). This phenomenon of
Acute Respiratory Infections
Pneumonia is the leading cause of death after the neona-
poor health among those living in material deprivation
tal period, killing more than 1 million children annually.
compared with those around them who are better off has
Although data on the microbiologic causes of pneumo-
been documented from New York to Glasgow to
nia are limited, pneumococcus and Haemophilus influen-
Uganda. Unfortunately, income gaps between rich and
zae type b (Hib) are the leading causes of childhood
poor have increased dramatically over the past few de- pneumonia deaths globally. This predominance is not
cades, along with increases in health disparities. For unlike the causes of pneumonia deaths in industrialized
example, in 21 developing countries where U5MRs have countries, especially before Hib and pneumococcal
recently decreased, gaps in mortality rates between the vaccine introduction. Other causes include viruses (eg,
rich and the poor increased. respiratory syncytial virus); other bacteria (eg, Staphylo-
A plethora of studies have documented the strong coccus aureus); and where HIV prevalence is high, Pneu-
correlation between child survival and the mother’s ed- mocystis jiroveci pneumonia (PCP). PCP is responsible
ucation level (Fig. 3). In Bolivia, for example, babies for 5% of childhood pneumonia deaths worldwide, de-
born to women who have no education are 2.5 times spite the availability of effective prophylaxis and treatment.
more likely to die before their first birthdays compared Preventive measures are important in reducing the
with those whose mothers have completed secondary incidence of pneumonia and case fatality rates. Under-
school. Rural residence, slum dwelling, race, and ethnic- nourished children lack adequate immunity and have
ity are other important social determinants of health. more difficulty in clearing secretions due to weakened

Pediatrics in Review Vol.32 No.2 February 2011 e27


Downloaded from http://pedsinreview.aappublications.org. Provided by Pakistan:AAP Sponsored on February 1, 2011
preventive pediatrics global child health

countries toward the end of the


20th century. However, despite the
fact that the largest burden of Hib
and pneumococcal disease has been
and continues to be in developed
countries, serotype coverage in
pneumococcal vaccine has largely
focused on markets in high-income
countries. Furthermore, cost has
been a major impediment for Hib
and pneumococcal vaccine intro-
duction in poor countries. Since
1997, Hib vaccine has gradually
been incorporated into at least 115
national childhood vaccine sched-
ules. Pneumococcal vaccine rollout
is in its infancy in the developing
world. At the time of this writing,
Figure 3. Infant mortality rates by mother’s education in multiple countries. Reproduced
South Africa, Rwanda, and Gambia
from Closing the Gap in a Generation: Health Equity Through Action on the Social
are the only sub-Saharan African
Determinants of Health. Final Report of the Commission on Social Determinants of Health.
Geneva, Switzerland: World Health Organization; 2008. countries to have introduced the
vaccine nationally, although other
countries have plans to do so in the
respiratory muscles. Furthermore, children younger than near future. Measles vaccine plays an important preven-
6 months of age who are exclusively breastfed and chil- tive role because pneumonia is a common sequela of
dren who continue to breastfeed through the second measles infection. Cotrimoxazole prophylaxis is impor-
postnatal year are less likely to develop acute respiratory tant for PCP prevention among HIV-infected individu-
infections. Micronutrients (especially zinc) play an im- als.
portant role in immune function and respiratory infection Indoor air pollution is associated with a 1.8-fold risk
prevention. of contracting pneumonia and is largely caused by burn-
Hib and pneumococcal vaccines were incorporated ing of solid fuels (eg, dung, wood, charcoal) in dwellings
into routine vaccination schedules in industrialized for heat and cooking. These polluting fuels are used
primarily by poor households that
are unable to afford stoves that use
cleaner burning fuels. Efforts to
mitigate this exposure are ongoing
through research to develop afford-
able and acceptable clean energy
systems and through increasing the
utilization of existing methods,
such as solar cookers.
Although prevention plays an
important role in pneumonia con-
trol, treatment with a full course
of appropriate antibiotics remains
a critical intervention to reduce
rates of fatal pneumonia. The term
pneumonia, as used in this article
Figure 4. Coverage of key neonatal survival interventions in the developing world. and customarily in the global child
Reproduced and modified from UNICEF ChildInfo website: http://www.childinfo.org:80/ health field, actually refers to “sus-
newborncare.html. pected pneumonia.” Diagnostic

e28 Pediatrics in Review Vol.32 No.2 February 2011


Downloaded from http://pedsinreview.aappublications.org. Provided by Pakistan:AAP Sponsored on February 1, 2011
preventive pediatrics global child health

test availability, including radiography, is extremely lim-


ited in resource-poor settings. Diagnosis of suspected Glossary of Terms
pneumonia hinges on clinical assessment, based espe-
cially on tachypnea or retractions in a child who has • U5MRⴝunder five mortality rate, also referred to as
child mortality rateⴝnumber of deaths before
cough or cold symptoms.
5 years of age/1,000 live births
Despite increasing resistance to first-line antibiotics • Infant mortality rateⴝnumber of deaths before
(amoxicillin and cotrimoxazole), clinical trials continue 1 year of age/1,000 live births
to demonstrate the efficacy of these inexpensive oral • Neonatal mortality rateⴝnumber of deaths before
drugs, even in the treatment of severe pneumonia. They 28 days of age/1,000 live births
• Absolute povertyⴝliving below a minimum level of
remain the mainstay of recommended treatment in the income or wealth that is necessary for attainment of
World Health Organization (WHO) and UNICEF basic needs (eg, adequate shelter and nutrition) and
guidelines. Prompt and appropriate treatment is critical services (access to education and essential health
to reducing pneumonia deaths and hinges on: 1) parent/ care)
• Relative povertyⴝin a society with large differences
caretaker recognition of tachypnea and retractions as between rich and poor, living at an income or
signs for which prompt care should be sought, 2) ability wealth level significantly less than the societal
to access appropriate health care without delay (ie, ser- average, so those living in relative poverty are
vices are available, geographically proximate, affordable, excluded from the ordinary way of living of most
of good quality, and nondiscriminatory), 3) accurate people around them
• Skilled birth attendantⴝa health professional such
diagnosis and treatment by health workers, and 4) avail- as a midwife, doctor, or nurse who has been trained
ability and affordability of treatment and completion of a to manage normal pregnancies, childbirth, and the
full therapeutic course. However, fewer than one fifth of immediate postnatal period and can identify, manage,
parents/caregivers recognize that fast breathing and re- and refer complications in women and newborns
• Community health workers (CHWs)ⴝpersons living
tractions require urgent medical attention. Only 54% of in a community and selected by their community to
children who contract pneumonia in developing coun- receive basic training to provide preventive health
tries are taken for appropriate health care. An even services, offer counseling on prevention and care-
smaller percentage actually receives antibiotics. Tragi- seeking practices, identify common illnesses, and in
many countries, provide basic health services for
cally, there has been little improvement in these coverage
common problems. CHWs are referred to with a
statistics over the past decade, especially in sub-Saharan variety of names in different settings, including
Africa. community health volunteers or village health
Hundreds of thousands of lives could be saved if workers in Africa, health promoters in Latin America,
pneumonia prevention and treatment interventions were and lady health workers in Pakistan
• Improved water sources are those that are protected
universally available. Universal treatment with antibiotics from fecal contamination and include piped water,
alone could reduce pneumonia mortality by approxi- public taps, standpipes, boreholes, protected wells,
mately 40%. To make this treatment a reality, implemen- protected spring, and rainwater
tation strategies must focus on getting treatment to • Improved sanitation facilities are those that
hygienically separate human waste from human
children in the communities where they live. Studies have
contact, including units that flush or pour-flush into
consistently shown that community health workers sewer or septic systems or pit latrines, ventilated
(CHWs) trained in pneumonia case management can improved pit latrines or pit latrines with slab, and
accurately identify and effectively treat childhood pneu- composting toilets
monia. However, a recent study (2) demonstrated that For more information, see WHO/UNICEF Joint
only 27 of 54 countries that have a high pneumonia Monitoring Programme for Water Supply and
burden have implemented some type of community- Sanitation http://www.wssinfo.org/definitions-methods/
introduction
based pneumonia case management program. One third
of countries have policies that restrict community case
management of pneumonia (eg, prohibitition of CHWs in clinics and hospitals are critical in increasing the num-
in dispensing or prescribing antibiotics). Policies and ber of patients who receive pneumonia treatment.
programs that are conducive to getting interventions to
children who need them the most (eg, the poor, rural Diarrheal Disease
dwellers), efforts to educate families about danger signs Diarrhea is the second leading cause of child death after
of pneumonia, and improved access to and quality of care the neonatal period and accounts for nearly 4 billion

Pediatrics in Review Vol.32 No.2 February 2011 e29


Downloaded from http://pedsinreview.aappublications.org. Provided by Pakistan:AAP Sponsored on February 1, 2011
preventive pediatrics global child health

hygiene could also contribute to


control of diseases such as Guinea
worm, trachoma, and respiratory
infections.
Slightly more than 50% of the
world’s population has access to
piped water (going into their dwell-
ing or yard), while 87% have access to
any type of improved water sources,
up from 77% in 1990. Further
progress is still needed, especially in
sub-Saharan Africa, where only 58%
of the population have access to im-
proved water sources. Much slower
progress has been made in sanitation
coverage. Only 62% of people glo-
bally have access to improved sanita-
tion facilities (compared with 54% in
1990), and coverage in southern
Figure 5. The vicious cycle of infectious diarrhea, malabsorption, and undernutrition.
Asia and sub-Saharan Africa is abys-
mally poor at less than 33%.
cases (all ages) and 1.1 million child deaths annually. Adequate nutrition, including breastfeeding and ade-
Microbiologic causes vary by locale and have not been quate micronutrients, also plays an important role in
well elucidated, especially in developing countries where preventing diarrhea. In addition, new immunizations,
the incidence and mortality rates are highest. A variety of such as rotavirus vaccine, hold promise for preventing
bacteria (eg, Campylobacter, Salmonella, Escherichia coli, diarrhea due to specific agents.
and Shigella), viruses (especially rotavirus), and parasites Key treatment interventions are oral rehydration ther-
(eg, Cryptosporidium, Giardia) cause infectious diarrhea apy (ORT) and zinc. For more than 2 decades, ORT has
and are spread via water, food, utensils, flies, and hands. been the cornerstone of treatment for preventing and
Deaths are due largely to fluid and electrolyte losses.
treating dehydration and electrolyte losses. ORT is esti-
Undernourished children are at greater risk of contract-
mated to save the lives of 1 million children annually.
ing and dying from diarrheal disease. Furthermore, re-
ORT employs prepackaged oral rehydration salts (ORS)
peated and persistent infections can lead to or exacerbate
mixed with water or the use of appropriate other fluids
underlying undernutrition (Fig. 5).
such as homemade solutions. Most recently, the compo-
The incidence of diarrhea and its mortality could be
dramatically reduced by widespread implementation of sition of internationally used ORS has been reformulated
currently available prevention and treatment interven- to a lower osmolarity, which has been found to shorten
tions. Access to “improved” (see Glossary of Terms) the duration of diarrhea and decrease the need for un-
water sources and sanitation facilities as well as sufficient scheduled intravenous fluids compared with the older
quantities of water to facilitate improved hygiene could formulation. International treatment guidelines also rec-
reduce diarrhea-related mortality by up to 88%. Further- ommend a 10- to 14-day course of oral zinc for all
more, widespread coverage of these interventions could children who contract diarrhea and live in areas that have
reduce the impact of other water-related diseases. For a presumed high prevalence of zinc deficiency (ie, most
example, access to improved sanitation facilities reduces developing countries). Treatment with zinc reduces the
the risk of acquisition of helminth infections (eg, hook- severity and duration of illness and decreases the likeli-
worm, whipworm, roundworm) from contact with con- hood of subsequent diarrheal episodes in the following
taminated soil (eg, walking shoeless in at-risk environ- 2 to 3 months.
ments) or ingestion of contaminated food. These Because it is a relatively new recommendation, data
parasites are responsible for significant morbidity, includ- on zinc coverage are limited. However, progress in ORT
ing intestinal blood loss and reduced nutrient absorp- coverage has been sluggish. In 2008, only 38% of chil-
tion. Availability of a sufficient water supply for adequate dren who had diarrhea in developing countries (data

e30 Pediatrics in Review Vol.32 No.2 February 2011


Downloaded from http://pedsinreview.aappublications.org. Provided by Pakistan:AAP Sponsored on February 1, 2011
preventive pediatrics global child health

plicated” malaria in pregnant


women and young children (due to
relative immunosuppression and
lack of acquired immunity, respec-
tively) in areas where P falciparum
cause disease. Severe hemolytic
anemia and cerebral malaria (pre-
senting with altered mental status,
convulsions, or coma) are the most
common forms of complicated ma-
laria and are associated with high
fatality rates. Malaria parasites in-
fect the placenta and contribute to
low birthweight, a major cause of
neonatal mortality.
Reduction in malaria mortality
requires a multipronged approach
consisting of prevention and treat-
Figure 6. Malaria transmission areas and reported P falciparum resistance, 2004. ment interventions. Insecticide
Reproduced from Guidelines for the Treatment of Malaria. Geneva, Switzerland: World treated nets (ITNs) are one of the
Health Organization; 2006. most effective methods of prevent-
ing transmission. Studies have
demonstrated a 17% reduction in
exclude China) received recommended fluid therapy all causes of child mortality in malaria-endemic areas
compared with 32% in 2000. associated with regular ITN use. Not only do ITNs
The unacceptable toll of deaths from diarrhea is a protect sleeping individuals with a physical barrier
tragic reminder of the work that remains in delivering against mosquitoes, but the insecticide also offers protec-
known, effective prevention and treatment to those chil- tion to nonusers up to several hundred meters away.
dren most in need. Initially, ITN implementation strategies focused on cre-
ating demand for ITN purchase. However, even at sub-
Malaria sidized prices, nets were not affordable to those in most
Almost 50% of the world’s population lives in areas where need and at highest risk, and coverage rates were low.
malaria is endemic. The Plasmodium parasites that cause A recent shift to a mass distribution strategy (eg, through
malaria are transmitted by the bite of the Anopheles antenatal care and immunization clinics) has increased
mosquito. Infection with any of the four malaria-causing coverage multifold, although much more progress is
species (P falciparum, P vivax, P malariae, and P ovale) is required to move current 20% coverage rates to 80%
associated with significant morbidity, although the vast coverage goals (set by the WHO World Health Assembly
majority of malaria deaths are due to P falciparum. The in 2005) by 2010.
geographic distribution of Plasmodium species varies; Malaria parasites can become resistant to antimalarial
Figure 6 depicts all areas where transmission from any of drugs quickly; conventional therapies (eg, chloroquine,
the species occurs and specifically denotes areas where P sulfadoxine-pryimethamine) are no longer effective in
falciparum resistance to conventional antimalarials has many areas (Fig. 6). Artemisinin combination therapy
been reported. (ACT), once deemed too costly for wide-scale use, is
People living in malaria-endemic areas frequently ex- now the recommended first-line treatment for uncom-
perience asymptomatic and “uncomplicated” malaria plicated malaria in areas characterized by high resistance
that primarily presents with the flulike symptoms of fever, to conventional therapies. Artemisinin is an effective
headache, and vomiting. Malaria parasites replicate in antimalarial, and when used in combination with another
and lyse hematocytes, causing anemia, which exacts a toll antimalarial agent, progression to resistance is slowed
on physical and mental development in children and dramatically. To prevent rapid development of artemisi-
economic productivity in adults. Untreated uncompli- nin resistance, the WHO recommends against the pro-
cated malaria can rapidly lead to life-threatening “com- duction, marketing, or use of artemisinin oral mono-

Pediatrics in Review Vol.32 No.2 February 2011 e31


Downloaded from http://pedsinreview.aappublications.org. Provided by Pakistan:AAP Sponsored on February 1, 2011
preventive pediatrics global child health

ongoing to identify IPT drugs


that have fewer current and po-
tential resistance issues.

Other Major Causes of Child


Mortality
Measles is a leading cause of
vaccine-preventable mortality,
contributing to 1% of child deaths.
Two doses of vaccine are recom-
mended. The first dose is usually
administered at 9 months of age in
endemic settings. Approximately
80% of 12- to 23-month-old chil-
dren have received at least one
dose. Children infected with mea-
sles are at high risk for secondary
complications, including corneal
ulceration, diarrhea, and pneumo-
nia. Concomitant VA deficiency is
associated with increased morbid-
ity and mortality. VA plays an im-
Figure 7. Trends in U5MR from 1970 to 2008.
portant role in immune and epithe-
lial tissue function. VA deficiency is
common in developing countries,
therapy, especially because there are no effective and measles infection further depletes VA stores. Treat-
treatment alternatives on the market. Unfortunately, oral ment with high-dose VA reduces measles-related mor-
artemisinin monotherapy is available in many countries, bidity and mortality and is recommended for all children
and resistance has recently been reported in Cambodia. who have measles living in areas with high VA deficiency.
Furthermore, even in countries that have adopted ACT Much recent progress has been made in reducing measles
as first-line treatment, few children who have malaria mortality: deaths have declined by 74% and 89% world-
symptoms are receiving the agent. In fact, recent data wide and in sub-Saharan Africa, respectively, between
from sub-Saharan Africa demonstrated that fewer than 2000 and 2007.
60% of children who have had malaria symptoms have AIDS and injuries account for 2% and 3% of childhood
received any type of antimalarial. Updated WHO treat- mortality globally, respectively, but represent higher pro-
ment guidelines recommend that all children presenting portions of deaths, depending on geographic setting. For
with fever and living in malaria-endemic countries be example, injuries account for higher proportions of
tested for malaria infection (by microscopy or rapid deaths in middle-income countries where infectious dis-
diagnostic tests) and treated accordingly. In settings eases are better controlled. Most deaths due to injuries
without testing capacity, febrile children should be are preventable with interventions that employ a multi-
treated for malaria presumptively. disciplinary approach to the problem. AIDS is responsi-
Malaria infection among pregnant women is com- ble for a high proportion of child deaths in many sub-
mon, even when asymptomatic. Intermittent presump- Saharan African countries. Transmission from mother to
tive treatment (IPT) during pregnancy is effective in child accounts for 90% of childhood HIV acquisition and
reducing maternal (eg, anemia) and neonatal (eg, low has been virtually eliminated in industrialized countries.
birthweight) complications of infection by clearing par- This progress is not the case in developing countries due
asites from the bloodstream and placenta. IPT, consist- to poor access to antiretrovirals and less-than-optimal
ing of two doses of oral sulfadoxine-pyrimethamine, is substitutes for breastfeeding due to cost and sanitation
recommended during pregnancy for those residing in issues. Furthermore, only a small fraction of children
endemic areas. IPT for young children holds great meeting criteria for treatment of HIV/AIDS are receiv-
potential for reducing malaria mortality. Research is ing treatment.

e32 Pediatrics in Review Vol.32 No.2 February 2011


Downloaded from http://pedsinreview.aappublications.org. Provided by Pakistan:AAP Sponsored on February 1, 2011
preventive pediatrics global child health

and health systems components. To achieve any sig-


nificant reduction in child mortality through the IMCI
strategy, full attention to all three components is
needed as well as implementation of services on a
larger scale.
Efforts are being made, including by the WHO, to
revitalize the Primary Health Care movement, first ad-
vanced in the 1970s. This movement encourages inte-
grated care but additionally focuses on universal access to
essential medicines and health services and a comprehen-
sive approach to health that includes addressing under-
lying environmental (eg, water and sanitation) and social
determinants of health (eg, education).
Figure 8. Vila Manica District Hospital, Mozambique, 1991.
Reproduced with permission from Stephen Gloyd. Trends in Child Mortality
Tracking of health data globally started in the 1960s.
Child Health Intervention Implementation There has been a continual decline in U5MRs and in
Within the Health System absolute numbers of child deaths annually over the
In the 1990s, in response to programs focused solely on intervening decades. Remarkable gains were seen in
controlling and treating single diseases, WHO and reducing child mortality in the first couple of decades
UNICEF developed the Integrated Management of after commencement of data monitoring, followed by
Childhood Illness (IMCI), a strategy that focuses on a marked diminution in progress, even stagnating and
caring for the whole child. IMCI includes preventive and sometimes reversing course, with increased U5MRs
curative elements, such as monitoring growth, develop- in some countries. Worldwide, U5MRs dropped 20%
ment, and immunization status during all childhood sick in each decade between 1970 and 1990, but they
visits. Effective IMCI implementation includes three dropped only 12% in the 1990s. Data to date from the
components: 1) improving health worker case manage- first decade of the 21st century indicate modest im-
ment skills, 2) improving health practices within families provements in reducing U5MRs compared with the
and communities, and 3) improving overall health sys- previous decade (Fig. 7).
tems. Nonpediatric physicians and nonphysician provid- Sociopolitical and economic factors and policies can
ers of care are trained in IMCI clinical algorithms. These influence child health profoundly. An examination of
types of HCWs, particularly nonphysician providers (eg, trends in global health would not be complete without
nurses, mid-level providers), provide the bulk of child examination of some important sociopolitical and eco-
health services in resource-poor settings. IMCI algo- nomic issues affecting these trends:
rithms focus on identification of common causes of mor-
tality, appropriate treatment, and referral for more spe-
cialized or hospital-based care when necessary. The
community and family practices component encom-
passes education to increase key preventive practices in
the household and community and to ensure that parents
know symptoms for which to seek health care. Last, but
not least, a functioning health system is critical for train-
ing and regular supervision of health workers and for
ensuring timely access to referred care and steady stocks
of drugs, vaccines, equipment, and supplies.
Evaluations of the IMCI strategy conducted in
multiple countries have demonstrated improved per-
formance by health workers and improved clinical care
by those who have taken the IMCI course. However,
the evaluations also showed that less attention has Figure 9. Laboratory technician, Somalia, 1983. Reproduced
been focused on implementing the family/community with permission from Stephen Gloyd.

Pediatrics in Review Vol.32 No.2 February 2011 e33


Downloaded from http://pedsinreview.aappublications.org. Provided by Pakistan:AAP Sponsored on February 1, 2011
preventive pediatrics global child health

• Conflict: Most countries experiencing U5MR in- being given. (For the purposes of context, $16 billion
creases are embroiled in conflict. Since World War II, a month has been spent by the United States on the
civilian deaths, including those of women and chil- direct costs of the wars in Iraq and Afghanistan, and
dren, outnumber combatant deaths, accounting for recent United States stimulus and financial institution
90% of conflict-related deaths and half among those bailout packages were allocated more than $700 bil-
younger than 18 years. Most civilian deaths are non- lion each.) The Millennium Declaration is a compact
trauma-related; rather, they are due to disruptions in between poor and rich countries whereby the latter
civilian infrastructure, including health services. For committed to supporting poor countries to meet
example, pneumonia, diarrhea, and undernutrition MDGs. Extending back to 1970, wealthy countries
rates typically increase during conflict. have pledged and reiterated their pledge to give 0.7%
• Global free trade policies: Free trade policies have of their gross national income as donor aid to help
largely been negotiated to the benefit of rich coun- developing countries achieve basic development goals.
tries. The United Nations estimates that trade rules However, only five countries (Denmark, Luxem-
that are unfair to poor countries deny them $700 bourg, the Netherlands, Norway, and Sweden) have
billion in income every year. For example, subsidies to actually met this pledge. The United States gives the
farmers in rich countries have shut many farmers in most in terms of total dollars, but the least in relation
poor countries out of markets, including in their own to the pledge, donating 0.18% of gross national in-
countries. Other policies have encouraged privatiza- come (compared with 0.3% for 22 Western donor
tion of health care, which leaves the poor with fewer countries overall).
access options. Trade policies on drug patents have • Aid effectiveness and priority setting: There is increas-
resulted in limited access to medicines. ing recognition that more donor aid* without aid
• Debt: Many poor countries are servicing debt burdens effectiveness measures can translate to wasted dollars,
from high-interest rate loans owed to international or worse, negative impact. Donor aid is often allocated
lending agencies, such as the International Monetary to countries not on the basis of need but on the basis
Fund and World Bank. African countries pay more in of strategic interests. For example, less than one third
debt than they receive in donor aid. of United States donor aid goes to the least developed
• International economic policies: Structural adjust- countries. In 2007, the top five recipients of United
ment policies (now often incorporated as part of “pov- States donor aid (Iraq, Afghanistan, Sudan, Columbia,
erty reduction initiatives”) are conditions imposed by and Egypt) received approximately one third of
international lending institutions on loaning money or United States donor funds.
forgiving debt and are intended to rein in borrower Other problems can translate to less effective donor
government spending. As such, these conditionalities aid. For example, aid is increasingly being given
call for reduced subsidies for public services that di- through nongovernmental organizations (both for-
rectly affect those who most need the services. For profit and not-for-profit) based in donor countries
example, as many sub-Saharan countries went from instead of directly to developing countries. These or-
free to fee-based health care and education, utilization ganizations often have high overhead costs, using up
subsequently declined. Many Latin American coun- to 50% of project funding. Furthermore, internation-
tries privatized water sources, with resultant decreases ally run projects often draw away indigenous talent
in water access among the poor. These policies also with substantially higher salaries, leaving local institu-
include reductions in civil service workforces (eg, tions further understaffed. It is also not unusual to
health professionals, teachers) through layoffs. Salary have hundreds of internationally run projects working
caps have effectively reduced buying power because in the same country and not communicating with each
pay cannot keep pace with high inflation rates of many other or with local structures. They often work outside
countries. Slashed funds for improvement and even of national health systems, but this practice only di-
upkeep have resulted in crumbling health facilities verts from and weakens existing systems. Moreover,
(Fig. 8), lack of even basic equipment (Fig. 9), and
reduction in quality of services.
• Unmet development aid promises: High-income *The term donor aid refers to funds that are for the purpose of development to
countries have recognized that to attain development resource-poor countries. Foreign assistance includes donor aid but may also
include military assistance, for example. Health expenditures comprise only 15%
goals, low-income countries require at least $50 bil- of total donor aid, with the remaining going to other areas of development,
lion in external funds in addition to what is currently emergency aid, and debt repayment.

e34 Pediatrics in Review Vol.32 No.2 February 2011


Downloaded from http://pedsinreview.aappublications.org. Provided by Pakistan:AAP Sponsored on February 1, 2011
preventive pediatrics global child health

funders determine where, how, and on what priorities and coverage could reduce child mortality by 63%.
their money will be spent. • Inequity: Although the recent financial crisis has
Donor aid often mandates that recipient countries struck developing countries even harder than devel-
buy donor goods in return. It is not uncommon for oped countries, it takes a few years before the
more than half of donor aid to funnel right back to impact of such events on child survival is exacted
donor countries, diminishing the “effectiveness” of and measured. The economic boom that preceded
aid dollars in producing intended outcomes. Projects the current financial crisis, however, did not neces-
tend to concentrate in capital cities and at national- sarily result in improved child health. In fact, India
level hospitals and less commonly reach underfunded, experienced unprecedented economic growth but
understaffed, more outlying district hospitals, clinics, only weak reductions in child mortality. This out-
and communities. come is in contrast to neighboring Bangladesh,
The last few years have seen an influx of funding for which simultaneously made weak economic strides
high-profile diseases, especially AIDS, tuberculosis, but strong improvements in child survival. Al-
and malaria, scourges certainly worthy of global atten- though national economic stability is an important
tion. However, funding for more common maternal element in the ability to provide resources to health
and childhood killers has been abysmal. For example, systems and programs, studies have indicated that
pneumonia kills more children than AIDS, tuberculo- national wealth and economic growth in the ab-
sis, and malaria combined and yet receives far less sence of equity-oriented strategies do not translate
attention and funding than any of them. The same can to improved child health, especially among the
be said for diarrhea and neonatal problems. poor.
• Unmet government commitment to health: Gov- • Global food crisis: Even before the onset of the
ernments of developing countries have also pledged economic crisis, soaring food prices resulted in
to prioritize health, including by providing re- 100 million more people joining the ranks of the 1
sources for the health sector. For example, African billion people living with hunger. Reasons for in-
heads of state have agreed to devote 15% of domes- creased food prices are multifactorial and include
tic expenditures to the health sector. Malawi has decreased food production due to climatic causes,
made substantial increases in health funding, and diversion of crops for biofuel, and an increasing
Zambia has met the pledge. Most other countries share of grains going to feed domestic animals to
lag far behind their pledges, and many have de- meet growing worldwide consumer demand for
creased the proportion of funds going to the health meat among those in better-off strata of society.
sector. Although there are alarming predictions of a future
• Skewed research priorities: In 1998, the term lack of sufficient food supplies to meet an increasing
“10/90 gap” was coined by the Global Forum for global population, global food supplies currently
Health Research to express that only 10% of health are adequate to meet global nutrition needs. How-
research funding was directed toward the 90% of the ever, sharp price increases have dramatically affected
disease burden that affects the poor globally. The the amount of food that the poor are able to pur-
term continues to be used to express the continued chase, and global rates of hunger have increased.
wide gaps, despite recent increases in global health The full impact on undernutrition rates have yet to
research spending that have largely focused on be measured.
HIV/AIDS, to a lesser extent on tuberculosis and
malaria, and relatively little on neonatal problems, What Will it Take to Meet MDG4?
pneumonia, or diarrhea, which cause more than half MDG4 is based on earlier trends in U5MRs and data
of all child deaths. Furthermore, 97% of child health demonstrating that two thirds of child deaths can be
research funding goes toward the development of averted with interventions that are already available and
new technologies (eg, new drugs or immunizations are recommended for full implementation. MDG4 is
to prevent or treat disease) that have the potential to not “pie in the sky”; it is an achievable goal. However,
reduce child mortality by 22%. Only 3% of child examination of current trends in child health demon-
health research dollars are spent on research to strates that the world is alarmingly far from meeting its
determine how to increase coverage rates of inter- MDG4 commitment (Fig. 10). Perhaps most distressing
ventions and get effective services delivered to chil- is that of 64 countries with the highest U5MRs, only
dren who need them the most. Increasing delivery nine are on track to meet MDG4.

Pediatrics in Review Vol.32 No.2 February 2011 e35


Downloaded from http://pedsinreview.aappublications.org. Provided by Pakistan:AAP Sponsored on February 1, 2011
preventive pediatrics global child health

need and are consistent with in-


country priorities and interna-
tional care guidelines. Through
careful consideration of ethical
engagement in participatory and
sustainable global health work to
address local needs and build lo-
cal health institution and health
professional capacity, individuals
and organizations can avoid un-
intended negative consequences
and provide maximal benefit.
The contribution of American
pediatricians to improving global
health is not limited to work on
foreign soil. Pediatricians can advo-
cate for much needed accountabil-
ity among the growing number of
players in global health, making
certain that: 1) major causes of
Figure 10. Accelerated improvements in child mortality are needed to meet MDG4. childhood morbidity and mortality
are no longer neglected; 2) re-
MDG4 is attainable, even in the poorest countries, search agendas expand to determine how best to deliver
as evidenced by remarkable progress in several high- available, effective interventions and services to those in
mortality countries: Nepal, Bangladesh, Eritrea, Loa most need; 3) international initiatives have a more com-
People’s Democratic Republic, Mongolia, Bolivia, and prehensive and participatory outlook and are less ori-
Malawi. (3) Malawi, for example, has reduced its ented toward single diseases; and 4) research and service
U5MR by 53% over a 17-year period. Resource-poor programs avoid contributing to the further collapse of
countries making great strides in child health are doing health systems, but rather work in conjunction with
so through equity-oriented national policies or ministries of health and their clinical institutions, such as
through targeting of simple, effective, inexpensive in-
district hospitals and clinics, to facilitate the strengthen-
terventions (such as antibiotics for pneumonia, ORT
ing of health systems.
for diarrhea, improved home care for newborns, and
Perhaps the most effective approach for American
ITNs to prevent malaria) to those who need them the
pediatricians to make an impact is by supporting policies
most, especially poor children and those living in rural
that address poverty, the major underlying cause of child
areas. These real-world case studies provide optimism
that dramatic improvements in child survival can be deaths globally. Such policies include meeting United
attained. Long-term inroads in promoting child health States donor aid pledges and effectively allocating aid
additionally require addressing the underlying deter- based on need, debt relief, elimination of structural ad-
minants of health. justment policies, and transparent trade proceedings and
trade policies that are fair to poor countries. Surveys have
Opportunities for Progress demonstrated that most Americans are concerned about
American pediatricians can and do play an important global child health, but there are many misconceptions
role in improving the lives of children in poor coun- about the nature of global health problems and activities.
tries, which is the majority of children in the world. Pediatricians can play an important educational role to
Many pediatricians are working to improve child sur- aid community members in becoming better informed
vival and health through overseas work or are support- advocates.
ing organizations that are on the ground in resource- Just as American pediatricians have played and continue
limited settings. On-site work can contribute greatly to play a vital role in advocacy for American children, we can
(eg, through training and clinical work), especially support children everywhere through advocacy on their
when services are delivered to those in the greatest behalf.

e36 Pediatrics in Review Vol.32 No.2 February 2011


Downloaded from http://pedsinreview.aappublications.org. Provided by Pakistan:AAP Sponsored on February 1, 2011
preventive pediatrics global child health

9. Schell CO, Reilly M, Rosling H, Peterson S, Ekstrom AM.


Summary Socioeconomic determinants of infant mortality: a worldwide study
of 152 low-, middle-, and high-income countries. Scand J Public
Health. 2007;35:288 –297
• Approximately 8 million children die annually (based
on 2009 data), and 99% of child deaths occur in 10. Victora CG, Wagstaff A, Schellenberg JA, Gwatkin D, Claeson
developing countries. (3) M, Habicht JP. Applying an equity lens to child health and mortal-
• Eighty-four percent of global child deaths are due to ity: more of the same is not enough. Lancet. 2003;362:233–241
seven major causes: neonatal problems, acute 11. Bhutta ZA, Ahmed T, Black RE, et al. What works? Interven-
respiratory infections, diarrhea, malaria, measles, tions for maternal and child undernutrition and survival. Lancet.
injuries, and HIV/AIDS. (4) 2008;371:417– 440
• Undernutrition is an underlying cause in 35% of 12. Darmstadt GL, Bhutta ZA, Cousens S, Adam T, Walker N, de
child deaths. (5) Bernis L. Evidence-based, cost-effective interventions: how many
• Determinants of child mortality include lack of newborn babies can we save? Lancet. 2005;365:977–988
access to improved water sources and sanitation 13. World Health Report: Make Every Mother and Child Count.
facilities, lack of access to care, lack of maternal Geneva, Switzerland: World Health Organization; 2005
education, absolute poverty, and relative poverty. 14. De Vogli R, Birbeck GL. Potential impact of adjustment
(6)(7)(8)(9)(10) policies on vulnerability of women and children to HIV/AIDS in
• Two thirds of child deaths could be prevented with sub-Saharan Africa. J Health Pop Nutr. 2005;23:105–120
interventions that are currently available, effective, 15. Labonte R, Schrecker T. Globalization and social determinants
and feasible for widespread implementation (1)(11) of health: promoting health equity in global governance (part 3 of
(12) as well as consistent with MDG4. However, 3). Global Health. 2007;3:7
trends in decreasing child mortality have slowed, 16. Labonte R, Schrecker T. Globalization and social determinants
(13) and based on current trends, the world is not of health: the role of the global marketplace (part 2 of 3). Global
on track to meet MDG4. Health. 2007;3:6
• Sociopolitical and economic factors and policies have a 17. Labonte R, Schrecker T. Globalization and social determinants
tremendous influence on health and the determinants of health: introduction and methodological background (part 1 of
of health (6)(13)(14)(15)(16)(17)(18)(19) 3). Global Health. 2007;3:5
18. O’Hare BA, Southall DP. First do no harm: the impact of
recent armed conflict on maternal and child health in Sub-Saharan
Africa. J R Soc Med. 2007;100:564 –570
ACKNOWLEDGMENTS. Grateful thanks to Drs Tessa 19. Impact of Armed Conflict on Children. New York, NY:
Wardlaw and Cynthia Boschi-Pinto for assistance with UNICEF; 1996
epidemiologic data; Drs Charles Mock and Elinor Gra-
ham for thoughtful manuscript review; and Elizabeth
Wolf for manuscript preparation.
Suggested Reading
General References on Global Child Health:
References 2003 Lancet Child Survival Series:
1. Jones G, Steketee RW, Black RE, Bhutta ZA, Morris SS. How Bryce J, el Arifeen S, Pariyo G, Lanata C, Gwatkin D, Habicht JP.
many child deaths can we prevent this year? Lancet. 2003;362:65–71 Reducing child mortality: can public health deliver? Lancet.
2. Marsh DR, Gilroy KE, Van de Weerdt R, Wansi E, Qazi S. 2003;362:159 –164
Community case management of pneumonia: at a tipping point? Black RE, Morris SS, Bryce J. Where and why are 10 million
Bull WHO. 2008;86:381–389 children dying every year? Lancet. 2003;361:2226 –2234
3. You D, Wardlaw T, Salama P, Jonesw G. Levels and trends in Claeson M, Gillespie D, Mshinda H, Troedsson H, Victora CG.
under-5 mortality, 1990 –2009. Lancet. 2010;376:931–933 Knowledge into action for child survival. Lancet. 2003;362:
4. UNICEF. Childinfo Child Mortality website. Accessed Novem- 323–327
ber 2010 at: http://www.childinfo.org:80/undernutrition.html Jones G, Steketee RW, Black RE, Bhutta ZA, Morris SS. How many
5. Black RE, Allen LH, Bhutta ZA, et al. Maternal and child child deaths can we prevent this year? Lancet. 2003;362:65–71
undernutrition: global and regional exposures and health conse- Victora CG, Wagstaff A, Schellenberg JA, Gwatkin D, Claeson M,
quences. Lancet. 2008;371:243–260 Habicht JP. Applying an equity lens to child health and mortality:
6. Closing the Gap in a Generation: Health Equity Through Action more of the same is not enough. Lancet. 2003;362:233–241
on the Social Determinants of Health. Final Report of the Commission
on Social Determinants of Health. Geneva, Switzerland: World The State of the World’s Children 2010: Child Rights. New York, NY:
Health Organization; 2008 UNICEF; 2009. UNICEF publishes an annual State of the
7. Cleland JG, Van Ginneken JK. Maternal education and child World’s Children Report that includes indicators on child health
survival in developing countries: the search for pathways of influ- and development. Each report covers a special theme. The 2008
ence. Soc Sci Med. 1988;27:1357–1368 report focused on child survival, the 2009 report on maternal
8. Levine RA, Rowe ML. Maternal literacy and child health in and newborn care, and the 2010 special edition on the UN
less-developed countries: evidence, processes, and limitations. J Dev Convention on the Rights of the Child. Accessed November
Behav Pediatr. 2009;30:340 –349 2010 at: http://www.unicef.org/sowc/

Pediatrics in Review Vol.32 No.2 February 2011 e37


Downloaded from http://pedsinreview.aappublications.org. Provided by Pakistan:AAP Sponsored on February 1, 2011
preventive pediatrics global child health

World Health Report: Make Every Mother and Child Count. Geneva, IMCI Clinical Algorithms:
Switzerland: World Health Organization; 2005 Integrated Management of Childhood Illness – Standard Chart
General Clinical Information: Booklet. Geneva, Switzerland: World Health Organization; 2008
Managing Newborn Problems: A Guide for Doctors, Nurses, and Mid- Integrated Management of Childhood Illness for High HIV Settings
wives. Geneva, Switzerland: World Health Organization; 2003. Chart Booklet. Geneva, Switzerland: World Health Organiza-
Pocket Book of Hospital Care for Children: Guidelines for the Man- tion; 2008
agement of Common Illnesses With Limited Resources. Geneva, Primary Health Care:
Switzerland: World Health Organization; 2005 Declaration of the 1978 Alma-Ata International Conference on
Disease-specific References: Primary Health Care. Accessed November 2010 at: www.
Diarrhea: Why Children are Still Dying and What Can Be Done About who.int/publications/almaata_declaration_en.pdf
It. Geneva, Switzerland: World Health Organization, UNICEF; World Health Report: Primary Health Care: Now More than Ever.
2009 Geneva, Switzerland: World Health Organization; 2008
Guidelines for the Treatment of Malaria. 2nd ed. Geneva, Switzer- Socio-political-economic Issues and Policies Affecting Global
land: World Health Organization; 2010 Health:
Wardlaw T, Johansson E, Hodge M. Pneumonia: The Forgotten Farmer P. Pathologies of Power: Health, Human Rights, and the New
Killer of Children. Geneva, Switzerland: World Health Organi- War on the Poor. Berkeley, CA: University of California Press; 2003
zation, UNICEF; 2006
Fort MP, Mercer MA, Gish O, eds. Sickness and Wealth: The
World Report on Child Injury Prevention. Geneva, Switzerland:
Corporate Assault on Global Health. Cambridge, MA: South
World Health Organization; 2008
End Press; 2004
UNICEF ChildInfo website. http://www.childinfo.org/index.html
Global Health Watch 2: An Alternative World Health Report.
2005 Lancet Neonatal Survival series: London, United Kingdom: People’s Health Movement; 2008
Lawn JE, Cousens S, Zupan J. 4 million neonatal deaths: When? Kim JY. Dying for Growth: Global Inequality and the Health of the
Where? Why? Lancet. 2005;365:891–900 Poor. Monroe, ME: Common Courage Press; 2000
Darmstadt GL, Bhutta ZA, Cousens S, Adam T, Walker N, de
Ethical Considerations in Global Health and Development and
Bernis L. Evidence-based, cost-effective interventions: how
Research:
many newborn babies can we save? Lancet. 2005;365:977–988
Leroy JL, Habicht JP, Pelto G, Bertozzi SM. Current priorities in
Knippenberg R, Lawn JE, Darmstadt GL, et al. Systematic scaling
health research funding and lack of impact on the number of
up of neonatal care in countries. Lancet. 2005;365:1087–1098
child deaths per year. Am J Public Health. 2007;97:219 –223
Martines J, Paul VK, Bhutta ZA, et al. Neonatal survival: a call for
NGO Code of Conduct for Health Systems Strengthening. Accessed
action. Lancet. 2005;365:1189 –1197
November 2010 at: http://ngocodeofconduct.org/
2008 Lancet Maternal and Child Undernutrition series: Organisation for Economic Co-operation and Development. The
Bhutta ZA, Ahmed T, Black RE, et al. What works? Interventions Paris Declaration and Accrea Agenda for Action on Aid Effec-
for maternal and child undernutrition and survival. Lancet. tiveness. 2005. Accessed November 2010 at: http://www.oecd.
2008;371:417– 440 org/document/18/0,2340,en_2649_3236398_35401554_
Black RE, Allen LH, Bhutta ZA, et al. Maternal and child under- 1_1_1_1,00.html
nutrition: global and regional exposures and health conse- Suchdev P, Ahrens K, Click E, Macklin L, Evangelista D, Graham
quences. Lancet. 2008;371:243–260 E. A model for sustainable short-term international medical
Bryce J, Coitinho D, Darnton-Hill I, Pelletier D, Pinstrup- trips. Ambul Pediatr. 2007;7:317–320
Andersen P. Maternal and child undernutrition: effective action
Tracking MDGs:
at national level. Lancet. 2008;371:510 –526
Countdown to 2015: Maternal, Newborn and Child Survival
Morris SS, Cogill B, Uauy R. Effective international action against
website. Accessed November 2010 at: http://www.
undernutrition: why has it proven so difficult and what can be
countdown2015mnch.org/index.php
done to accelerate progress? Lancet. 2008;371:608 – 621
United Nations Development Programme MDGs website. Ac-
Victora CG, Adair L, Fall C, et al. Maternal and child undernutri-
cessed November 2010 at: http://www.undp.org/mdg/
tion: consequences for adult health and human capital. Lancet.
2008;371:340 –357 Opportunities for Involvement in Global Child Health:
American Academy of Pediatrics Section on International Child
Determinants of Health:
Health (SOICH) “is committed to improving the health and
Closing the Gap in a Generation: Health Equity Through Action on
well-being of the world’s children. . .through education, advo-
the Social Determinants of Health. Final Report of the Commis-
cacy, research, service, and the facilitation of effective global
sion on Social Determinants of Health. Geneva, Switzerland:
partnerships.” The website lists opportunities and information
World Health Organization; 2008
related to these areas. Accessed November 2010 at: http://
IMCI: www.aap.org/sections/ICH/default.cfm
World Health Organization Child and Adolescent Health and Dueger C, O’Callahan C, eds. Working in International Child
Development Department website on IMCI. Accessed Novem- Health. 2nd ed. Elk Grove Village, IL: American Academy of
ber 2010 at: http://www.who.int/child_adolescent_health/ Pediatrics Section on International Child Health; 2008. A prac-
topics/prevention_care/child/imci/en/index.html tical guide for volunteering and working overseas

e38 Pediatrics in Review Vol.32 No.2 February 2011


Downloaded from http://pedsinreview.aappublications.org. Provided by Pakistan:AAP Sponsored on February 1, 2011
Global Child Health
Donna Denno
Pediatr. Rev. 2011;32;e25-e38
DOI: 10.1542/pir.32-2-e25

Updated Information including high-resolution figures, can be found at:


& Services http://pedsinreview.aappublications.org/cgi/content/full/32/2/e25

Subspecialty Collections This article, along with others on similar topics, appears in the
following collection(s):
Infectious Diseases
http://pedsinreview.aappublications.org/cgi/collection/infectious
_diseases Nutrition and Nutritional Disorders
http://pedsinreview.aappublications.org/cgi/collection/nutritional
_disorders Preventive Pediatrics
http://pedsinreview.aappublications.org/cgi/collection/preventive
_pediatrics Fetus and Newborn Infant
http://pedsinreview.aappublications.org/cgi/collection/fetus_new
born_infant
Permissions & Licensing Information about reproducing this article in parts (figures,
tables) or in its entirety can be found online at:
http://pedsinreview.aappublications.org/misc/Permissions.shtml
Reprints Information about ordering reprints can be found online:
http://pedsinreview.aappublications.org/misc/reprints.shtml

Downloaded from http://pedsinreview.aappublications.org. Provided by Pakistan:AAP Sponsored on February 1, 2011

Vous aimerez peut-être aussi